Review Multiple Choice

Pataasin ang iyong marka sa homework at exams ngayon gamit ang Quizwiz!

When can a mortgagor exercise her statutory right of redemption?

After the foreclosure sale

The Appointments Clause

Permits congress to vest appointments of inferior officers only in the president, the courts or heads of departments Enforcement is an executive act thus congress cannot appoint members of a commission that exercise enforcement powers

Notice Recording Statute

Under a notice statute, a later purchaser of land will prevail over an earlier grantee if she takes without actual or constructive (e.g., record) notice of the earlier grant.

What interest does "jointly, as tenants in common." create?

Tenants in common because there is no mention of right of survivorship

Equitable redemption is the right of a mortgagor to recover the land by paying

amount overdue on the mortgage, plus interest, at any time before the foreclosure sale.

Breach of Merchantability?

In every sale of goods, unless expressly disclaimed, there arises a warranty that the goods will be merchantable, which means that they will be fit for the ordinary purposes for which such goods are used.

What forum is a court considered?

Limited Public Forum or nonpublic forum, so the gov't can regulate speech in such forum to reserve the forum for its intended use. They will be upheld as long as 1. viewpoint neutral and 2. reasonably related to a legit gov't purpose

Easement by implication

created by operation of law rather than by written instrument. It is an exception to the Statute of Frauds. An easement is implied if, prior to the time the property is divided, a use exists on the "servient part" that is reasonably necessary for the enjoyment of the "dominant part," and the parties intended the use to continue after division of the property. The use must be continuous and apparent at the time the property is divided. Reasonable necessity is determined by many factors, including the cost and difficulty of alternatives, and whether the price paid reflects the expected continued use.

What is a profit and how can it be terminated?

nonpossessory interest in land, allowing the grantee to enter on the land and remove resources of the land, Terminated by nonpossessory interest in land, allowing the grantee to enter on the land and remove resources of the land,

What are the 6 covenants in a warrant deed?

1. Right to seisin 2. Right to convey 3. A covenant against encumbrance 4. a covenant against quiet enjoyment 5. covenant of further assurances 6. general warranty

During the trial of her personal injury action against a chemical company, the plaintiff testifies in response to a question by her own counsel that, shortly after she and her family were forced to leave their home because of fumes from its plant, the president of the chemical company telephoned her motel room and said, "If you or any member of your family requires medical treatment, our company will pay all medical expenses in full. We will not have it said that our company's negligence resulted in the illness of a local family." The company's counsel makes a motion to strike all of the plaintiff's testimony, and the court does so. Was the court's action correct? A) Yes, because the testimony relates to inadmissible hearsay. B) Yes, because the statement was made in connection with an offer to pay medical expenses. C) No, because the statement includes an opposing party's statement that it was negligent. D) No, because the statement is a factual admission made in connection with an offer to compromise.

C The court's action was not correct. Federal Rule 409 excludes offers to pay medical expenses, but not statements made in connection with such offers.

On February 1, the owner of a bowling alley read in a magazine an ad from a major manufacturer of bowling balls offering sets of 40 balls in various weights and drilled in various sizes for $10 per ball. The owner immediately filled out the order form included in the ad for the 40 balls and deposited it, properly stamped and addressed, into the mail. On February 2, the bowling alley owner received in the mail a letter from the manufacturer, sent out as part of its advertising campaign, stating in relevant part that it will sell the bowling alley owner 40 bowling balls at $10 per ball. A day later, on February 3, the manufacturer received the bowling alley owner's order. On February 4, the balls were shipped. On what day did an enforceable contract arise?

February 4, the day the balls were shipped That an offer can be accepted by performance or promise to perform unless the offer clearly limits the method of acceptance. Here the offer would be the bowling alley owner's order because the ad is merely solicitation rather than an offer.

A defendant was involved in an accident in which her car struck the rear end of the car driven by the plaintiff. The police issued tickets to the defendant, charging her with reckless driving and speeding. When the defendant's case came before the traffic court, her attorney entered into a plea bargain with the prosecutor. Under the plea bargain, the defendant agreed to plead guilty to speeding and to pay a fine of $100, and the prosecution agreed to drop the reckless driving charge. Accordingly, the defendant pleaded guilty and the court fined her $100. In the later civil suit, where the plaintiff is seeking damages from the defendant for personal injuries, is the guilty plea before the traffic court admissible?

Yes because it is an opponent party's statement plea of guilty to a traffic infraction is a formal judicial statement. The statement is conclusive in a prosecution for that infraction, but if the plea is used in another proceeding, it is merely an evidentiary statement (i.e., it is not conclusive and can be explained). Here, the defendant has acknowledged by her guilty plea that she was speeding at the time of the accident. This fact is relevant to the plaintiff's suit for personal injuries because it increases the likelihood that the defendant was at fault in the accident that caused those injuries. Therefore, the defendant's guilty plea is admissible in the current civil action as an evidentiary statement.

A buyer purchased a parcel of land from a seller for $500,000. The buyer financed the purchase by obtaining a loan from the seller for $300,000 in exchange for a mortgage on the land. The seller promptly and properly recorded his mortgage. Shortly thereafter, the buyer gave a mortgage on the land to a creditor to satisfy a preexisting debt of $100,000 owed to the creditor. The creditor also promptly and properly recorded its mortgage. Within a year, the buyer stopped making payments on both mortgages, and the seller brought an action to foreclose on his mortgage. The creditor was not included as a party to the foreclosure action. The seller purchased the property at a public foreclosure sale in satisfaction of the loan. The creditor subsequently discovered the sale and informed the seller that it was not valid. Who has title to the land?

The seller but he must redeem the creditor's mortgage to avoid foreclosure The seller has title to the land, but he must redeem the creditor's mortgage to avoid foreclosure. As a general rule, the priority of a mortgage is determined by the time it was placed on the property. When a mortgage is foreclosed, the purchaser at the sale will take title as it existed when the mortgage was placed on the property. Thus, foreclosure will terminate interests junior to the mortgage being foreclosed but will not affect senior interests. However, if a lien senior to that of a mortgagee is in default, the junior mortgagee has the right to pay it off (i.e., redeem it) to avoid being wiped out by its foreclosure. Thus, those persons with interests subordinate to those of the foreclosing party are necessary parties to the foreclosure action. Failure to include a necessary party results in the preservation of that party's interest despite foreclosure and sale. Hence, the seller's failure to include the creditor as a party to the foreclosure action preserved the creditor's mortgage on the property. To avoid the creditor's foreclosing (because the buyer was in default of the creditor's mortgage as well), the seller will need to pay off the creditor's mortgage.

A landowner was the owner in fee simple of a tract of land. The landowner conveyed the tract to her neighbor, "for life and then to the first child of my neighbor's only sister who shall reach the age of 21." The sister was unmarried and childless at the time of the conveyance. Five years later, the sister married, and she gave birth to a son the following year. When the son reached the age of 21, he brought an action in ejectment against the neighbor, who is still alive and living on the tract of land. What interest does that son currently have, and what is the neighbors defense if the son wishes to eject her?

The son has a indefeasible vested remainder because he is 21, when the interest was created he did have a contingent remained because he was not 21. And the neighbors life estate was not subject to termination during her life time

The defendant is charged with the battery of a bouncer at a local tavern. At the trial, the prosecutor introduces evidence that while the bouncer was attempting to question the defendant about her intoxicated demeanor, the defendant committed a battery on the bouncer. The defendant attempts to defend against the charge on the basis of self-defense, insisting that the bouncer used excessive force in stopping her from entering the tavern. The defendant attempts to introduce into evidence an authenticated copy of the tavern records that show that three patrons had written complaints against the bouncer within the past six months for the use of excessive force. The prosecutor objects on the grounds that the records are inadmissible character evidence. Should the court sustain the objection?

Yes, because the character evidence of a victim can be established only by reputation or opinion evidence Defendant may introduce bad character trait of a victim but only if it relates to the charge or the defense and limited to OPINION or REPUTATION evidence Evidence of specific acts of the person permitted in specific instances, such as if the acts are relevant to some issues other than disposition to commit the crime charged- here no issues raised by this evidence other than the bouncer's propensity to use excessive force.

A skier broke his leg when he was knocked down by the chair lift as he tried to avoid other skiers who had fallen off while disembarking. The ski resort employee operating the lift had not been paying attention and had failed to stop the lift. Ski patrol personnel placed the skier on a stretcher, which they then hooked up to a snowmobile to bring him down the mountain. The route down ran along the edge of a ski trail. Midway down, a novice snowboarder tried to see how close he could come to the stretcher without hitting it, but he lost control and landed on top of the skier's leg, damaging it further. The skier filed a lawsuit against the snowboarder and the resort in a jurisdiction that has adopted a comparative contribution system in joint and several liability cases. At trial, the skier's physician testified that the skier's leg was permanently disabled, but that neither injury, by itself, would have caused the permanent disability and it was impossible to quantify how much each injury contributed to the disability. The jury determined that the damages from the permanent disability equaled $2 million, and that the snowboarder and the resort were each 50% at fault. What amount of damages can the skier recover from the snowboarder for his permanent disability?

$2 million because it was not possible to identity the proportion of injury the snowboarder caused

A breeder of quarter horses entered into an agreement with a rancher to sell and deliver two quarter horses, one to the rancher and the other to the rancher's fiancée as a gift. Although the fair market value of each horse was $3,000, the horse breeder agreed to sell both horses together for a total price of $5,000. Under the agreement that the rancher wrote out and both parties signed, the horse breeder agreed to deliver one horse to the rancher on August 1, at which time the rancher agreed to pay the horse breeder $5,000. The horse breeder further agreed to deliver the other horse to the rancher's fiancée on August 12. On August 1, the horse breeder delivered the first horse to the rancher and, at the same time, the rancher gave the horse breeder a certified check for $5,000. On August 12, the horse breeder brought the second horse to the residence of the rancher's fiancée and told her that the horse was a gift from the rancher. The rancher's fiancée told the horse breeder that she loathed quarter horses and she refused to take the horse. The horse breeder brought this horse back to his farm and sent an e-mail to the rancher, informing him that his fiancée refused delivery and that he (the horse breeder) could not keep the horse. Two weeks later, after not hearing from the rancher, the horse breeder sold the horse to an interested party for $3,000. If the rancher sues the horse breeder, how much should the rancher recover?

$2,000, the difference between the value of the horse delivered to the rancher and what the horse breeder received from the rancher. that is the amount by which the horse breeder would be unjustly enriched. In a proper tender of delivery under UCC section 2-503, the seller must put and hold conforming goods at the buyer's disposition for a time sufficient for the buyer to take possession. The seller must give the buyer notice reasonably necessary to enable him to take possession of the goods. Proper tender of delivery entitles the seller to acceptance of the goods and to payment according to the contract. [UCC §2-507] Having made a proper tender of delivery at the place designated by the rancher and having notified the rancher of his fiancée's nonacceptance, the horse breeder has discharged his duty under the contract. When a party's duty of performance is discharged, the other party is entitled to restitution of any benefits that he has transferred to the discharged party in an attempt to perform on his side. With the horse breeder's contractual duty to deliver the second horse to the rancher's fiancée discharged, the horse breeder would be unjustly enriched, to the detriment of the rancher, if he were permitted to keep the entire $5,000 paid to him by the rancher. The rancher conferred a benefit upon him by paying him $5,000 in exchange for two horses, one of which was to be delivered to the rancher, the other to the rancher's fiancée. Because delivery to the fiancée cannot be accomplished, the rancher finds himself in a position of having paid $5,000 for one horse, the fair market value of which is $3,000. Thus, if the horse breeder is permitted to retain the sum of $5,000, he will be unjustly enriched by $2,000

A general contractor about to bid on a construction job for an office building invited a carpenter and several others to bid on the carpentry work. The carpenter agreed to bid if the general contractor would agree to give the carpenter the job provided that his bid was lowest and the general contractor was awarded the main contract. The general contractor so agreed. The carpenter, incurring time and expense in preparing his bid, submitted the lowest carpentry bid. The general contractor used the carpenter's bid in calculating its own bid, which was successful. Which of the following best supports the carpenter's position that the general contractor is obligated to award the carpentry subcontract to the carpenter? A) The carpenter detrimentally relied on the general contractor's conditional promise in preparing his bid. B) The carpenter gave consideration for the general contractor's conditional promise to award the carpentry subcontract to the carpenter. C) The general contractor has an implied duty to deal fairly and in good faith with all bidders whose bids the general contractor used in calculating its main bid to the building owner. D) The general contractor has an obligation to the owner of the building to subcontract with the carpenter because the carpenter's bid was used in calculating the general contractor's bid, and the carpenter is an intended beneficiary of that obligation.

(B) is correct. The carpenter's bid was consideration for the general contractor's promise to award the carpentry subcontract to the carpenter if his bid was the lowest and the general contractor was awarded the main contract. Thus, the general contractor and the carpenter formed a contract. Two elements are necessary to constitute consideration. First, there must be a bargained-for exchange between the parties; and second, that which is bargained for must constitute a benefit to the promisor or a detriment to the promisee. The carpenter and the general contractor agreed that the carpenter would supply a bid that the general contractor could use in its own bid. The carpenter's bid was bargained for and was a benefit to the general contractor, so it constitutes consideration sufficient to support the general contractor's conditional promise to award the subcontract to the carpenter. Conditional promises are enforceable, but the duty to perform does not become absolute until the condition has been met or is legally excused. The conditions in this contract were met-the carpenter's bid was the lowest and the general contractor was awarded the main contract. Thus, the general contractor is under a duty to perform his promise to award the subcontract to the carpenter. (A) is not the best answer. The carpenter is seeking specific performance of their agreement so that he will be awarded the carpentry subcontract. If the carpenter uses a detrimental reliance or promissory estoppel argument, he would be conceding that he gave no consideration and there is no contractual obligation, but he should be awarded damages to prevent injustice. Courts will often limit damages under this theory to reliance damages, which could be much less than the value of the subcontract.

Seven years ago, a man conveyed vacant land by warranty deed to a woman, a bona fide purchaser for value. The woman did not record the warranty deed and did not enter into possession of the land. Five years ago, the man conveyed the same land to a neighbor, also a bona fide purchaser for value, by a quitclaim deed. The neighbor immediately recorded the quitclaim deed and went into possession of the land. Two years ago, the neighbor conveyed the land to a friend, who had notice of the prior conveyance from the man to the woman. The friend never recorded the deed but went into immediate possession of the land. The jurisdiction has a notice recording statute and a grantor-grantee index system. If the woman sues to eject the friend, will the woman be likely to succeed? A) No, because the friend took possession of the land before the woman did. B) No, because the neighbor's title was superior to the woman's title. C) Yes, because the friend had notice of the conveyance from the man to the woman. D) Yes, because the woman, unlike the friend, took title under a warranty deed.

(B) is correct. Under a notice recording system, a subsequent bona fide purchaser ("BFP") prevails over a prior grantee who failed to record. A BFP is a purchaser who gives valuable consideration and has no notice of the prior grant. Notice includes actual, record, or inquiry notice. Also, under the shelter rule, a person who takes from a BFP will prevail against any interest that the transferor-BFP would have prevailed against. This is true even where the transferee had actual knowledge of the prior unrecorded interest. Here, the neighbor was a BFP. The woman had not recorded her deed and had not taken possession of the land; the neighbor gave value without any notice of the woman's claim. The neighbor's title was superior to the woman's title because of the recording statute. When the neighbor sold the land to the friend, the friend was protected under the shelter rule despite having actual knowledge of the woman's interest. Thus, the friend's title is also superior to that of the woman.

A young woman who recently graduated from college landed her dream job teaching kindergarten at the same elementary school she had attended as a child. The young woman's contract provided that she would be paid $40,000 for the school year, and that she could be fired only for just cause. Days before the young woman was to begin teaching, the school's principal fired her, without cause, so that he could hire his cousin for the job instead. The young woman submitted her resume to an employment agency, but was so depressed over the loss of her dream job that she turned down a nearby school's offer for a similar job paying a $30,000 salary, and instead spent the rest of the school year miserably unemployed. In an action by the young woman against the elementary school for damages, which of the following awards is most likely?

10K When there is a breach of an employment contract by the employer, the standard measure of the employee's damages is based on the full contract price. However, a nonbreaching party cannot recover damages that could have been avoided with reasonable effort. If a breaching employer can prove that a comparable job in the same locale was available, the damages against the employer will be reduced by the wages the plaintiff would have received in that comparable job. Here, the full contract price is $40,000, but the young woman could have avoided $30,000 damages by accepting the job at the nearby school. Thus, her damages will be reduced by $30,000, leaving her with $10,000 in damages.

A young woman went to her local shoe shop and selected a pair of shoes. She gave the salesperson cash for the shoes. As the salesperson was putting the shoes into a bag, a robber brandishing a gun entered the store, forced the salesperson to put all of the money in the register into the bag with the shoes, and fled with the bag, the money, and the shoes. After the police had come, the young woman asked the salesperson to get her another pair of shoes. He told the young woman that she would have to pay for them again. The young woman refused. If the young woman sues the shoe shop for another pair of shoes, who will prevail? A) The young woman, because she did not yet have possession of the shoes. B) The young woman, because the purpose of the contract had been made impossible by an unforeseen event. C) The shoe shop, because title to the shoes had already passed to the young woman. D) The shoe shop, because the contract goods had already been identified.

A

A seller contracted to manufacture 1,000 toasters for a buyer for a specified price. The contract contained a provision that clearly stated: "This contract may not be assigned, and any violation of this prohibition voids the contract." After the contract was signed, the seller informed the buyer that the toasters would be manufactured by a competitor of the seller. Citing the non-assignment provision, the buyer claimed that it was no longer bound by the contract. Toasters manufactured by the competitor were of equal quality to toasters manufactured by the seller. Is the buyer bound by the contract? A) No, because "this contract may not be assigned" means that duties may not be delegated, and the seller delegated a duty. B) No, because the seller assigned a right despite the contractual prohibition. C) Yes, because even though the seller breached the contract, there are no damages since the competitor's toasters are of equal quality to the seller's toasters. D) Yes, because the non-assignment provision is not enforceable since public policy favors free assignment and delegation.

A (A) is correct. A contract clause prohibiting the assignment of the contract will be construed as barring only the delegation of the assignor's duties. Here, the seller delegated its duty to manufacture the toasters to a competitor. This voided the contract, and the buyer is no longer bound. (B) is incorrect. The seller delegated its duty to manufacture the toasters to a competitor. It did not assign its right to payment, which would have been permissible.

A construction contractor brought a breach of contract claim in federal court against a homeowner who had hired the contractor to build an apartment over an existing garage. The action turned on the scope of the work covered by the contract. The contractor and the homeowner were the only witnesses at the bench trial, and they strongly disagreed about the scope of the work. At the end of the trial, the judge stated findings of fact on the record but never issued a written opinion. Neither party objected to the findings. The judge found in favor of the homeowner, and the contractor appealed. Is the appellate court likely to overturn the findings? A) No, because the appellate court must give due regard to the trial judge's opportunity to determine witness credibility. B) No, because the contractor failed to object to the findings when the judge stated them in open court. C) Yes, because a judge must set forth findings of fact in a written opinion or memorandum of decision. D) Yes, because there were disputed issues of fact at trial.

A (A) is correct. In a bench trial, the judge acts as the fact-finder, and the appellate court will give deference to the judge's findings of fact. There are no facts in the question to raise any question about the judge's findings, and he must have found the homeowner to be the more credible witness. (B) is incorrect. An objection to the judge's findings of fact is not required. (C) is incorrect, as it is an incorrect statement of the law. Under Federal Rule of Civil Procedure 52, the judge's findings of fact (and conclusions of law) may be stated on the record after the close of the evidence or may appear in an opinion or a memorandum of decision filed by the court.

An individual investor purchased stock through a company's stock offering. When the price of the stock plummeted, the investor sued the company in a state court in State A, claiming that the company's offering materials had fraudulently induced him to purchase the stock and seeking $25,000 in damages. A university that had purchased the company's stock through the same offering sued the company in federal court in State B, claiming that the offering materials violated federal securities laws and seeking $1 million in damages. The individual investor's suit proceeded to trial. The state court ruled that the company's offering materials contained false information and awarded the investor a $25,000 judgment. The university immediately moved for partial summary judgment in its federal action against the company, arguing that the state court judgment bound the federal court on the issue of whether the company's offering materials contained false information. Neither State A nor State B permits nonmutual issue preclusion. Should the court grant the university's motion? A) No, because State A does not permit nonmutual issue preclusion. B) No, because the federal court sits in a state that does not permit nonmutual issue preclusion C) Yes, because federal law permits nonmutual issue preclusion. D) Yes, because the issue of whether the materials contained false information was actually litigated and necessarily decided.

A A) is correct. Generally speaking, when dealing with the preclusive effect of a judgment, the recognizing court should not give a judgment any greater effect than the rendering state would. (When "case one" has been decided in state court, the court in case two generally will apply the claim or issue preclusion of the jurisdiction that decided case one.) Here, the rendering state, State A, does not recognize nonmutual collateral estoppel, so a federal court sitting in State B should not give the judgment issue preclusive effect.

Under the authority of a federal voting rights statute, some states drew congressional districts in a manner calculated to increase the likelihood that members of historically disadvantaged minority racial groups would be elected. The U.S. Supreme Court declared these districts to be unconstitutional, as improper racial gerrymanders. In response to this ruling, Congress passed a new statute that explicitly denies the Supreme Court appellate jurisdiction over all future cases challenging the constitutionality of action taken under the authority of the federal voting rights statute. Which of the following is the most persuasive argument for the constitutionality of the new statute restricting the Supreme Court's appellate jurisdiction? A) Article III of the Constitution explicitly states that the Supreme Court's appellate jurisdiction is subject to such exceptions and regulations as Congress shall make. B) The constitutional principle of separation of powers authorizes Congress to pass statutes calculated to reduce the effects of Supreme Court decisions that interfere with the exercise of powers that have been delegated to the legislative branch. C) The establishment and apportionment of congressional districts directly affect interstate commerce, and the Constitution authorizes Congress to use its plenary authority over such commerce for any purpose it believes will promote the general welfare. D) The Fifteenth Amendment authorizes Congress to enforce the amendment's voting rights provisions by appropriate legislation, and Congress could reasonably determine that this restriction on the Supreme Court's appellate jurisdiction is an appropriate means to that end.

A Article III explicitly gives Congress the power to make exceptions to the Supreme Court's appellate jurisdiction

As an aide to a member of the Congress of the United States, you are expected to provide an analysis of the constitutionality of proposed legislation that your employer is called to vote on. A bill has been proposed that would create a mandatory price schedule for every motor vehicle sold in the United States. Which of the following should you tell your employer is the strongest constitutional basis for the proposed legislation? A) All motor vehicle transactions in the United States, taken as a whole, have a significant impact upon interstate commerce. B) Because the purchase or sale of a motor vehicle, by definition, involves commerce, the federal government may regulate such transactions under the commerce power. C) Congress has the power to regulate transportation in the United States. D) Congress has the power to legislate for the general welfare of the people of the United States.

A Congress may regulate any activity, local or interstate, which either in itself or in combination with other activities has a substantial economic effect upon, or effect on movement in, interstate commerce. Because all motor vehicle transactions in the United States, in the aggregate, have a significant impact upon interstate commerce (as (A) states), Congress is constitutionally empowered to regulate such transactions by, e.g., enacting a mandatory price schedule.

A restaurant owner properly sued a food supplier in federal district court for breach of contract and timely demanded a jury trial. The complaint asserted both legal and equitable claims. Which of the following statements correctly states the proper order for trying both claims? A) All legal claims should be tried first by the jury. B) All equitable claims should be tried first by the court. C) Legal and equitable claims may not be tried together, so the order does not matter since there will be separate trials. D) It is up to the federal district court judge's discretion which claim will be tried first

A If legal and equitable claims are joined in one action involving common fact issues, the legal claim should be tried first to the jury and then the equitable claim to the court (the jury's finding on fact issues will bind the court in the equitable claim).

A state statute provided for criminal penalties for "knowingly selling alcoholic beverages in violation of the regulations of the State Liquor Commission to any person under the age of 18." One of the State Liquor Commission regulations provided that "before an alcoholic beverage is sold to any person between the ages of 17 and 24, the seller must demand some form of photo identification to determine the buyer's age." A minor who looked much older than his age of 17 walked into a tavern located in the state and asked the bartender for a beer. The bartender never asked the minor for any form of identification, as he thought that he was at least 25 years old. Had the bartender asked for identification, the minor would have shown him a fake identification card showing that he was 21 years old. The bartender served the beer to the minor, who consumed it on the premises. The bartender was subsequently charged under the state statute for selling the beer to the minor. Is the bartender guilty? A) No, because he reasonably believed that the minor was older than 25 years. B) No, because the minor had fake identification with which he could have obtained the beer. C) Yes, because he sold an alcoholic beverage to a minor, a strict liability crime. D) Yes, because he failed to ask for identification, and the regulation does not provide for a mens rea requirement.

A Ignorance or mistake as to a matter of fact will affect criminal guilt only if it shows that the defendant did not have the state of mind required for the crime. In addition, the mistake must be reasonable unless the offense is a specific intent crime. Here, the statute requires that the defendant have acted "knowingly" with respect to each of the material elements of the offense. A person acts knowingly with respect to the nature of his conduct when he is aware that his conduct is of that nature or that certain circumstances exist. At least one of the material elements of the offense here is that the sale be to a person under the age of 18. If the bartender believed that the minor was 25 years old, the bartender has not acted knowingly with respect to the fact that the purchaser was under 18, and he cannot be convicted of violating the statute. Although the regulation apparently does not have a state of mind requirement, it is not entirely clear that criminal liability can result from the violation of the regulation. There would have to be a separate statute providing for criminal penalties for failing to check for identification. Furthermore, the criminal statute in question clearly has a state of mind requirement that must be satisfied, as explained above.

A defendant is charged with trafficking in firearms, in violation of federal firearms control laws, as well as receiving stolen property. The charges arise from the defendant's having attempted to sell a semi-automatic weapon identified as one of dozens that were stolen from a warehouse a year ago. The defendant denies intending to sell the gun or knowing that it had been stolen. At trial, assuming all notice requirements have been met, which of the following would the court be LEAST likely to allow the prosecution to introduce as evidence against the defendant? A) Evidence that the defendant was once convicted of armed robbery with a semi-automatic weapon. B) The testimony of a witness that, the day before the defendant's arrest, he asked the witness how much she would be willing to pay for a semi-automatic weapon. C) The testimony of a member of a secret paramilitary group that the defendant had been supplying the group with weapons for several months. D) Evidence that the defendant had been previously convicted of receipt of stolen weapons.

A In a criminal case, evidence of the defendant's other crimes or misconduct is inadmissible if offered solely to establish criminal disposition. A broad exception to the general rule permits evidence of other crimes or misconduct to be admitted if such acts are relevant to some issue other than the character of the defendant to commit the crime charged. Such evidence may be used to show motive, opportunity, intent, preparation, plan, knowledge, identity, or absence of mistake. because evidence of the defendant's previous conviction for armed robbery does not come within any permissible use of evidence of other crimes or bad acts. Because the defendant apparently is not contesting the issue of whether he possessed the semi-automatic weapon, it is irrelevant that the robbery conviction shows possession of such a weapon at some earlier time. The only use to which evidence of this conviction can be put is to show the defendant's bad character and disposition to commit the crimes with which he is presently charged.

An employee of the United States Department of Labor was instructed by his superior to solicit subscriptions to the Department's bulletin on a door-to-door basis in the city in which he worked. While doing so, the employee was arrested for violation of a city ordinance that prohibited commercial solicitation of private residences. What is the employee's best defense? A) Intergovernmental immunity. B) The First Amendment freedom of expression as it applies to the states through the Fourteenth Amendment. C) The Equal Protection Clause as it applies to the states through the Fourteenth Amendment. D) The city ordinance effectively restricts interstate commerce.

A State and local governments cannot tax or regulate the activities of the federal government. This principle is often termed "intergovernmental immunity." The arrest and prosecution of a federal employee who was on the job violates this principle, which is based on the supremacy of the federal government and federal law. (B) is not a bad answer because door-to-door solicitation is protected by the First Amendment. However, at best, (B) would subject the city's actions to strict scrutiny and allow the city to prevail if it could prove that its action was necessary to achieve a compelling government purpose.

A victim and his former business partner, the defendant, had a bitter falling out after the victim accused the defendant of embezzling company funds. The defendant threatened to get even. Shortly thereafter, while driving on the expressway, a car swerved suddenly in front of the victim's car. Although the victim applied the brakes immediately, his car failed to stop. To avoid colliding with the car ahead of him, he swerved to the right and smashed into a concrete retaining wall. A passing motorist stopped and came to the aid of the victim. Bleeding profusely from a head wound, and rapidly losing consciousness, the victim said, "I don't think I'm going to make it. My former partner must have tampered with my brakes to get back at me." With that, the victim lapsed into unconsciousness, and has been in a coma and on life support ever since. A personal injury suit has been filed on his behalf by a court-appointed guardian against the defendant. At trial, can the motorist testify as to the statement made by the victim? A) No, because the victim merely suspected that the defendant tampered with the brakes. B) No, because the victim is still alive. C) Yes, because the victim thought he was about to die. D) Yes, because this is a civil case.

A Testimony as to the statement made by the victim is inadmissible as a statement under belief of impending death, because the victim did not actually have firsthand knowledge that the defendant was responsible for the collision. The statement is hearsay because it is a statement made by the declarant (the victim), other than while testifying, offered to prove the truth of the matter asserted therein. Here, the plaintiff wants to present this testimony to prove the truth of the statement that the defendant was responsible for the brake failure, and will argue that the statement falls under the hearsay exception for dying declarations. In a civil case or a homicide prosecution, a statement made by a now unavailable declarant while believing his death to be imminent, that concerns the cause or circumstances of what he believed to be his impending death, is admissible. [Fed. R. Evid. 804(b)(2)] For this exception to apply, the declarant need not actually die. Rather, the declarant must be "unavailable" when the statement is offered. A declarant is unavailable if he: (i) is exempted from testifying on the ground of privilege, (ii) refuses to testify despite a court order, (iii) testifies to lack of memory of the subject matter of the statement, (iv) cannot be present or testify because of death or physical or mental illness, or (v) is beyond the reach of the court's subpoena and the statement's proponent has been unable to procure his attendance or testimony by process or other reasonable means. Regarding the statement at issue here, the victim certainly thought he was about to die from his injuries. In addition, he is unavailable, as his physical condition prevents him from testifying. However, the victim's statement represents a mere suspicion that the defendant tampered with the brakes. As well-founded as such a suspicion may be (given the history between the victim and the defendant), a statement based on mere suspicion rather than actual knowledge does not constitute a statement concerning the cause or circumstances of an "impending death" for purposes of the dying declarations exception.

An accountant employed by a federal agency was offended by jokes and cartoons that employees would post in the office cafeteria. Although none of the material violated any agency policy, the accountant lodged a number of complaints with his supervisor that went unheeded. Finally, the accountant posted his own notice chastising the agency for allowing cartoons the employee found offensive. The notice prompted a great deal of disruption in the office, particularly after it was posted on another employee's blog and received some media attention. The accountant did not have an employment contract with the agency and was not covered by any policy entitling him to dismissal only for certain grounds. Which of the following statements is most accurate regarding the agency's right to dismiss the accountant? A) The accountant has a liberty interest in the exercise of his First Amendment rights that entitles him to a hearing to contest the grounds of his dismissal. B) The accountant has a property interest as a public employee that precludes him from being fired without notice and an opportunity to respond. C) The accountant has no right to a hearing because his statements were not an expression of views on public issues. D) The accountant has both a liberty interest and a property interest that entitles him to a pretermination evidentiary hearing.

A The accountant is entitled to a hearing because he has a liberty interest in the exercise of his First Amendment rights. If the accountant is fired, he has a right to a hearing to determine whether his First Amendment rights were violated by his dismissal. Under the Due Process Clause of the Fifth Amendment, a person has a liberty interest in the exercise of specific rights provided by the Constitution, including freedom of speech. A government employee may not be fired for expressing his views regarding public issues, but can be fired for speech that disrupts the employer's policies or undermines the employer's authority. Under the Court's expansive interpretation of what a public issue is in this context [see Rankin v. McPherson (1987)], the accountant's statement would probably qualify. At the very least, he can make enough of a showing that his termination violates his free speech rights to be entitled to a hearing on the issue under procedural due process principles. [See Givhan v. Western Line Consolidated School District (1979)] (

A defendant, a nurse at a nursing home, is charged with murdering a resident at the home by adding an allegedly lethal substance to the resident's food. At trial, to prove that the substance added to the resident's food could result in death, the prosecutor, without first calling any witnesses, offers to read into evidence several pages from a standard medical treatise that support the prosecution's claim that the substance the defendant added to the food is lethal. Is the evidence offered admissible? A) No, because the treatise excerpts were not offered during the examination of a qualified expert. B) No, because the treatise itself must be introduced as an exhibit. C) Yes, although hearsay, under the learned treatise exception to the hearsay rule. D) Yes, because the lethal nature of the substance is relevant to the defendant's state of mind and intent.

A The content of the treatise is hearsay because it is an out-of-court statement being offered for the truth of its contents. There is a hearsay exception that allows portions of learned treatises to be read into evidence, but this exception applies only when the treatise is being used on direct or cross-examination of an expert witness. Here the prosecutor is trying to read portions of the treatise before calling any expert witness to the stand, so the treatise is inadmissible hearsay.

A small cruise ship struck a whale swimming underwater, causing the ship to suddenly lurch sideways. A passenger on the ship who was walking down a corridor lost his balance and bumped his head on the edge of a doorway. Because of a previously existing medical condition that made him susceptible to bleeding on the brain, he suffered a cerebral hemorrhage and permanent mental impairment, despite prompt medical attention on the ship. The passenger brought suit against the cruise ship owner for his damages. At trial, the passenger presented evidence of how he was injured as he walked down the hallway, his previous medical condition, and his medical expenses and other damages. The cruise ship owner presented evidence that the cruise ship was following its approved route and that the whale could not have been detected before impact, and that the bump would not have injured someone in ordinary health. At the close of the evidence, the cruise ship owner moved for a directed verdict. How should the court rule? A) Grant the motion, because there is no evidence that the crew operated the ship negligently. B) Grant the motion, because the cruise ship owner introduced uncontroverted evidence that a person in normal health would not have been injured by the bump. C) Deny the motion, because the jury could find that the cruise ship owner, as a common carrier and innkeeper, breached its high duty of care to the passenger. D) Deny the motion, because the fact that the severity of the passenger's injuries was not foreseeable does not cut off the cruise ship owner's liability.

A The court should grant the cruise ship owner's motion because the passenger has not established a prima facie case of negligence against the cruise ship. To establish a prima facie case for negligence, a plaintiff must show (i) a duty of care, (ii) breach of that duty, (iii) actual and proximate cause, and (iv) damages. As a common carrier and/or an innkeeper, the cruise ship owed its passengers a high duty of care, and therefore would be liable for slight negligence. However, the passenger has offered no evidence to establish that the cruise ship employees breached that duty, and res ipsa loquitur is not applicable here because the collision with the whale swimming underwater is not the type of event that would occur only as a result of negligence. Because the passenger failed to establish breach of duty, the court should grant the cruise ship owner a directed verdict.

An entrepreneur entered into a written lease-option to purchase an office building. The option to purchase the building could be exercised at any time during the five-year term of the lease by giving the building's owner a written 30-day notice of the intent to exercise the option. A few months later, the entrepreneur assigned his lease-option to a dentist looking for office space by written agreement. If the dentist fully performs under the lease, can the dentist exercise the option to purchase that was given to the entrepreneur? A) Yes, because both the burden and benefit of the covenant to convey run with the land. B) No, because the covenant to convey does not touch or concern the land. C) No, because the option to purchase was personal to the entrepreneur. D) No, because the burden of the covenant to convey given in a lease does not run with the land.

A The dentist can exercise the option to purchase. A covenant to convey touches and concerns both the leasehold and reversion, and therefore runs with those respective interests in the land.

A landowner conveyed his land "to my son for life, then to my son's widow for her life, then to my son's children." At the time of the conveyance, the son was 20 years old and unmarried. The son eventually married and had two children, the landowner's grandson and granddaughter. Many years later, the landowner and the grandson were involved in a train accident. The landowner was killed instantly. The grandson died a short time later of his injuries. The landowner left his entire estate by will to his friend. The grandson's will devised his entire estate to the city zoo. The son's wife was so grief-stricken that she became ill and died the next year, leaving her entire estate to her husband. Eventually the son met and married a 21-year-old. Ten years later, the son died, leaving everything to his second wife. When the second wife moved onto the land, the granddaughter filed suit to quiet title to the land, joining all of the appropriate parties. If the jurisdiction recognizes the common law Rule Against Perpetuities, unmodified by statute, in whom will the court most likely find that title to the land is held? A) One-half in the granddaughter and one-half in the city zoo, subject to the second wife's life estate. B) One-half in the granddaughter and one-half in the second wife, because the second wife took the son's interest. C) Entirely in the friend, subject to the second wife's life estate, because the gift to the son's children violates the Rule Against Perpetuities. D) Entirely in the granddaughter, subject to the second wife's life estate, because the grandson did not survive the son.

A The granddaughter and the city zoo each own one-half of the land, subject to the second wife's life estate. At the time of the conveyance by the landowner, the son had a life estate, the son's widow had a contingent interest (because the son's "widow" cannot be ascertained until the son's death), and the son's children had a contingent remainder (because they have not yet been born). When the grandson and the granddaughter were born, however, their interests became vested subject to open (i.e., if the son had more children). Thus, when the grandson died, he had a vested remainder subject to open that he was free to devise by will; the city zoo took his vested remainder subject to open. At the son's death, the class of his "children" closed (because the son could not have any more children), and the granddaughter's and the zoo's vested remainders subject to open became indefeasibly vested. Also at the son's death, his widow was ascertained and her interest vested in possession. Because the second wife was the son's widow, she is entitled to the valid life estate. Thus, the granddaughter and the city zoo hold one-half interests, subject to the second wife's life estate. (B) is wrong because the son had no interest in the land when he died. He merely had a life estate, which ended at his death. He did not inherit any interest in the property from anyone else. The only person he inherited from in these facts was his first wife, and she had no interest in the land. Furthermore, this choice overlooks the city zoo's interest, which was inherited from the grandson. (C) is wrong because the son's children's interest does not violate the Rule Against Perpetuities. To be valid under the Rule, an interest must vest if at all within a life in being at its creation plus 21 years. The son is a life in being. At the son's death, his children's interest is certain to vest or fail: If the son had any children, at his death, the children's interest would become indefeasibly vested (i.e., the class would close and the children's interest would no longer be subject to open). Note that the children need not come into possession within the perpetuities period; the only requirement is that their interests vest within the period. Likewise, if the son had no children, the gift to them was certain to fail at his death. Thus, the children's interest does not violate the Rule. Because the son had children and their interest was valid, there was no interest to revert to the landowner and to be devised to the friend. Note that the unborn widow aspect of this question is a red herring. The fact would be relevant only if the children's gift were conditioned on their surviving the widow, in which case the takers would remain unascertained and their interest would remain contingent until that time. But because the children's interest vested at the son's death, it is irrelevant that the son's "widow" was not a life in being at the creation of the interest.

A buyer for a chain of shoe stores ordered 1,000 pairs of shoes from a shoe manufacturer. The shoes cost $50 per pair, so the total contract price was $50,000. It happened that the manufacturer owed $50,000 to a trucking company. The manufacturer assigned, in writing, "all proceeds from the contract with the buyer" to the trucking company. The manufacturer notified the buyer that he had assigned the proceeds of the contract to the trucking company and then shipped the 1,000 pairs of shoes to the buyer. Upon receipt of the shoes, the buyer discovered that 10% of the shoes were defective. He sent a check for 90% of the contract price ($45,000) to the manufacturer, who deposited the check. Shortly thereafter, the manufacturer closed down its business and disappeared without a trace. The trucking company, meanwhile, demanded payment from the buyer, to no avail. If the trucking company sues the buyer for the $45,000 that the buyer paid on the contract, will the trucking company prevail? A) Yes, because the buyer had notice from the manufacturer that the contract had been assigned to the trucking company.] B) No, because the manufacturer wrongfully took the money that was assigned to the trucking company and is solely liable to the trucking company. C) No, because the buyer fulfilled his obligations under the contract by paying the manufacturer. D) No, because the trucking company could not have performed the other side of the contract by furnishing the shoes.

A The trucking company will be able to recover the $45,000 from the buyer because the buyer had notice of the assignment. Most contract rights may be assigned, and the right assigned here (to receive money) falls within the general rule. Once the assignment is effective, the assignee (the trucking company) becomes the real party in interest, and he alone is entitled to performance under the contract. (The assignor has been replaced by the assignee.) Once the obligor (the buyer) has knowledge of the assignment, he is bound to render performance to the assignee. Here, the assignment was effective as soon as the assignor (the manufacturer) manifested his intent that the right should be assigned (i.e., in his written assignment to the trucking company). The buyer was given notice of the assignment and, thus, was bound to pay the trucking company. The buyer breached his duty by paying the manufacturer instead of the trucking company.

A man conveyed the eastern half of a tract of vacant land to a woman by a warranty deed. The woman promptly recorded the deed. The land conveyed to the woman fronted on a public highway. The land retained by the man was landlocked. One year later, the man died intestate, leaving a cousin as his only heir. The cousin visited the man's land for the first time and discovered that it had no access to a public highway. A neighbor who owned adjoining land fronting on the public highway offered to sell the cousin a right to cross the neighbor's land for access to the highway. Although the neighbor's price was reasonable, the cousin rejected the offer. The woman has refused to allow the cousin to cross her land for access to the public highway even though the woman's land is still vacant. The cousin has sued the woman, seeking access across the woman's land to the public highway. Who is likely to prevail? A) The cousin, based on necessity. B) The cousin, because the woman's land is still vacant. C) The woman, because the cousin could obtain an alternative access to the highway from the neighbor. D) The woman, because the man failed to reserve an easement in his deed to the woman.

A When the owner of a tract of land sells a part of it and by this division deprives one lot of access to a public road, a right-of-way by absolute necessity is created by implied grant or reservation over the lot with access to the public road. The owner of the servient parcel has the right to locate the easement, provided the location is reasonably convenient. Here the man owned the original tract and sold a part of it to the woman. When he sold the part, his remaining tract was landlocked. Therefore, a right-of-way by absolute necessity was created to give him access to the public road. The woman can decide where to locate the right-of-way, but she cannot deny it altogether. This easement by necessity is an easement appurtenant, meaning that the right of special use benefits the holder of the easement in his physical use or enjoyment of another tract of land. The benefit of an easement appurtenant becomes an incident of the possession of the dominant tenement. All who possess or subsequently succeed to title to the dominant tenement become, by virtue of the fact of possession, entitled to the benefit of the easement. Therefore, the cousin who inherited the land is entitled to use the right-of-way.

A defendant is on trial for violating a statute forbidding possession of a concealed weapon within 100 yards of a government building. The prosecution presents evidence that the defendant was arrested on a street corner with a handgun in his pocket. The building housing the local city hall occupies the entire block on the north and east sides of the two streets where the defendant was apprehended. Which of the following statements is most accurate regarding judicial notice of the location of the city hall? A) The judge may take judicial notice of this fact without resort to a map, and should instruct the jury that it may, but need not, accept this fact as evidence of an element of the offense.The judge may take judicial notice of this fact without resort to a map, and should instruct the jury that it may, but need not, accept this fact as evidence of an element of the offense. B) The judge may take judicial notice of this fact only upon reference to an official street map of the city.The judge may take judicial notice of this fact only upon reference to an official street map of the city. C) The judge may not take judicial notice of this type of fact in a criminal case without a request by the prosecution.The judge may not take judicial notice of this type of fact in a criminal case without a request by the prosecution. Incorrect D) If the judge properly takes judicial notice of this fact, a presumption is created that shifts the burden of persuasion to the defendant to disprove this fact.If the judge properly takes judicial notice of this fact, a presumption is created that shifts the burden of persuasion to the defendant to disprove this fact.

A Because The judge may take judicial notice of this fact because it is a matter of common knowledge in the community, but the jury is not required to accept the fact as conclusive in a criminal case. Judicial notice may be taken of facts that are not subject to reasonable dispute because they are generally known within the territorial jurisdiction of the trial court. [Fed. R. Evid. 201(b)] The facts need not be known everywhere as long as they are known in the community where the court is sitting. The location of the city hall is such a fact. As choice (A) also states, in a criminal case the jury should be instructed that it may, but is not required to, accept as conclusive any fact that is judicially noticed. [Fed. R. Evid. 201(f)]

A plaintiff sued the insurer of her home after the insurer denied coverage for water damage to the home allegedly caused by a frozen plastic pipe that burst. At trial, the insurer called as an expert witness an engineer, who testified that the pipe had burst because of age rather than freezing. On cross-examination, the engineer admitted that, five years earlier, he had been convicted of tax fraud, even though he had asserted that it was his accountant's error. In response, the insurer calls a witness, who is well acquainted with the engineer and his reputation, to testify that (1) in the witness's opinion, the engineer is a truthful person, and (2) the engineer's neighbors all describe him as a truthful person. How much, if any, of the witness's testimony is admissible? A) All of the testimony is admissible to support the engineer's credibility B) Only the portion concerning the engineer's reputation is admissible, because where both opinion and reputation evidence are available, only the latter is admissible under a rule of preference. C) Only the portion concerning the witness's opinion of the engineer's character, because the witness's reporting of the neighbors' comments is hearsay. D) None of the testimony is admissible, because it is collateral, having no bearing on the engineer's qualifications as an expert.

A Once a witness is impeached with evidence of his bad character for truthfulness, he may be rehabilitated with reputation and opinion evidence of his good character for truthfulness. Here the plaintiff has attacked the engineer's character for truthfulness by presenting evidence of his prior tax fraud conviction, so the insurer may now present evidence to rehabilitate the engineer. The witness's opinion and reputation testimony as to the engineer's good character for truthfulness is proper.

An interior decorator asked a woodworker she met at a crafts fair to build a curly maple armoire. They entered into a written contract, with a contract price of $6,500 to be paid upon the decorator's receipt of the armoire. When the work was completed, the woodworker shipped the armoire to the decorator. After inspecting it, the decorator felt that it was not of the same high level of workmanship as she was expecting, given the other furniture that the woodworker had showcased at the fair, and a good faith dispute arose between the parties as to the workmanship. The decorator sent the woodworker a check for $4,000 marked "payment in full." The woodworker indorsed and cashed the check, then sued the decorator to recover the $2,500 balance. What would most courts likely hold? A) The woodworker's cashing of the check constituted an accord and satisfaction, discharging the decorator's duty to pay the balance. B) The woodworker can recover the $2,500 balance from the decorator. C) The woodworker is estopped to sue for the balance because he cashed the check knowing that it was being tendered in full settlement. D) The woodworker's indorsing a check so marked constituted a written release, thereby discharging the contract.

A Most courts would hold that there is a good faith dispute, and the check thus proposed an accord; the woodworker's act of cashing it is a satisfaction. A contract may be discharged by an accord and satisfaction. An accord is an agreement in which one party to an existing contract agrees to accept, in lieu of the performance that he is supposed to receive from the other party, some other, different performance. Satisfaction is the performance of the accord agreement. An accord and satisfaction generally may be accomplished by tender and acceptance of a check marked "payment in full" where there is a bona fide dispute as to the amount owed. Here, there is a good faith dispute between the parties as to the workmanship on the armoire. Therefore, the decorator's tender of the check marked "payment in full" and the woodworker's cashing of the check constituted an accord and satisfaction, discharging her duty to pay the balance

A developer and an investor had been in the real estate business for many years. Because of their long-standing relationship, the developer and the investor, neither of whom was an attorney, often dispensed with certain legal formalities when dealing with each other, thus saving the costs of lawyers' fees and other attendant expenses. The investor owned a parcel of land that the developer was interested in. At lunch one day, the developer offered to buy the parcel from the investor for $50,000. The investor accepted the developer's offer, and the parties agreed on June 15 as the closing date. The developer wrote out and handed the investor a check for $2,500 with "earnest money" written in the memo, and they shook hands on their deal. A few weeks before closing, the developer called the investor and told him she had changed her mind about purchasing the land because of a sudden economic downturn in the area. The investor appeared at the developer's office on June 15 with the deed to the land in his hand. The developer refused to tender the balance due, and the investor sued the developer for specific performance. Will the investor prevail? A) No, because the agreement does not comply with the Statute of Frauds and is, therefore, unenforceable. B) No, but the court will allow the investor to keep the $2,500 earnest money as damages. C) Yes, because the $2,500 payment constituted part performance of the contract. D) Yes, because the developer and the investor had established a course of dealing.

A The investor will not succeed in a suit for specific performance because the agreement is unenforceable under the Statute of Frauds. Under the Statute of Frauds, a land sale contract is unenforceable unless it is in writing and signed by the party to be charged. The Statute of Frauds requires the writing to contain all essential terms of the contract, which are: (i) a description of the property, (ii) identification of the parties to the contract, (iii) the price, and (iv) the signature of the party to be charged. Here, the agreement between the investor and the developer concerns the sale of land; thus, the agreement must be evidenced by a writing to comply with the Statute of Frauds. The only writing mentioned in the facts is the check given to the investor by the developer. This check contains neither a description of the property that is the subject of the agreement nor the price. Thus, the check is not a writing sufficient to satisfy the Statute of Frauds. Consequently, the agreement is unenforceable, and the investor will not prevail. C) is incorrect. The doctrine of part performance generally requires two of the following: possession, improvements, and full or partial payment of the purchase price. A few states will grant specific performance of a contract despite the absence of a writing if there has been payment of the purchase price. Even under this view, however, the developer's payment of $2,500 out of a total price of $50,000 will not constitute sufficient performance to remove this agreement from the purview of the Statute of Frauds.

A man of seemingly modest means died, leaving his nephew as his sole heir. Among the items inherited by the nephew were some old oil paintings. The nephew knew nothing about art and had no place to put the paintings in his home. He placed an ad in the paper offering to sell the paintings at a price to be mutually agreed upon. A buyer for an art gallery responded to the ad. The buyer did not identify himself as an art gallery buyer or tell the nephew that he was knowledgeable about art. Rather, he concocted a story about wanting the paintings for his country estate. The nephew, for his part, revealed his lack of knowledge about art when he told the buyer that his uncle had probably painted the pieces himself. From the signature and the style, the buyer recognized that the artist was a renowned 19th century American portrait artist. The nephew and the buyer agreed upon a price and executed a contract. However, before the nephew delivered the paintings to the buyer, or the buyer paid him, he sought to rescind the contract. The buyer insisted that the nephew deliver the paintings to him and threatened to sue for breach of contract if he did not. Which argument would give the nephew the best basis for rescinding the contract with the buyer? A) The nephew told the buyer that his uncle had probably painted the paintings himself. B) The nephew did not know that the buyer was a professional buyer for an art gallery and was knowledgeable about art. C) The buyer falsely told the nephew that the paintings were going to be used to furnish his (the buyer's) country estate. D) The contract was still executory on both sides.

A) The nephew may be able to rescind the contract on the grounds of unilateral mistake if the buyer was aware that the nephew was mistaken about the identity of the artist. Where only one of the parties is mistaken about facts relating to the agreement, the mistake usually will not prevent formation of the contract. However, if the nonmistaken party is aware of the mistake made by the other party, he will not be permitted to snap up the offer; i.e., the mistaken party will have the right to rescind the agreement.

An automotive engineer announced that he had developed a carburetor that will enable cars to achieve 100 miles per gallon of fuel, and that he will allow the carburetor to be inspected next month. Soon after, a former employer of the engineer brought an action to prohibit the engineer from displaying the carburetor, claiming that the engineer probably had stolen the carburetor's design from the employer. The court granted the employer a temporary restraining order prohibiting the engineer from disclosing any mechanical details of his carburetor, and ordered a hearing to be held in one week to determine whether a preliminary injunction should be issued. Because each party would have to reveal the mechanical details of his designs at the hearing, the employer requested that the hearing be closed to the public and that the record be sealed to avoid revelation of his designs. The court granted the request. A reporter for a monthly automobile magazine heard about the case and wanted to attend the hearing. When he was told that the hearing would be closed, he filed an action to have it opened. What is the reporter's best argument for opening the hearing? A) Closure is not necessary to preserve an overriding interest here. B) The right of freedom of the press is extensive and allows the press to attend all hearings of interest to the public. C) Closure here amounts to a prior restraint. D) Under the fairness doctrine, the magazine will be required to give each litigant an opportunity to present his side of the case.

A) Closure is not necessary to preserve an overriding interest here. because trials and pretrial hearings generally must be open to the public. The Supreme Court has held, at least in the context of criminal cases, that trials and pretrial proceedings can be closed only if closure is necessary to preserve an overriding interest and the closure order is narrowly tailored to serve the overriding interest. While the Court has not yet established the standard for civil matters such as the case here, several Justices and commentators have suggested that the same standard will be applied in civil cases since they too have historically been open to the public.

A sports apparel distributor agreed with a clothing manufacturer to purchase from it "all the branded athletic shirts that our retailers request" during the next 12 months. One month after entering this agreement, investigative reports indicated that the manufacturer was using forced labor in some of its factories. Although nothing in the parties' agreement addressed the situation, the distributor informed the manufacturer that it was cancelling the agreement because of its unacceptable labor practices. At that point, the distributor had received orders for several hundred athletic shirts from its retailers. May the manufacturer enforce the agreement with the distributor? A) Yes, because the distributor's promise to purchase from the manufacturer all the shirts requested by its retailers makes the agreement enforceable. B) Yes, because the distributor's retailers submitted orders for the shirts prior to the distributor's attempted cancellation. C) No, because the absence of a minimum quantity term rendered the distributor's promise illusory. D) No, because the manufacturer's labor practices made the agreement unconscionable.

AYes because the distributor's promise to purchase from the manufacturer all the shirts requested by its retailers makes the agreement enforceable The manufacturer may enforce the agreement with the distributor because it is an enforceable requirements contract. In a requirements contract, a buyer promises to buy from a certain seller all of the goods the buyer requires, and the seller agrees to sell that amount to the buyer. Although no specific quantity is mentioned in offers to make these contracts, the offers are sufficiently definite because the quantity is capable of being made certain by reference to objective, extrinsic facts. Consideration also is present, as the promisor is suffering a legal detriment; it has parted with the legal right to buy the goods from another source. Here, the distributor has promised to use the clothing manufacturer to satisfy all the orders for branded athletic shirts for 12 months, forgoing its use of any other manufacturer. Hence, the agreement is enforceable.

While working on a construction project, a plaintiff was injured when a heavy object struck his knee. Although the plaintiff was fully compensated for his injuries at the time of the incident, he now seeks disability payments from the construction company because he has developed arthritis in the same knee. The construction company claims that the arthritis has nothing to do with the plaintiff's on-the-job injury and refuses to pay him disability money. The plaintiff sues. A doctor takes the stand to testify for the plaintiff. He is qualified as an expert witness and during direct examination states that in his opinion the blow to the plaintiff's knee caused his arthritis. On cross-examination, the construction company's attorney produces a treatise on arthritis and asks the doctor if the treatise is considered to be authoritative. The doctor responds that the treatise is a standard authority in the field, but that he did not rely on it in forming his professional opinion regarding the plaintiff's condition. The attorney then seeks to introduce into evidence a statement in the treatise that "the idea that arthritis can be caused by a single traumatic event is purely folklore, although it is widely believed by the ignorant who have no scientific basis for their beliefs." The plaintiff's attorney objects. How should the court rule on the admissibility of the statement from the treatise?

Admissible, both as substantive and impeachment The statement from the treatise is admissible to impeach and as substantive evidence. Under the Federal Rules, learned treatises can be used either for impeachment or as substantive evidence. One way the credibility of an expert witness may be attacked is by cross-examining him as to his general knowledge of the field in which he is claiming to be an expert. This can be done by cross-examining the expert on statements contained in any scientific publication that is established as reliable authority. Reliability of a publication may be established by: (i) the direct testimony or cross-examination admission of the expert, (ii) the testimony of another expert, or (iii) judicial notice. The Federal Rules recognize an exception to the hearsay rule for learned treatises and admit them as substantive evidence if: (i) the expert is on the stand and it is called to his attention, and (ii) it is established as reliable authority (see above). The doctor has admitted on cross-examination that the treatise is authoritative in the field. Thus, the attorney may use the statement in the treatise to attack the doctor's general knowledge of the field of arthritis by showing that the doctor's opinion that the blow to the plaintiff's knee caused his arthritis is considered to be ignorant and unfounded in the text of the treatise. As noted above, such an attack on the doctor's general knowledge of the field is a proper means of impeaching his credibility. In addition, pursuant to the Federal Rules, the statement may be read into the record as substantive evidence (i.e., as a means of proving that the plaintiff's arthritis could not have been caused by a single traumatic event, such as the blow to his knee). The statement may be used as substantive evidence because it has been brought to the attention of the doctor during cross-examination and he established it as a reliable authority, and it will be read into evidence while he is on the stand.

A passenger in a vehicle that was struck by another car sued the other car's driver, claiming that the collision severely injured his right leg. The defendant claimed that the plaintiff's leg injury resulted from an earlier, unrelated industrial accident. At trial, after having testified to his pain from the injury allegedly caused by the defendant, the plaintiff called as a witness the physician who treated him. The physician offers to testify that the plaintiff told him that his earlier leg problems had completely cleared up before the alleged injury caused by the defendant. If the defendant objects to the admission of this testimony, how should the court proceed? Admit it as a statement of a party or admit it as a statement for purpose of diagnosis and treatment

Admit it, as a statement for purpose of diagnosis and treatment a statement of a party-opponent is considered nonhearsay; i.e., a party's statement may be introduced by an opponent, and the party cannot keep the statement out of evidence, but the party cannot introduce his own out-of-court statement into evidence unless it falls within a hearsay exception.

A horse breeder offered to sell a colt to his neighbor and they agreed on a purchase price. The horse breeder subsequently received a letter from the neighbor thanking him for the sale and summarizing their agreement. The letter contained the neighbor's alleged signature. When the horse breeder attempted to set up transfer of the colt, the neighbor denied that she agreed to purchase it. In a breach of contract action against the neighbor, the horse breeder offers into evidence the letter. The horse breeder testifies that he is familiar with the neighbor's handwriting and recognizes the signature on the letter as being hers. Assuming appropriate objection by the neighbor, who claims that she did not sign the letter, how should the trial court rule on the admissibility of the letter?

Admit the letter but instruct the jury that it is up to them to decide whether the letter is authentic All that is necessary is proof sufficient to support a jury finding of genuineness. The authenticity of a document is a preliminary fact to be decided by the jury. Here, the horse breeder's testimony that he is familiar with the neighbor's handwriting and that he recognizes the signature on the letter to be that of the neighbor is sufficient to support a jury finding of genuineness. Thus, the letter should be admitted and authenticity should be left to the jury to decide.

junkyard kept a vicious watchdog on the premises to deter thieves. One night an unknown person cut the lock on the junkyard's gate and the dog escaped. The dog, snarling and barking, chased after a bicyclist passing by, causing the bicyclist to swerve into the path of a car, which struck and injured the bicyclist. If the bicyclist does not prevail in a suit against the junkyard owner, what is the likely reason? A) The bicyclist was not an invitee or licensee as to the junkyard. B) An unknown person allowed the dog to escape. C) The injury to the bicyclist was not caused by the dangerous nature of the dog. D) The junkyard owner's use of the dog to protect its property was permissible.

B A defendant in a strict liability action is liable only to "foreseeable plaintiffs"-persons to whom a reasonable person would have foreseen a risk of harm under the circumstances. Here, the bicyclist was just passing by the junkyard, and nothing in the facts suggests that the junkyard owner knew or should have known that an unknown person might let the dog escape.

A defendant is on trial for stealing jewelry from his co-worker. The defendant claims that the co-worker sold the jewelry to him because she needed money to buy medicine for her sick mother. The defense witness is asked to testify as to the co-worker's reputation in the community. The witness testifies that the co-worker is known as a dishonest person who makes her living as a "con artist." Assuming appropriate objections by defense counsel, which of the following questions would NOT be proper on cross-examination of the witness by the prosecutor? A) "Isn't it true that you're maligning the defendant's co-worker because she and your wife have been enemies since childhood? B) "Isn't it true that you were charged last year with assault for striking your wife?" C) "Have you heard that the defendant's co-worker teaches Sunday School classes on morality and has received an award from her church based on her outstanding moral character?" D) "Do you know that the defendant's co-worker teaches Sunday School classes on morality and has received an award from her church based on her outstanding moral character?"

B A witness may be interrogated upon cross-examination with respect to an act of misconduct only if it is probative of truthfulness. An assault is not probative of truthfulness, so it would not be proper impeachment evidence. Had the witness been convicted of the assault, the conviction would have been admissible, provided it was a felony.

A driver was driving his car negligently along a mountain road. He lost control of his car and careened over the side of a cliff. A jogger saw the driver's car go off the cliff and stopped to see if he could help. The jogger started to climb down the cliff to render aid to the driver. In doing so, the jogger slipped and broke his leg. The jogger sued the driver to recover damages for his broken leg. Regarding any defenses the driver might raise, which of the following statements is correct? A) A rescuer acts at his own peril. B) The excitement of the accident and the speedy response of the rescuer would be considered in a case such as this. C) Assumption of the risk cannot be invoked against rescuers. D) The driver would not have a valid defense.

B All of the circumstances will be considered when evaluating the conduct of the rescuer, including the excitement of the accident and the speedy response of the rescuer. A rescuer is a foreseeable plaintiff as long as the rescue is not reckless; hence, the defendant is liable if he negligently puts himself in peril and the plaintiff is injured attempting a rescue. A plaintiff may take extraordinary risks when attempting a rescue without being considered contributorily negligent. The emergency situation is one of the factors taken into account when evaluating the plaintiff's conduct.

A police officer stopped a driver who had run a red light. Upon approaching the car, the officer noticed a strong odor of alcohol and immediately asked whether the driver had been drinking. The driver admitted having had several alcoholic drinks that evening. The driver, charged with driving while intoxicated, moved to suppress the officer's testimony regarding the driver's statement about his drinking. The driver argued that the officer had elicited the statement without providing the requisite Miranda warnings. The prosecutor has responded that the statement should be allowed in the prosecution's case-in-chief or, at a minimum, should be allowed as impeachment in the event the driver testifies and denies drinking. How should the court rule regarding the driver's statement admitting his drinking? A) The statement should be allowed, because although the driver was in custody, the officer's spontaneous utterance upon smelling alcohol did not rise to the level of interrogation. B) The statement should be allowed, because the driver was not in custody for Miranda purposes when the admission was made. C) The statement should be suppressed both in the prosecution's case-in-chief and as impeachment evidence, even if the driver testifies. D) The statement should be suppressed in the prosecution's case-in-chief, but it may be used as impeachment evidence if the driver testifies.

B As a general rule, to offset the coercive nature of custodial interrogation by a police officer and protect defendants' Fifth Amendment right against compelled self-incrimination, the Supreme Court has made Miranda warnings a prerequisite to the admissibility of confessions obtained during a custodial police interrogation. The officer's question here was an interrogation, as the term "interrogation" extends to any words or conduct used by an officer intentionally to elicit an incriminating response. Nevertheless, the driver's response here is admissible despite the lack of Miranda warnings because the driver was not in custody. Determining whether a person is in custody for Miranda purposes involves a two-step inquiry. The Court will first look to whether a reasonable person in the circumstances would feel free to terminate the encounter and leave. If not, the Court looks to the whether the relevant environment presents coercive pressures similar to those of a station house interrogation. The more a setting resembles a traditional arrest, the more likely the Court will find the person to be in custody. Here, the driver, no doubt, felt that he was not free to leave, having just been pulled over for a traffic violation. However, a traffic stop does not resemble a traditional arrest. Given their brief nature and the motorist's knowledge that he will soon be on his way, the Supreme Court has found traffic stops to be noncustodial. Therefore, a police officer can ask questions during a routine traffic stop without giving Miranda warnings, and responses to those questions are admissible at trial in a case-in-chief and for impeachment purposes.

A man filed a federal diversity action against a bus company, seeking damages for injuries he had sustained in an accident while riding a bus owned by the company. The man demanded a jury trial. After the parties' attorneys examined the prospective jurors and exercised their challenges, six jurors and two alternate jurors were chosen. During the trial, two jurors became ill and were replaced by the alternate jurors. At the conclusion of the trial, a third juror also became ill, and the court excused that juror. The parties' attorneys stipulated to the return of a verdict from a five-person jury. The jury then deliberated and returned a verdict for the company. The man timely filed a motion for a new trial, arguing that the five-person jury was not large enough to return a verdict. Should the court grant the motion? A) No, because the court properly excused the three jurors due to illness. B) No, because the parties stipulated to a verdict from a jury of fewer than six jurors. C) Yes, because there must be at least six jurors on a federal civil jury. D) Yes, because there must be at least 12 jurors on a federal civil jury.

B If a jury drops below 6 a mistrial must result unless the parties agree to a lesser number

An elderly woman entered into a contract with a company in the business of providing home care services. Believing that she had been duped by representatives of the company, the woman commenced an action in federal court, properly based on diversity, seeking rescission of the contract. The company answered, denying the principal allegations of the woman's complaint and asserting a counterclaim against the woman for breach of contract. In addition, the company timely served a demand for a jury trial. The woman did not. Which statement best describes the roles of the judge and jury as finders of fact in the trial of the parties' claims? A) The judge will first determine the issues relating to the woman's claim for rescission, and if it concludes the rescission is not warranted, the jury will determine the issues relating to the company's breach-of-contract counterclaim. B)The jury will first determine the issues relating to the breach of contract claim, and the judge will determine the issues relating to the rescission claim that have not already been resolved by the jury. C) The jury will act as the sole finder of fact.The jury will act as the sole finder of fact. D) The judge may either act as the sole finder of fact on both the claim and the counterclaim, or allow an advisory jury to try the issues relating to the company's counterclaim.

B If legal and equitable claims are joined in one action involving common fact issues, the legal claim is tried first before the jury, and then the equitable claim is tried to the court. The jury's finding on fact issues will bind the court in the equitable claim.

A brother and a sister each hold an undivided one-half interest in a tract of land. By the terms of their agreement, each has the right to possess all portions of the property and neither has the right to exclusive possession of any part. The brother wrongfully ousts the sister from the property. What can the sister recover in an action against the brother? A) The fair rental value of the property for the time excluded. B) One-half of the fair rental value of the property for the time excluded. C) One-fourth of the fair rental value of the property for the time excluded. D) Nothing, because each co-tenant has the right to possess all portions of the property and neither has the right to exclusive possession of any part.

B If one co-tenant wrongfully ousts another co-tenant from possession of the whole or any part of the premises, the ousted co-tenant is entitled to receive her share of the fair rental value of the property for the time she was wrongfully deprived of possession. The sister was wrongfully ousted and therefore, as one of two co-tenants with the right to possess all portions of the property, she would be entitled to one-half of the fair rental value of the property during the period when she was ousted. (A) is incorrect because the sister is entitled only to her share (i.e., one-half) of the fair rental value.

While practicing their target shooting at the firing range, a man and woman got into an argument that almost erupted into physical combat, except that they were restrained and separated by bystanders. Later, in the parking lot of the range, the man shot the woman in the shoulder. Bystanders who rushed to the scene immediately after hearing the man's shot found the woman on the pavement with a black flashlight in her hand. The woman's pistol was in her locker at the firing range. At the trial of the woman's civil action for battery against the man, the woman established that the man intentionally shot her. In defense, the man testified that the woman approached him, saying, "We'll settle this once and for all, right now," and raised an object toward the man. He testified that he feared that the woman was about to shoot him with a pistol, so he fired in self-defense. Assuming that the jury decides that the man is telling the truth, what else must the jury find for him to prevail? A) No additional facts. B) That a reasonable person in the same circumstances would have believed that the woman was about to shoot. C) That the woman was at fault in raising a black object toward the man while threatening him. D) That the woman was the original aggressor.

B If the man prevails, it will be because the jury determined that he acted reasonably under the circumstances. One may act in self-defense not only where there is real danger but also where there is a reasonable appearance of danger. An honest but mistaken belief that the woman was about to shoot would justify the use of deadly force by the man if a reasonable person would have acted similarly under those circumstances. The test is an objective one-an honest belief alone is not sufficient (D) is incorrect because it does not resolve whether the man had the right to use deadly force. Even if the man started the altercation at the range, he would have the right to use deadly force if the woman escalated the fight with deadly force.

While traveling on a commercial bus line, a passenger was injured when some luggage fell on him. As required by applicable state law, the bus company's in-house attorney conducted an investigation and filed the required report with the state transportation department. The passenger subsequently filed a civil action against the bus company in federal district court, seeking compensatory damages for the injuries he suffered. During discovery, the passenger's lawyer served on the bus company a request for production of documents, including a request for the report that the bus company filed with the state. The bus company objected to the request for the report and refused to produce it on the grounds that the report was privileged and protected from discovery under the work product doctrine. It did, however, produce other documents that were requested. The passenger then filed a motion to compel production of the report. If the court finds that the bus company's claims of privilege and work product were not substantially justified, what orders must the court make relating to the passenger's request for production of the report? A) The court must sanction the bus company for improperly obstructing discovery by entering a default judgment against the bus company. B) If the passenger's attorney has attempted in good faith to resolve the dispute, the court must order the bus company to produce the report and to pay the passenger's reasonable costs in making the motion. C) Regardless of whether the passenger's attorney has attempted in good faith to resolve the dispute, the court must order the bus company to produce the report and to pay the passenger's reasonable costs in making the motion. D) If the passenger's attorney has attempted in good faith to resolve the dispute, the court must order the bus company to produce the report, but need not order the company to pay the passenger's reasonable costs in making the motion.

B If the passenger's attorney has attempted in good faith to resolve the dispute, the court must order the bus company to produce the report and to pay the passenger's reasonable costs in making the motion, including attorney's fees. Costs will be awarded unless: (i) the movant filed the motion before attempting in good faith to obtain disclosure or discovery without court action; (ii) the opposing party's nondisclosure, response, or objection was substantially justified; or (iii) other circumstances exist that make an award of expenses unjustified. [Fed R. Civ. P. 37(a)] None of these exceptions apply

A homeowner contracted for construction of a custom-built, elevated deck in his backyard. The deck's designer supervised the construction, which was carried out by several employees of a local building company. The homeowner was pleased with the appearance of the deck, but the first time he stepped on it, a support on one side of the deck gave way, causing the homeowner to fall and be injured. The homeowner brought an action joining the building company and the deck's designer as defendants, alleging negligence. In his complaint, he alleged that he does not know which of the defendants is responsible for the damages. Which of the following doctrines would be most helpful against the designer? A) Respondeat superior. B) Res ipsa loquitur. C) Contribution. D) Indemnity.

B Res ipsa loquitur will be most helpful against the designer. Res ipsa loquitur means the thing speaks for itself. It is appropriate in situations where an injury does not usually occur unless someone was negligent and the plaintiff does not know which of the defendants caused the injury. While res ipsa loquitur is sometimes not available where more than one person may have been in control of the instrumentality causing the injury, it is available in a case where a particular defendant had the power of control over the site of the injury. Even if the homeowner does not know why the deck collapsed, the deck's designer would be responsible because he designed the deck and was supervising the construction. Hence, res ipsa loquitur likely could be used.

A buyer agreed to buy a limited edition guitar from a seller for $12,000 and a contract memorializing the agreement was signed by both parties. The next day, after the seller received an offer of $20,000 for the guitar, he called the buyer and said that he could not sell the guitar to him for $12,000. The buyer did not respond. On the delivery date, the buyer fails to tender $12,000 and the seller does not deliver the guitar. On these facts: A) The seller can recover from the buyer for breach of contract. B) The buyer can recover from the seller for breach of contract. C) Neither the seller nor buyer can recover until one of the parties tenders performance. D) The contract is terminated.

B The buyer can recover because the seller's phone call was an anticipatory repudiation. Anticipatory repudiation occurs where a promisor, prior to the performance time, unequivocally indicates that he cannot or will not timely perform, allowing the nonrepudiator the option of suspending performance and waiting to sue until the performance date, or to sue immediately. The seller's phone call was an unequivocal statement that he would not sell the guitar for $12,000. This repudiation excused the buyer's duty to tender $12,000 on the delivery date.

In litigation over whether an uncle conveyed a parcel of land to his nephew, the nephew wishes to offer into evidence a tape recording of his uncle made by a well-known oral historian at the nearby state university. The voice on the tape is discussing various conveyances of the parcel of land and other property owned by the uncle. The nephew wishes to have the historian testify that the voice on the tape is the uncle's. If the court allows the historian to testify, it will be because: A) The historian is testifying regarding an admission by a party-opponent. B) The historian has heard the uncle speak before. C) The historian became familiar with the uncle's voice before the dispute over the property arose. D) The historian's experience as an oral historian qualifies him as an expert in voice recognition.

B The court will allow the historian to testify as to the identity of the voice simply because he is familiar with the uncle's voice. Where the identity of a speaker is important, the oral statements require authentication as to the identity of the speaker. A voice, whether heard firsthand or through a tape recording, may be identified by the opinion of anyone who has heard the voice at any time. As long as such a foundation is laid to show familiarity with the voice, a lay opinion as to the identity of the speaker is permissible. Thus, because the historian became familiar with the uncle's voice when he made the tape recording, he will be permitted to testify that the voice on the tape was the uncle's.

Legislation permitting states to completely ban the sale of cigarettes and other tobacco products was passed by Congress and upheld by the United States Supreme Court. Following the lead of other states, a state legislature declined to enact a complete ban on tobacco products. However, it passed a compromise measure that banned all advertisements for cigarettes and tobacco products by any print or broadcast media located in the state. A state tobacco distributor that wished to advertise in local newspapers brings an action in federal court to challenge the state statute. How is the court likely to rule? A) The statute is unconstitutional, because by choosing not to exercise its right to impose a complete ban on the sale of tobacco products, the state can no longer claim that the regulation of advertising serves a substantial government interest. B) The statute is unconstitutional, because the right to ban all truthful advertising for a product is not automatically justified by the right to ban the sale of a product entirely. C) The statute is constitutional, because the state's power to ban advertising for a product is implicit in its power to ban the product altogether. D) The statute is constitutional, because the ban on advertising constitutes a restriction on commercial speech that is rationally related to the legitimate state interest in reducing the use of tobacco products.

B The court will probably find the statute unconstitutional as an improper restriction of commercial speech. If the speech regulated concerns a lawful activity and is not misleading or fraudulent, the regulation will be valid if it (i) serves a substantial government interest, (ii) directly advances the interest, and (iii) is narrowly tailored to serve the substantial interest. While this test does not require that the least restrictive means be used, there must be a reasonable fit between the legislation's end and the means chosen. The greater the restriction on speech, the less likely it will be deemed to be reasonable. A complete ban on truthful advertising of a lawful product is very unlikely to be upheld because such a restriction is not narrowly tailored.

A homeowner and a carpenter entered into an oral agreement, under which the carpenter agreed to construct an addition to the homeowner's home, using materials supplied by the homeowner, in exchange for $20,000. After the work had been completed but before the homeowner had made any payment, the carpenter called the homeowner and instructed him to pay the $20,000 due on the addition work to a creditor of the carpenter. If the homeowner fails to pay the creditor, and the creditor thereafter brings an action against the homeowner for $20,000, is the creditor likely to prevail? A) Yes, because the creditor was the intended beneficiary of the contract between the homeowner and the carpenter. B) Yes, because there has been a proper assignment by the carpenter. C) No, because personal service contracts are not assignable. D) No, because the performance was not executory on both sides at the time of the assignment.

B The creditor will prevail because the carpenter has made a valid assignment of his right to payment from the homeowner. The general rule is that all contractual rights can be assigned, and all the carpenter assigned here was his right to payment.

A statute passed by both houses of Congress and signed by the President authorizes a federal agency to select a site for and to construct a monument honoring members of the capitol police force killed in the line of duty. The statute appropriates the necessary funds but provides that the funds may not be expended until both houses of Congress have adopted a concurrent resolution, not subject to presentment to the President, approving the agency's plans for the monument's location and design. Is the provision requiring further congressional approval before expenditure of the funds constitutional? A) No, because decisions regarding the placement and design of government-owned structures are an exclusively executive function with which Congress may not interfere by any means. B) No, because the provision amounts to an unconstitutional legislative interference with an executive function. C) Yes, because Congress may attach reasonable conditions to its appropriation of funds to executive departments, and its special interest in the members of its own police force makes the provision a reasonable condition. D) Yes, because the provision is part of a statute that was passed by both houses of Congress and signed by the President.

B The enactment of laws by Congress requires passage of the law in both houses (bicameralism) and approval of the law by the President (that is, the presentment requirement) or an override of a presidential veto. Here, Congress passed a law and the President signed it, but Congress sought further control by requiring expenditures to be approved specifically by Congress without presentment to the President. Such a requirement usurps the power of the executive branch to execute laws and places it in the hands of Congress in violation of the doctrine of separation of powers.

A police officer saw a car containing three teenagers driving slowly down the street at 1 a.m. She waited for it to go by her and, after it was far enough ahead, started to follow it. Several blocks later, the car rolled through a stop sign. The officer immediately pulled the car over and requested the driver's license. A license check showed that the driver had five outstanding parking tickets. A statute in the jurisdiction permits an arrest to be made if a driver has four or more outstanding parking or traffic violations. The officer decided to take the driver in on the tickets. She informed the driver that he was under arrest and asked him to step out of the car. When the driver got out, the officer patted him down and found a gun in his waistband. Calling for backup, she decided to haul all three teenagers to jail. Subsequent testing showed that the gun had been used in a recent homicide during a store robbery by three young men. One of the passengers made a motion to prevent the introduction of the gun at his trial for murder and robbery. How should the judge rule? A) Deny the motion, because the gun was found after the driver had been arrested. B) Deny the motion, because the officer lawfully stopped the car. C) Grant the motion, because the officer had no valid reason to be following the automobile. D) Grant the motion, because the officer had not arrested the driver for suspicion of robbing the store or committing the homicide.

B The judge should deny the motion. Evidence will be suppressed if it was obtained in violation of the defendant's constitutional rights. Each passenger in a car has standing to challenge a stop of the car. If a stop is invalid, under the fruit of the poisonous tree doctrine, evidence obtained as a result of the invalid stop will be suppressed. Here, the officer had a valid reason to stop the car. An officer may stop a car for violating a traffic law, and here the driver of the car failed to stop at a stop sign.

A landowner owned a large tract of undeveloped land in fee simple. Although no excavation had been done on the land, it was believed to contain gold. The landowner therefore began to mine the land, financing his operation with a $100,000 mortgage to a bank. Subsequently, the landowner sold all of the interest in gold on the land to a miner. Shortly thereafter, the landowner conveyed his ownership in the land to a mining company. Realizing that none of their interests had been recorded, the bank recorded its mortgage first, the miner recorded her deed second, and the mining company recorded its deed third. None of the parties dealing with the landowner had any knowledge of the others at the time of their transactions. The jurisdiction in which the land is located has the following statute: "No conveyance or mortgage of an interest in land is valid against any subsequent purchaser for value without notice thereof, unless it is recorded." If the mining company brings an action to quiet title in the land, what is the most likely result? A) Because the rights of the bank, the miner, and the mining company are different in nature, the court would most likely validate all of the interests, with the mining company having the ownership subject to the payment of the mortgage to the bank. B) The mining company would be successful in quieting title to the land. C) The bank's mortgage would be declared valid because it is first in time to all of the grants by the landowner concerning the land. D) The bank's mortgage would take priority over the miner and the mining company because the bank had no notice at the time it recorded its mortgage.

B The mining company would likely be successful. Under a notice statute, which the jurisdiction in this question has, a subsequent bona fide purchaser prevails over a prior grantee who fails to record. The important fact under a notice statute is that the subsequent purchaser had no actual or constructive notice at the time of the conveyance, not at the time of recording. When the property was conveyed to the mining company, it had neither actual nor constructive notice of the conveyances to the bank or the miner, whose interests were not recorded at that time. Therefore, the mining company was a bona fide purchaser and would be entitled to protection under the statute.

A 13-year-old boy who lived on a farm with his parents in a rural area had learned to drive the family's tractor when he was 11, and had operated it regularly since then. A state statute permitted persons without a driver's license to operate farm vehicles on public roads for short distances. One morning the boy took the tractor onto a public road to reach one of the outlying fields a few hundred yards away. As he neared the field he was distracted by a girl riding by on a bicycle, and cut in front of a milk delivery truck that was starting to pass him. The truck swerved off the road, injuring the driver. If the driver sues the boy to recover damages for his injuries, which of the following statements is most correct regarding the standard of care to be applied? A) The state statute replaces the general common law standard of care with a statutory standard. B) The trier of fact should take into account the boy's experience at driving a tractor when considering the applicable standard of care. C) Persons 13 years of age or older are held to the same standard as adults. D) An adult standard of care will not be applied because it is common in that region for children of that age to be operating tractors.

B The most correct statement is that the trier of fact should take into account the boy's experience when considering the applicable standard of care. The usual standard of conduct to which a child must conform is that of a child of like age, education, intelligence, and experience. However, a child must conform to an adult standard of care (the reasonable person standard) when engaging in a potentially dangerous activity in which usually only adults engage. But even when the reasonable person standard applies, someone with knowledge or experience superior to that of the average person is required to use that knowledge or experience. While the facts do not establish that the boy does have superior knowledge or experience, the trier of fact must evaluate that possibility, taking into account the fact that the boy had driven a tractor regularly since he was 11 years old.

A toy collector had purchased 10 antique toys over the last several years and had had them restored by an expert in toy restoration. On June 1, the collector sent the 11th antique toy to the expert with a signed note that read: "Here is another toy for you to restore. As with all prior jobs, I will pay $500 for the work, but no more." On June 4, after receipt of the collector's June 1 note and the toy, the expert began restoring the toy. On June 6, the collector unexpectedly died. On June 7, unaware of the collector's death, the expert sent the collector a note that stated that the restoration work had begun on June 4. The following day, the expert learned of the collector's death. Does a contract exist that binds the expert and the collector's estate? A) Yes, because the expert sent the June 7 note before learning of the collector's death. B) Yes, because the offer was accepted before the collector's death. C) No, because the collector died before the expert sent the June 7 note. D) No, because the offer lapsed when the collector died.

B The offer was accepted before the collector's death. The collector's letter was an offer. It created a reasonable expectation in the expert that the collector was willing to enter into a contract on the basis of the offered terms. Unless an offer specifically provides that it may be accepted only through performance, it will be construed as an offer to enter into a bilateral contract and may be accepted either by a promise to perform or by the beginning of performance. Here, the expert began performance and, thus, accepted the offer when he began restoring the toy on June 4 - two days before the collector died on June 6th.

A state study indicated that an inordinately high percentage of homeless in the state were afflicted by alcoholism or addiction to illegal drugs. The legislature therefore decided to levy a special tax, with all proceeds marked for rehabilitative services for the homeless. However, the legislators determined that direct taxes on alcoholic beverages would be resented by the citizenry. Lobbyists from the state's growing wine industry also objected to anything that would retard the industry's development. There were no breweries or distilleries within the state. Thus, a tax was eventually passed requiring newspapers and magazines of general circulation published in the state to be taxed at a rate of 20% on all advertising space sold for beer or distilled spirits promotions. For certain historical reasons, a high proportion of the advertising revenue of a particular small newspaper within the state came from beer and wine ads. The publisher of the small paper filed suit to have the tax declared unconstitutional. A major wholesale beer and liquor distributor located within the state and several out-of-state brewers and distillers who sold and advertised their products in the state also joined in the suit as plaintiffs. If the tax is declared unconstitutional, what is the most likely reason? A) The tax burdens interstate commerce by exempting advertisements for the local wine industry from the tax, while the ads of out-of-state brewers and distillers are subject to the tax. B) The tax infringes on freedom of the press, which is guaranteed by the First and Fourteenth Amendments. C) The tax is unconstitutional because it is not properly apportioned D) The tax violates the Equal Protection Clause of the Fourteenth Amendment, because it does not treat all alcoholic products equally.

B The tax unconstitutionally burdens the freedom of the press. Press and broadcasting companies can be subject to general business taxes, but a tax applicable only to the press or based on the content of a publication will not be upheld absent a compelling justification. Mere need for revenue probably is not a sufficiently compelling interest. (A) is incorrect because there is no unconstitutional burden on interstate commerce here. The law treats all businesses subject to the tax (namely breweries and distilleries) equally, and so is not protecting local business against out-of-state competition. The fact that the law treats breweries and distilleries differently from wineries and that the state has no breweries or distilleries but does have wineries probably does not change this, because a court will probably find these to be distinct businesses for purposes of advertising.

In a drug raid, police in a city searched 25 apartments selected at random in a 300-unit housing project. In a class action approved by the federal court, the 25 tenants sued the city for violation of their constitutional rights. The tenant named as class representative gave notice to all unnamed class members, including another tenant who decided not to opt out. The class action was then certified as a "common question" type. After negotiating with the class representative, the city police agreed to pay each tenant $500 and to conduct no further raids without proper warrants. The representative and the city signed a settlement agreement and a stipulation of dismissal of the class action. The other tenant objects to the amount of damages he is to receive and would rather opt out now and proceed on his own. May the tenant opt out now? A) Yes, because a class member of a "common question" class action always has the right to opt out. B) Yes, if the court allows the tenant a second opportunity to opt out. C) No, because the tenant did not opt out after receiving notice of the class action. D) No, if class certification was proper.

B The tenant may opt out if the court allows class members a second opportunity to opt out. In a "common question" class action, a judge may refuse to approve a settlement of a class action unless the class members are given a second opportunity to opt out.

A homeowner was injured when an automatic cutoff switch failed to function on a snowblower he was using. The cutoff switch had functioned well for a year after he purchased the snowblower but failed after the machine had been improperly repaired by a mechanic. The snowblower's operating manual contained a clear and prominent warning against making the very alteration to the switch mechanism that was made by the mechanic. The mechanic, however, did not have a manual available when he repaired the snowblower. Does the homeowner have a viable claim against the manufacturer of the snowblower for damages? A)No, because the homeowner was contributorily negligent in failing to furnish the snowblower's manual to the mechanic. B) No, because the injury resulted from a substantial alteration of the snowblower by a third party. C) Yes, because a defect in the snowblower caused the homeowner's injury. D) Yes, because the manufacturer should have made the manual available to repair personnel.

B To hold a commercial supplier liable for a product defect, the product must be expected to, and must in fact, reach the user or consumer without substantial change in the condition in which it is supplied. Here, the cutoff switch that failed had functioned well for a year after the homeowner purchased the snowblower but failed after the machine had been improperly repaired by a mechanic. The mechanic's alteration to the switch mechanism, which the manual warned against making, will preclude the homeowner from recovering against the manufacturer.

A construction worker was working at the construction site of a new building. An open elevator, which had been installed in the building by the elevator manufacturer, was used to haul workers and building materials between floors. While the worker was riding the elevator, it stalled between floors due to a manufacturing defect in the elevator. The worker called for assistance and was in no danger, but after waiting 15 minutes for help, he became anxious and jumped 12 feet to get out. He severely injured his back when he landed. In an action by the worker against the elevator manufacturer to recover for his back injury, is the worker likely to obtain a judgment for 100% of his damages? A) No, because such risks are inherent in construction work. B) No, because the worker was not in danger while on the stalled elevator. C) Yes, because the elevator stalled due to a manufacturing defect. D) Yes, because the worker was falsely imprisoned in the stalled elevator.

B Under contributory negligence principles, a plaintiff is required to act as a reasonably prudent person, the same as the defendant. A plaintiff who fails to act reasonably and who negligently contributes to his injury will have his recovery reduced in jurisdictions following a pure comparative negligence rule (which is the rule applicable to MBE questions). In a case where contributory negligence is shown, the trier of fact weighs the plaintiff's negligence against that of the defendant and reduces the plaintiff's damages accordingly. Here, the worker was in no danger in the stalled elevator but nevertheless jumped 12 feet to the ground after waiting 15 minutes. The trier of fact is likely to find that the worker was at least partly at fault for his injury by jumping from that height. Hence, he will not obtain a judgment for 100% of his damages.

A brick mason was hired by a builder under a written one-year contract, at an annual salary of $45,000, with employment to begin on March 1. Because the builder was unable to secure enough building contracts to keep all its employees busy during the season beginning March 1, it notified the brick mason on February 15 that it could not afford to employ him as a mason. At the same time, however, the builder offered to employ the mason, for the same contract period, as a night guard at an annual salary of $25,000. The mason declined the offer and remained unemployed during the year. No employment for brick masons was available in the community during the year, but the mason could have obtained other employment as a day laborer that would have paid up to $25,000 a year. At the end of the year, in an action against the builder for breach of contract, how much, if anything, is the mason entitled to recover? A) $20,000 (the $45,000 contract price less the $25,000 the mason could have earned in other employment) B) $45,000 (the contract price). C) Nothing, because the builder did not act in bad faith when it discharged the mason. D) Nothing, because the mason did not mitigate his damages.

B When an employer breaches an employment contract, the standard measure of the employee's damages is the full contract price. However, the nonbreaching party cannot recover damages that could have been avoided with reasonable effort. If the breaching employer can prove that a comparable job in the same locale was available, then contract damages against that breaching employer for lost wages will be reduced by the wages that the plaintiff would have received from that comparable job. Here, there was no comparable employment available in the community for the mason. The only jobs available, a night guard or a day laborer, were not the same type of work as a brick mason and were at a significantly lower salary.

A defendant is on trial for the murder of the victim, who was found beaten to death in his home. Evidence already presented has shown that the victim was killed when no one was at home except for the victim and his dog. The prosecution wishes to call a neighbor to the stand who is prepared to testify that she went to the victim's home the day after his murder and that when the defendant came by, the dog ran to a corner, where he cringed and whimpered. The neighbor is also prepared to testify that the dog is normally a very friendly dog, usually greeting visitors to the house, including the defendant, by approaching them with his tail wagging. The defense objects to the neighbor's proposed testimony. How should the court rule on the neighbor's testimony regarding the dog's behavior? A) Admissible, because the dog could be brought into court for a demonstration of his reaction to the defendant. B) Admissible, as circumstantial evidence against the defendant. C) Inadmissible, because the dog may have been reacting as he did for reasons other than those implied by the neighbor's testimony. D) Inadmissible, because even though the testimony has probative value, such value is outweighed by its prejudicial nature.

B Generally, all relevant evidence is admissible unless it is barred by a specific exclusionary rule or by the general balancing test of Rule 403, which permits exclusion of relevant evidence if its probative value is substantially outweighed by the danger of unfair prejudice, confusion of the issues, etc. The neighbor's testimony is relevant because the dog's behavior when the defendant came by tends to prove circumstantially (i.e., indirectly) the prosecution's contention that the defendant beat the victim to death (in the dog's presence). The neighbor is competent to testify as to the dog's behavior toward the defendant both before and after the murder, and no other competency rule warrants excluding the testimony; hence, it should be admitted. The trier of fact will attack the creditability

To help reduce a rising crime rate among teenage boys in a city, a scout leader decided to organize an overnight jamboree to get teens interested in scouting. The scout leader met with the city's parks commissioner and requested a permit to camp at a large city park located on the oceanfront. The parks commissioner told the scout leader that a city ordinance prohibited large organized use of the park during the evening and all overnight camping. The commissioner explained that the city wished to keep the park open for general use during the evening, when most people were off work, and the park was cleaned overnight. The scout leader brought an action in federal district court, seeking to compel the city to allow overnight camping for this one special occasion. If the court determines that the ordinance is valid, what will be the basis for its decision? A) The ordinance is rationally related to a legitimate government interest and burdens the First Amendment rights involved no more than is reasonable under the circumstances. B) The ordinance is narrowly tailored to serve an important government interest and does not unreasonably limit alternative channels of communication. C) The ordinance is substantially related to a legitimate government interest and burdens the First Amendment rights involved no more than is reasonable under the circumstances. D) The ordinance is rationally related to a legitimate government interest and does not unreasonably limit alternative channels of communication.

B The court will base its decision on its determination that the ordinance is narrowly tailored to serve an important government interest and does not unreasonably limit alternative channels of communication. While the First Amendment protects the freedoms of speech and assembly, the government may reasonably regulate speech-related conduct in public forums through content-neutral time, place, and manner regulation. To avoid strict scrutiny and be upheld, government regulations on speech and assembly in public forums must be content neutral and narrowly tailored to serve an important government interest, and must leave open alternative channels of communication. Here, the ban on camping overnight in the park, a content-neutral regulation of a public forum, would be evaluated by the court using the standard in choice

A defendant was charged with sale and distribution of illegal drugs in violation of the state criminal code. The trial took place in the county in which most of the illegal sales occurred. The defendant was convicted and, based on the prosecutor's recommendation, received a sentence of 18 months from the judge. A prosecutor in a neighboring county who was running for reelection on a law and order platform publicly denounced the sentence. That prosecutor obtained a grand jury indictment against the defendant for sale and distribution of illegal drugs to juveniles, some of which had occurred in that county. This offense carried more severe penalties and required the additional element that the defendant knew drugs were being sold to juveniles. Will the defendant be successful in a motion to dismiss the indictment? A) Yes, because the prosecutor had improper motives in bringing the case before the grand jury. B) Yes, because the counties are not separate sovereigns. C) No, because the double jeopardy protection does not prohibit subsequent prosecution after a conviction or hung jury, only after an acquittal. D) No, because the subsequent offense requires proof of an element that the first offense does not.

B The defendant's motion to dismiss will be successful. Under the Double Jeopardy Clause, a defendant may not be retried for the same offense once jeopardy attaches. While this protection does not apply to trials by separate sovereigns, the two counties enforcing state law are not separate sovereigns. The offense is a lesser included offense and attachment of jeopardy for a lesser included offense generally bars retrial for the greater offense

An art student ordered from an art museum's gift shop a miniature reproduction of a favorite statue, which the museum's flyer indicated was for sale only at the gift shop. The student's order included a check to cover the price. The museum received and processed the order, but by mistake sent, by common carrier, a reproduction of a different statue. The statue was stolen from the carrier en route to the student's home. Between the student and the museum, who bears the loss? A) The loss falls on the museum because it was the museum that selected the carrier and made the arrangements for shipment. B) The loss falls on the museum because it sent the wrong statue. C) The loss falls on the student because the risk of loss shifted to the buyer when the museum received and accepted the payment. D) The loss falls on the student because the risk of loss passed to him when the statue was delivered to the carrier.

B The museum suffers the loss because the risk of loss was still with it when the statue was stolen. The UCC presumes a contract is a shipment contract in the absence of a contrary agreement. In a shipment contract, the seller must ship the goods by carrier but is not required to tender them at a particular destination. (A "ship to" address does not make a contract a destination contract.) In a shipment contract, the risk of loss generally passes to the buyer when the goods are delivered to the carrier. There is an exception, however, if the buyer has a right to reject the goods. In that case, the risk of loss does not pass to the buyer until the defects are cured or the buyer accepts the goods. The UCC requires perfect tender; that is, the goods and their delivery must conform to the contract in every way. Here, the shipment of the wrong statue (a nonconforming good) constituted a breach, and the buyer had a right to reject it. Thus, the risk of loss did not pass to the buyer, and the loss falls on the museum with respect to the theft.

A state legislature enacted a statute providing for loaning certain textbooks on secular subjects to students in all public and private schools. In accordance with the statute, the state board of education distributed textbooks to a private school that offered religious instruction and admitted only Caucasian students. Which of the following is the strongest argument against the constitutionality of free distribution of textbooks to the students at the private school? A) A state may not constitutionally aid private schools through distribution of textbooks.A state may not constitutionally aid private schools through distribution of textbooks. B) Segregation is furthered by the distribution of textbooks to these students.Segregation is furthered by the distribution of textbooks to these students. C) The distribution of textbooks advances religion because it is impossible to separate their secular and religious uses.The distribution of textbooks advances religion because it is impossible to separate their secular and religious uses. D) The distribution of textbooks fosters excessive government entanglement with religion.

B The strongest argument is that state provision of textbooks to the segregated private school violates the Equal Protection Clause by giving state support to a racially segregated educational process. This may not be a winning argument but it is clearly the best of the lot.

A boater taking his new powerboat out on a large lake ran out of gas because of a defective seal in the gas tank. The defect was not discoverable by an ordinary inspection. His frantic signaling alerted the captain of a sightseeing boat passing by. The captain pulled up alongside to assist and attempted to restart the boat. A spark ignited a pool of gas that had leaked from the gas tank and collected in the lower part of the boat, causing an explosion and fire. The captain was severely burned and died from his injuries. The captain's estate brought a wrongful death action based on strict liability against the powerboat dealer and the manufacturer. Evidence at trial established that the dealer had sold the manufacturer's boats for years without any problems reported by customers. Can the captain's estate recover any damages from the dealer? A) Yes, unless the jury finds that the boater was negligent in failing to investigate where the gas had gone. B) Yes, because harm to someone in the captain's position was a foreseeable result of the gas leak. C) No, because the dealer had no reason to anticipate that the manufacturer assembled the gas tank improperly. D) No, because the captain did not have a sufficient relationship to the boater to make the dealer liable for the captain's death.

B) Recovery in a wrongful death action is allowed only to the extent that the deceased could have recovered in a personal injury action had he lived. The captain could have recovered from the dealer in a products liability action based on strict liability because the dealer is a commercial supplier of the boat and is liable if it sold a product that was so defective as to be unreasonably dangerous. The defect in the assembly of the gas tank was unreasonably dangerous because it allowed gas to leak out and collect where it could be ignited. The dealer would be liable to the captain, despite the fact that he was not in privity with the dealer, because he was a foreseeable plaintiff. The disabling effect of the gas leak made it foreseeable that someone passing by would come to the boater's assistance and thereby come within the zone of danger from the leak (i.e., danger invites rescue). The explosion that resulted from the leak was the actual and proximate cause of the captain's death. Therefore, the captain's estate can recover damages from the dealer.

A mechanic sued his former employer in federal court, claiming that the employer had discharged him because of his age in violation of federal law. The employer answered, denying the claims and promptly moving for summary judgment. In support of the motion, the employer attached the mechanic's employment evaluations for the past three years, which rated his skills and performance as poor and culminated in a recommendation for his discharge. What is the mechanic's best argument to defeat the summary judgment motion? A) The allegations in the complaint conflict with the mechanic's employment evaluations, raising a genuine dispute as to material facts. B) The employer cannot rely in his motion on matters outside the pleadings. C) The essential facts are unavailable to the mechanic and therefore discovery is required. D) The motion was filed before the close of discovery.

C (C) is correct. If the mechanic (the nonmovant) shows by affidavit or declaration that he cannot present facts essential to justify his opposition to the summary judgment motion, Rule 56(d) authorizes him to ask the court to defer action or deny the motion to allow time to obtain affidavits or declarations or to take discovery. The employer moved for summary judgment right after answering and before any discovery. That timing would support defeating the summary judgment motion at this time. (A) is incorrect. Under Rule 56(c), a party asserting that a fact is genuinely disputed must support the assertion by citing particular parts of the record, including affidavits or declarations, stipulations, or discovery materials. The mechanic cannot simply rely on the complaint allegations to rebut the employer's evidence but must support his factual position with his own evidence that a factual dispute exists. If he cannot do so, Rule 56(d) authorizes him to ask the court to defer action or deny the summary judgment motion to allow time to obtain affidavits or declarations or to take discovery.

A state initiated a criminal prosecution against the owner of a video store for selling a video that consisted entirely of pictures of nude sunbathers on a beach in a foreign country where nude public sunbathing is common. The state charged that selling the video violated its anti-obscenity law. The store owner defended on the ground that the prosecution violated his constitutional right to freedom of speech. Should the store owner prevail in this defense? A) No, because the store owner is engaged in the commercial sale of the video, which is not protected by the First and Fourteenth Amendments. B) No, because the video consists entirely of portrayals of nudity, appeals to the prurient interest of viewers, and lacks serious social value as a whole. C) Yes, because mere portrayals of nudity are insufficient to justify a finding that the video is obscene as a matter of constitutional law. D) Yes, because the portrayals of nudity occurred outside the United States, and therefore the state lacks a compelling interest in applying its anti-obscenity law to the sale of the video.

C (C) is correct. The Supreme Court has defined obscenity as a description or depiction of sexual conduct that, taken as a whole by the average person, applying contemporary community standards: (1) appeals to the prurient interest in sex; (2) portrays sex in a patently offensive way; and (3) using a national standard, does not have serious literary, political, or scientific value.

A city ordinance provided that anyone who wanted to speak in a public park must have a permit to do so issued by the city. The ordinance granted the mayor the power to issue or deny such permits based on the mayor's judgment of whether the speech would be "in the public interest." The mayor has never denied a permit to anybody desiring to speak on a political topic. A city resident who was unhappy with the city government went to a public park in the city square. There, the resident made a 10-minute speech accusing the mayor and the city council of gross incompetence and urging voters to "throw the rascals out" at the next election. The city resident had not applied for a permit. After the resident completed his oration, the police arrested him and charged him with violating the permit ordinance. Would a conviction of the resident be constitutional? A) Yes, because the resident did not have a permit to speak, and a municipality has the right to regulate the time, place, and manner of speech. B) Yes, because the mayor would have issued the permit, because the resident's speech was on a political topic. C) No, because the ordinance is void on its face. D) No, if the resident could prove that the mayor would not have issued him a permit to speak.

C A conviction of the resident would not be constitutional because the ordinance is void on its face. Although a municipality can place reasonable time, place, and manner restrictions on certain aspects of speech, it may not adopt a regulation that gives officials broad discretion over speech issues. If a statute gives licensing officials unbridled discretion, it is void on its face, and speakers need not even apply for a permit. They may exercise their First Amendment rights even if they could have been denied a permit under a valid law, and they may not be punished for violating the licensing statute. Here, the law allows the mayor to grant or deny permits based on his assessment of public interest. This is too much discretion to be valid. Therefore, the ordinance is void

A smoothie retailer and a fruit processor entered into an oral agreement that provided that the processor would deliver to the retailer 100 barrels of fruit each month at a price of $10 per barrel, with delivery on the first of the month and payment of the $1,000 to a creditor of the fruit processor on the 15th of each month. However, when the agreement was reduced to a writing, the fruit processor's manager inadvertently wrote $20 per barrel, and neither party noticed before signing. The creditor first learned of the agreement between the parties when he received a copy of it the day after it was signed, showing that he was to receive $2,000 per month. One day later, the retailer discovered the mistake and alerted the food processor. The parties prepared a revised writing reflecting the correct contract price of $10 per barrel, and also agreed in writing that the retailer would receive a $2 per barrel discount the first month because it discovered the mistake by the fruit processor. The first delivery under the contract was made two days late, on the third of the month. On the 15th of the month, the creditor demanded payment of $2,000 from the retailer according to the terms of the original writing. If the retailer contends that it is not liable to pay the full $2,000, which of the following would NOT be relevant to its defense? A) The parties had modified the contract to provide for a $2 per barrel discount the first month. B) The parties had originally agreed that the price per barrel would be $10, and neither party noticed before signing that the manager of the fruit processor had inadvertently written $20 in the contract C) The fruit processor owed its creditor only $1,600. D) The fruit processor was late with its first delivery.

C Any defense that the fruit processor might have with respect to the money it owed to the creditor would not provide the retailer with a defense. If the promisor has made an absolute promise to pay the third-party beneficiary (and not simply a promise to pay whatever the promisee owed him), the promisor cannot assert the promisee's defenses. Hence, the fact that the processor owed only $1,600, even if it could be asserted as a defense by the processor against the creditor, cannot be asserted as a defense by the retailer. If the grantor intends the recording of the deed to be the final act in vesting title in the grantee, then such recording creates a presumption of delivery even where the grantee did not know of the recordation.

While the defendant was committing a robbery, he shot and killed the victim. The defendant is charged with first degree murder in a state that defines first degree murder as murders committed with premeditation or deliberation or during the commission of burglary, arson, rape, or robbery, and defines second degree murder as all other murders. The state also defines voluntary manslaughter as the unlawful killing of a human being with malice upon a sudden quarrel or heat of passion, and it defines involuntary manslaughter as the unlawful killing of a human being without malice in the commission of an unlawful act, not amounting to an enumerated felony, or in the commission of a lawful act that might produce death in an unlawful manner or without due caution and circumspection. Assuming evidence to support, what explanation for the shooting would best help the defendant in avoiding conviction for first degree murder? A) In an act of resistance, the victim suddenly attacked the defendant and knocked him down, so the defendant pulled the trigger because he was afraid the victim was going to hit him again. B) The defendant had the gun for many years, it was old and rusty, and he did not think it would fire. C) The defendant had taken "angel dust" before the incident and does not remember getting a gun or holding up the victim. D) When the defendant tried to hold up the victim, the victim said, "Get out of here, you dirty bum, or I'll kill you," and the defendant became so upset that he did not know what he was doing.

C Because the defendant was charged with first degree murder, the theory of the case is most likely felony murder, and (C) is the only choice that sets out a theory to avoid a felony murder conviction. If the defendant was so intoxicated that he could not form the intent to steal, then he is not guilty of robbery, and there would be no "felony" from which the felony murder rule is to arise.

A man borrowed money from a lender and mortgaged land that he owned to secure repayment of the loan. Before he had completely repaid the loan, the man conveyed the land to an investor, who expressly assumed the loan. The note and mortgage did not contain a due-on-sale clause. After the investor had made several payments on the loan, she defaulted on two payments. The lender notified the man and the investor of its intention to accelerate the loan pursuant to the terms of the note and mortgage unless the default was cured within 60 days. When neither the man nor the investor made the required payment, the lender accelerated the loan and initiated foreclosure proceedings, naming both the man and the investor as party defendants. The foreclosure sale resulted in a deficiency. The lender has sought a deficiency judgment against only the man, because the investor has become insolvent in the meantime. Will the court likely find the man liable for the deficiency? A) No, because the investor's express assumption of the loan released the man from liability. B) No, because the lender must first seek to obtain a deficiency judgment against the investor. C) Yes, because even after the assumption, the man remains liable as a surety of the investor in the absence of a release from the lender. D) Yes, because the note and mortgage did not contain a due-on-sale clause.

C If a mortgagor sells property subject to a mortgage, the grantee takes subject to the mortgage, which remains on the land. If the grantee signs an agreement promising to pay the mortgage loan ("assuming the loan"), he becomes primarily liable to the lender, while the original mortgagor becomes secondarily liable as a surety. The mortgagee may opt to sue either the grantee or the original mortgagor on the debt. Here the man conveyed the land to the investor who expressly assumed the mortgage. That fact means that the investor is primarily liable, but the man remains secondarily liable. If the mortgagee opts to sue the original mortgagor, that party may in turn sue the assuming mortgagor, who is primarily liable. That won't help the man here because the investor is insolvent. (A) is incorrect. Assumption of a mortgage by a grantee makes the grantee primarily liable for the mortgage but does not release the original mortgagor. The original mortgagor remains secondarily liable as a surety. Thus, the investor's assumption of the loan did not release the man from liability.

A customer selected a new wallet at a local department store that the salesperson said was made of the finest calfskin and was stitched by hand. After having an opportunity to inspect the wallet, the customer bought it and left the store. A few moments later, he took out the wallet to transfer his cash and credit cards into it. On closer inspection, he noticed a small nick in the leather. He immediately went back to the department store and demanded a refund. The salesperson refused. If the customer sues for a refund, who will prevail? A) The customer, because there was a breach of contract. B) The customer, because he had a reasonable time after purchase in which to inspect. C) The department store, because the customer accepted the goods. D) The department store, because the customer did not give written notice of the breach.

C Once a buyer has accepted goods, his right to reject for nonconformity generally lapses and his only remedy is a suit for damages. Acceptance usually occurs when the buyer takes possession of the goods. In some cases, the buyer can revoke acceptance, but the breach must be substantial and the buyer must have a good reason for accepting the goods (i.e., something more than not taking the time to carefully inspect).

A radio station was sued by a former program host for defamation. The station moved to dismiss the complaint for failure to state a claim and for judgment on the pleadings. In response, the host voluntarily dismissed the action and filed a new action, alleging the same claims but also addressing the pleading defects raised by the station. The station moved again for judgment on the pleadings and accompanied the motion with affidavits asserting that the alleged defamatory statements were true. The host's attorney should respond to the new motion by doing which of the following? A) Dismiss the action again and refile, addressing the issues raised by the affidavits. B) Move to strike the affidavits because matters outside the pleadings are irrelevant in a motion for judgment on the pleadings. C) Request additional time to respond to the allegations in the affidavits. D) File a cross-motion for summary judgment.

C The attorney should respond by requesting additional time to respond to the allegations in the affidavits. A party making a motion for judgment on the pleadings and accompanying it with an affidavit or other matters outside the pleadings may in reality be making a motion for summary judgment, putting the wrong label on the motion. The court is expressly authorized to treat such a motion as one for summary judgment and to conduct subsequent proceedings on the motion in accordance with the rule on summary judgment, giving the parties full opportunity to present material made relevant by that rule. Under the rule for summary judgment, a party opposing the motion may present reasons why they need additional time to submit affidavits or obtain depositions. Given that this action is just at the pleading stage, the court will likely grant additional time for the host's attorney to pursue discovery and respond to the affidavits.

An owner and a builder executed a contract providing that the builder was to construct a residence on a specified lot according to plans and specifications. The total contract price was $800,000. No date was included in the contract for completion of the home. After the builder completed 60% of the residence, a flash flood from a nearby river partially eroded the lot but left the construction undamaged. The builder determined that it would cost an additional $1.7 million to repair the lot so that the residence can be constructed according to the plans. Without the additional lot repair work, the residence cannot be constructed at all. Which of the following states the probable legal consequences of the lot erosion? A) The builder may avoid the contract if the increased costs of construction would bankrupt him. B) The contract is void because of mutual mistake. C) The builder is discharged from his duties under the contract because of impracticability. D) The builder remains obligated to perform under the contract, but he may bring an action against the owner for the increased costs of construction.

C The builder will be discharged from his duties under the contract. Modern courts recognize that impracticability due to excessive and unreasonable difficulty or expense is a defense to breach of contract for nonperformance. Since the cost to the builder to perform under the original contract would exceed more than double what he would be paid, he likely would be excused from performance by commercial impracticability.

A grandfather told his granddaughter that she could have his house because he was moving to a retirement home, and entered into a valid contract to convey it to her. He promised her that he would have another wing added to the house in the back before turning it over to her, and entered into a written contract with a builder to construct the addition for his granddaughter. Before the grandfather had entered into the contract with the builder, the granddaughter had paid $5,000 for a 60-day option to purchase another house because she was not sure she would like the addition. However, when her grandfather showed her the plans for his house prepared by the builder, she liked it very much and decided to let her option to purchase the other house lapse. Shortly thereafter, the local zoning authority increased the minimum lot line setbacks, making it impracticable to put the addition on the back of the house. The builder offered to put an addition above the existing floor rather than in the back, and the grandfather agreed. After the granddaughter's option had lapsed, she discovered that the addition was now going up rather than in the back. She angrily demanded that the builder either build the addition according to the original specifications that she approved or pay her damages. The builder refused and the granddaughter filed suit. Who is more likely to prevail? A) The granddaughter, because she was an intended beneficiary of the contract whose rights had vested. B) The granddaughter, because the subsequent agreement between her grandfather and the builder to modify the construction was unsupported by consideration. C) The builder, because he may raise all defenses that he had against the grandfather against the granddaughter. D) The builder, because the granddaughter is merely an incidental beneficiary of the contract between the grandfather and the builder and, as such, has no power to enforce the contract against the builder.

C The builder will prevail because he may raise all defenses that he had against the grandfather against the granddaughter. The granddaughter is an intended third-party beneficiary of the contract between the grandfather and the builder. Generally, a third-party beneficiary has rights under the contract as soon as she does something to vest her rights (manifests assent to the promise, brings suit to enforce the promise, or materially changes position by justifiably relying on the promise). Here, the granddaughter materially changed her position by justifiably allowing her option on the other house to lapse. Generally, once the third-party beneficiary's rights have vested, the original contracting parties may not modify the contract without the assent of the third-party beneficiary. However, the third-party beneficiary is subject to any defenses that the promisor could have used against the original promisee, and here the builder could have used the defense of impracticability against the promisee. Therefore, he could use that defense against the granddaughter to avoid having to pay damages for not building the house as he originally agreed.

building contractor entered into a contract with the local college to remodel a residence hall during the summer. As specified by the contract, the work had to be completed before the fall semester began at the beginning of September. Because the contractor received a great deal of other maintenance business from the college, his price of $400,000 was significantly lower than other contractors and he was not going to demand payment until the work was completed. By the end of the first week in August, the contractor had completed 75% of the project and had expended $350,000 in labor and materials. At that time, however, a labor dispute between the contractor and his employees prompted most of the workers to walk off the job. Because prospects for a quick settlement of the dispute were doubtful, the contractor informed the college that he would not be able to meet the completion deadline. A week later, the college obtained another contractor who was able to finish the project by the end of August. The college paid him $150,000, which included a substantial amount of overtime for his workers. The increase in value of the residence hall due to the remodeling was $425,000. The original contractor, who had not been paid, files suit against the college, which files a counterclaim against him. What should the contractor recover from the college? A) Nothing, because the contractor breached the contract. B) $200,000 in restitutionary damages, which is the difference between its expenditures and the amount the college paid the other contractor to complete the work. C) $250,000 in restitutionary damages, which is the contract price minus the amount the college paid the other contractor to complete the work. D) $275,000 in restitutionary damages, which is the difference between the value of the completed remodeling and the amount the college paid the other contractor to complete the work.

C The contractor should be able to recover $250,000 in restitutionary damages. Where a builder in a construction contract breaches during the construction, the nonbreaching party is entitled to the cost of completion plus compensation for any damages caused by the delay in completing the building. Most courts, however, will allow the builder to offset or recover for work performed to date to avoid the unjust enrichment of the owner. This restitutionary recovery is usually based on the benefit received by the unjustly enriched party. If substitute performance is readily obtainable, damages are measured by the unpaid contract price minus the cost of completion (up to the value of the benefit received by the defendant). Here, the contractor's duty to complete the project was not discharged by impossibility; he could have hired another contractor to take his place or yielded to his employees' demands. Hence, the contractor's failure to complete the remodeling constituted a breach of contract and resulted in the college having to expend $150,000 to have the building completed on time. However, the contractor did not receive any payments for the work that he did before breaching; the college would be unjustly enriched if it does not have to pay for any of this work. The benefit of the completed remodeling is measured by the contract price, $400,000, because a restitutionary recovery here would be based on the failed contract between the parties and substitute performance is readily obtainable. This amount is reduced by the $150,000 cost of completion that the college can recover from the contractor, leaving a net recovery of $250,000 for him.

A restaurant owner in State A bought two large freezers from a manufacturer of commercial refrigeration equipment with its principal place of business in State B. Within one week and after being fully stocked with meat, one of the freezers broke down. The restaurant owner filed a state-based products liability action against the manufacturer in federal court in State A, and included a demand for a jury trial. Under the law in State A, jury verdicts do not need to be unanimous, but the Federal Rules of Civil Procedure require jury verdicts to be unanimous. At trial, the restaurant owner makes a motion asking the court to apply the State A law. How should the court rule on the motion? A) Grant the motion, because applying the federal rule may change the outcome of the case. B) Grant the motion, because, when a federal court has diversity jurisdiction, it is required to apply the substantive law of the state in which it is sitting. C) Deny the motion, because the Federal Rules of Civil Procedure apply in federal court as long as they are consistent with the Rules Enabling Act and not unconstitutional. D) Deny the motion, because the Supreme Court's balancing factors indicate that federal law should apply.

C The court should deny the motion. Under the Erie doctrine, when a state law-based claim is brought in federal court based on diversity of citizenship, the federal court generally applies the substantive law of the state in which it is sitting. However, where a specific federal statute or the Federal Rules of Civil Procedure are on point, the federal court must apply federal procedural law as long as the federal rule is valid. Under the Rules Enabling Act, a Federal Rule is valid if it deals with "practice or procedure" and does not "abridge, enlarge, or modify" a substantive right. Here, there is a specific federal procedural rule that is on point [Fed. R. Civ. P. 48], which requires jury verdicts to be unanimous, unless the parties agree otherwise. Since there is no evidence of agreement, the federal procedural rule will apply, and the motion should be denied.

A state that had experienced a great deal of labor unrest in recent years enacted a law restricting striking workers from obtaining benefits from the state's child care assistance program while on strike. In contrast, there were no restrictions on receiving child care benefits for those who voluntarily left employment. A worker whose child care assistance benefits were suspended when she went on strike sued the appropriate state officials in federal district court, challenging the constitutionality of the law. Which of the following is the standard for the court to apply in reviewing the constitutionality of the law? A) The state must show that the law is necessary to serve a compelling government interest because it interferes with the fundamental right to strike. B) The state must show that the law is substantially related to an important government interest because it places an undue burden on women, who are statistically more in need of child care assistance than men. C) The worker must show that the state did not have a rational basis for enacting the law, which is an example of economic or social welfare legislation. D) The worker must show that the state did not have an important government interest in discriminating between those on strike and those who voluntarily left employment.

C The worker must show that the state did not have a rational basis for enacting the law. The state statute here does not unduly burden any fundamental right or discriminate against a suspect class; rather, it simply withholds a state-supplied economic benefit to those engaging in strikes.

A plaintiff filed a civil action based on negligence against a defendant in federal district court, alleging that the defendant negligently ran a red light at an intersection and collided with the plaintiff's vehicle, causing the plaintiff's injuries. A week after the close of discovery, the plaintiff filed a motion for summary judgment on the issue of whether the defendant was negligent. With the motion, the plaintiff filed (i) his own sworn affidavit, which stated that the traffic signal was green as he entered the intersection, (ii) an affidavit of a witness who was driving the car behind him, which stated that the witness saw the entire incident and that the plaintiff's traffic signal was green as he approached and entered the intersection; and (iii) an affidavit of another witness, which stated that she saw the entire incident and that the defendant's signal had been red for several seconds before the defendant entered the intersection and was still red when the defendant entered the intersection. The defendant filed a response to the motion, noting that her answer denied negligence and further denied that her traffic signal was red, and argued that the issue of negligence and the issue of whether her traffic signal was red were in dispute, so those issues should be tried to a jury. How should the court rule on the plaintiff's motion? A) Deny the motion, because whenever a defendant's pleadings deny allegations in a plaintiff's complaint, the disputed issues must be submitted to trial. B) Deny the motion, because the defendant has no duty to produce evidence on the issue of negligence since the plaintiff has the burden of proof on that issue. C) Grant the motion, because the plaintiff supported his motion with substantial evidence indicating that the defendant was negligent, and the defendant did not come forward with admissible evidence in rebuttal. D) Grant the motion, because whenever a plaintiff's complaint properly states a claim for relief, a defendant may not rest on the denials in the defendant's pleading, but must produce evidence that negates the claim.

C The court should grant the plaintiff's motion. If a party fails to support an assertion of fact or fails to properly address another party's assertion of fact, the court may consider the fact undisputed for purposes of the motion, grant summary judgment if appropriate, give the party an opportunity to address the fact, or issue any other appropriate order. Here, the plaintiff came forward with affidavits supporting his contention that the light was red when the defendant entered the intersection. The defendant here must counter those affidavits with her own or risk summary judgment being entered against her.

A landowner owned a tract known as Section 35. He subdivided Section 35, and sold a lot to a neighbor. The warranty deed that conveyed the property included the following language: COVENANTS *** Purchaser shall have a privilege to hunt and fish on all lands owned by Seller in Section 35. * * * These covenants shall run with the land. Years later, the landowner sold the remaining land in Section 35 to a builder. Shortly thereafter, the neighbor died, leaving the lot to her granddaughter. The builder posted "no trespassing" signs on his land. The granddaughter brought an action for declaratory judgment against the builder to enforce the granddaughter's ability to hunt and fish on the builder's land. What would be the likely result? A) The granddaughter will win, because the warranty deed granted her an easement. B) The granddaughter will win, because the warranty deed granted her an irrevocable license. C)The granddaughter will win, because the warranty deed granted her a profit. D) The builder will win, because the warranty deed granted a revocable license.

C The granddaughter will win because she has a profit. A profit is a nonpossessory interest in land, allowing the grantee to enter on the land and remove resources of the land, in this case, fish and game. Moreover, the profit can be conveyed from the original grantee to a third party, as it was here from the neighbor to her granddaughter. A profit can be terminated in one of several ways, such as by abandonment or misuse, but the facts here do not indicate that termination has occurred

A landowner's will left his ranch to a rancher, his heirs, and assigns, so long as the property was used exclusively for ranch purposes, then to the landowner's grandson. The remainder of the landowner's property passed through the residuary clause of his will to the grandson. Seven years after the landowner's death, the rancher began strip mining operations on the ranch. The grandson brought an action to quiet title to the ranch against the rancher, and the rancher counterclaimed on the same theory. Who should prevail? A) The rancher, because the condition imposed on his interest under the will is void as violating the Rule Against Perpetuities. B) The rancher, because the condition imposed is a restraint against alienation. C) The grandson, pursuant to the residuary clause. D) The grandson, because the condition imposed is valid and he takes according to the subsequent provision.

C The grandson prevails because the ranch passed through the residuary clause. Under the Rule Against Perpetuities, the attempt to give the grandson an executory interest is void,

A trial judge presiding over a lawsuit plans to call a witness to the stand and question her. May the judge do so? A) Yes, but only if the witness has already been called and examined by one of the parties. B) Yes, but only if the witness is not testifying as an expert. C) Yes, but the parties are entitled to cross-examine the witness. D) No, because only parties may call and examine witnesses.

C The judge may call and examine the witness, but the parties are entitled to cross-examine the witness. A court is entitled to examine any witness called by any party, and may also call a witness on its own or at a party's request.

An uncle validly executed and notarized a deed conveying his beach house to his nephew, and then validly recorded the deed. When the nephew, who was experiencing financial difficulties, learned of the recordation of the deed, he immediately told his uncle that he did not want the beach house and could not accept such an expensive gift anyway. Later, the nephew filed for bankruptcy and the trustee in bankruptcy asserted an ownership interest in the beach house on behalf of the debtor's estate. The bankruptcy court ruled that the property belonged to the uncle and not to the nephew, and thus was not part of the debtor's estate subject to distribution. Which of the following is the strongest reason in support of the bankruptcy court's ruling? A) There was no presumption of delivery created by recordation of the deed because the nephew did not know of the recordation. B) The nephew's statements to the uncle were a constructive reconveyance of the property. C) There was never an effective acceptance of delivery of the deed by the nephew. D) The recordation of the deed was invalid because it was done without the nephew's permission.

C The nephew's express rejection of the deed was sufficient to rebut any presumption of acceptance. As a general rule, delivery of the deed is the final operative act to complete a conveyance of title to the grantee, because courts will infer the grantee's acceptance if the conveyance is beneficial to him. However, all courts will consider evidence that is contrary to the presumption or inference. Hence, the nephew's express rejection of the gift is sufficient to establish that no conveyance of the property took place.

A salesman was employed by a florist, who owned a retail shop adjacent to a large wholesale nursery. The owner of the nursery liked to use a brand-name artificial fertilizer for her plants, although other effective fertilizers were available at comparable prices. She stored a large quantity of the fertilizer in a heap on the nursery's property, as did many other nursery owners without incident. The fertilizer gave off fumes that caused the salesman to suffer lung irritation. Occasionally, the salesman's irritations became so bad that he had to take off from work and seek medical attention. After losing a few hundred dollars in wages and amassing a few hundred dollars in medical expenses, the salesman sued the nursery owner for damages. The court is likely to rule in favor of: A) The salesman, because the nursery owner had equally effective fertilizers available at comparable prices to the fertilizer used. B) The salesman, because the nursery owner is strictly liable for injuries caused by emissions from her property. C) The nursery owner, because the selection of the fertilizer was reasonable and it was stored in a reasonable manner. D) The nursery owner, because the salesman is merely an employee of the florist and does not own the property on which the shop is located.

C The only basis of liability that the salesman could use in his suit is negligence-i.e., that the nursery owner negligently selected and stored the fertilizer. Because the nursery owner acted reasonably in selecting and storing the fertilizer, she will prevail. (A) is wrong because the nursery owner would not be liable simply because other fertilizers were available

A grantor executed and delivered a deed to his son conveying his land as follows: "To my son for life, but if my son dies survived by his spouse and children, then to my son's spouse for life, with the remainder in fee simple to my son's children." A year later, the son died survived by his spouse and two offspring, a girl and a boy. The boy died intestate two days after the son, leaving one child as his only heir. The common law Rule Against Perpetuities is unmodified in the jurisdiction. What are the respective interests of the spouse, the girl, and the child in the land? A) The spouse has a life estate, the girl has an absolutely vested remainder, and the child has nothing. B) The spouse has fee simple ownership of the land, and the girl and the child have nothing. C) The spouse has a life estate, and the girl and the child have absolutely vested remainders. D) The spouse has a life estate, and the girl has a vested remainder subject to open.

C The spouse has a life estate, the girl has an absolutely vested remainder, and the child, by intestate succession, will inherit the boy's absolutely vested remainder. The remainder to the son's children was vested subject to open upon the birth of his first child. Because the son cannot have any more children after his death, all members of the class are ascertained at that time and the remainder becomes indefeasibly vested. Because the grant was to the son's "children" rather than "issue" or "descendants," there is no unborn child problem. (A) is wrong because the boy's vested remainder is inheritable by the child.

To combat rising insurance rates, a state formed a state-owned insurance company that operated exclusively within the state. The company provided insurance on the basis of premiums calculated according to a schedule of fees. Under the schedule, premiums for residents of a particular city were 25% higher than the premiums for any other municipality in the state. Forty percent of that city's residents were of Mexican descent compared with a state-wide Mexican-American population of approximately 15%. A Mexican-American citizen living in the city brings suit, alleging that the state insurance company's rate structure violates the Equal Protection Clause. Will the citizen's suit prevail? A) Yes, because the higher rates have the effect of discriminating against the Mexican-American population. B) Yes, unless the state insurance company shows a compelling reason for the discrimination. C) No, unless the citizen shows that Mexican-American citizens pay higher rates than similarly situated non-Mexican-American citizens of that city. D) No, because discriminatory economic regulations are not a suspect classification.

C The state insurance company will prevail unless the citizen can show that the company charges Mexican-American citizens higher rates than other citizens of that city who are similarly situated. The mere fact that legislation or governmental action has a discriminatory effect is not sufficient to trigger strict scrutiny. There must be intent to discriminate on the part of the government, which can be shown by the discriminatory application of a law or regulation that appears neutral on its face. If the state insurance company is charging the city's Mexican-American citizens higher rates than citizens who are otherwise situated the same, the court will find that there is an intent to discriminate in the rate-setting process, triggering strict scrutiny because a suspect class is involved.

A plaintiff brought an action in federal court against a state government, seeking monetary damages. The state moved to have the case dismissed for lack of jurisdiction, citing the Eleventh Amendment of the United States Constitution. Which of the following facts would support a denial of the state's motion? A) The plaintiff is a private citizen of the defendant state. B) The plaintiff is a Native American tribe. C) The plaintiff is a neighboring state. D)The plaintiff is a foreign government.

C The state's motion should be denied if the plaintiff is a neighboring state. The Eleventh Amendment does not bar actions by one state government against another state government.

A defendant was charged with the murder of a victim. During the course of the criminal trial, a witness testified on behalf of the defense that, at the time the murder took place, he saw someone who looked like the defendant dancing at a local nightclub. The defendant is eventually acquitted of the charge. Following the acquittal, the appropriate survivors of the victim bring a wrongful death action against the defendant. As part of her defense, the defendant wishes to introduce the testimony given at the criminal trial by the witness, who the defendant shows is now incarcerated in a prison in another state. Is the testimony of the witness admissible? A) Yes, because the witness testified under oath at another hearing related to the same subject matter. B) Yes, because the defendant is a party to both proceedings. C) No, because the plaintiffs were not parties to the criminal proceeding. D) No, because the witness can be subpoenaed to testify.

C The witness's testimony is inadmissible. Under Federal Rule 804(b)(1), the testimony of a witness who is unavailable, given at another hearing, is admissible in a subsequent trial if there is sufficient similarity of parties and issues so that the opportunity to develop testimony or cross-examination at the prior hearing was meaningful. The former testimony is admissible upon any trial of the same subject matter. The party against whom the testimony is offered or, in civil cases, the party's predecessor in interest must have been a party in the former action. "Predecessor in interest" includes one in a privity relationship with the party, such as grantor-grantee, testator-executor, life tenant-remainderman, and joint tenants. These requirements are intended to ensure that the party against whom the testimony is offered (or a predecessor in interest in a civil case) had an adequate opportunity and motive to cross-examine the witness. In the civil suit here at issue, the survivors of the victim were not parties to the criminal case, nor were they in privity with any such party. (The parties to that case were the defendant and the government.) These survivors, who are the plaintiffs in the instant litigation, are the parties against whom the testimony of the witness is being offered. Because they were not parties to the action in which the witness testified, they had no opportunity to cross-examine him. Even if the government had a similar motive to cross-examine the witness as do the plaintiffs in the current action, that is not sufficient to make the government a predecessor in interest to the plaintiffs. Consequently, the testimony of the witness does not come within the former testimony exception to the hearsay rule, and the testimony is inadmissible hearsay.

A buyer sent a signed letter to a seller that stated: "Ship 100 boxes of nails at $3 per box, the price quoted in your circular." The seller mailed the buyer a signed form acknowledgment that agreed to the buyer's terms and stated on the reverse side: "Disputes regarding quality shall be arbitrated." The buyer did not reply to the seller's acknowledgment, and the seller shipped the nails. When the buyer received the nails, it found their quality to be unsatisfactory and sued the seller for breach of warranty. The seller has asked an attorney whether the parties' contract requires arbitration of the buyer's claim. What is the best advice the attorney can provide? A contract was formed pursuant to conduct when the buyer received the nails, and a court would exclude the arbitration provision from the contract. A) contract was formed pursuant to conduct when the buyer received the nails, and a court would exclude the arbitration provision from the contract. B) A contract was formed when the seller mailed its acknowledgment, and the arbitration term became part of the contract. C) A contract was formed when the seller mailed its acknowledgment, and the court must decide whether the arbitration term should be excluded as a material alteration of the contract. D) No contract exists, because the arbitration term in the seller's acknowledgment created a counteroffer that the buyer never accepted.

C This is a contract for the sale of goods (nails) so Article 2 of the UCC applies. Under Article 2, the proposal of additional or different terms by the offeree in a definite and timely acceptance does not constitute a rejection and counteroffer, but rather is effective as an acceptance, unless the acceptance is expressly made conditional on assent to the additional or different terms. If both parties to the contract are merchants, the additional terms in the acceptance will be included in the contract unless they materially alter the original terms of the offer; the offer expressly limits acceptance to the terms of the offer; or the offeror has already objected to those terms, or objects within a reasonable time after notice of the terms is received. Whether an alteration is material is a fact question. Here, the seller's acknowledgment contained an additional term - but was not made conditional on its acceptance. Therefore, a contract was formed. The seller is clearly a merchant, and given the size of the order, it appears the buyer is a merchant too. Thus, the court must decide whether the arbitration term should be excluded as a material alteration of the contract. (A) is incorrect. The contract was formed when the seller mailed the buyer the signed acknowledgment form.

A defendant was convicted of fraud after a jury trial in state court. The conviction was affirmed on direct appeal. The defendant timely moved for postconviction relief under the Sixth Amendment on the ground that his attorney had provided ineffective assistance of counsel. The trial judge, after a hearing, found that the attorney had performed deficiently by failing to raise a proper objection that would have resulted in exclusion of important prosecution evidence. What more, if anything, must the trial court find in order to sustain the defendant's Sixth Amendment claim? A) Nothing more, because the unjustifiable failure to object to important prosecution evidence is structural error. B) That the attorney was court-appointed and not privately retained. C) That there is a reasonable probability that the trial's outcome would have been different if the attorney had objected. D) That there is clear and convincing evidence that the trial's outcome would have been different if the attorney had objected.

C To obtain postconviction relief for ineffective assistance of counsel, the claimant must show her counsel's performance was deficient and that, but for the deficiency, the outcome of the trial would have been different.

An employer offered to pay a terminated employee $50,000 to release all claims the employee might have against the employer. The employee orally accepted the offer. The employer then prepared an unsigned release agreement and sent it to the employee for him to sign. The employee carefully prepared, signed, and sent to the employer a substitute release agreement that was identical to the original except that it excluded from the release any age discrimination claims. The employer signed the substitute release without reading it. Shortly thereafter, the employee notified the employer that he intended to sue the employer for age discrimination. Is the employer likely to prevail in an action seeking reformation of the release to conform to the parties' oral agreement? A) No, because the employer acted unreasonably by failing to read the substitute release prior to signing it. B) No, because the parol evidence rule will preclude evidence of the oral agreement. C) Yes, because the employee's fraudulent behavior induced the employer's unilateral mistake. D) Yes, because the parties were mutually mistaken regarding the contents of the signed release.

C Typically, reformation is not available for unilateral mistake. However, if, as in this case, the writing is inaccurate because of a misrepresentation by the person who created the writing, the innocent party can seek reformation of the contract to reflect the original expressed intent of the parties.

A seller contracted to sell land to a buyer for $300,000.The contract provided that the closing would be 60 days after the contract was signed and that the seller would convey to the buyer a "marketable title" by a quitclaim deed at closing. The contract contained no other provisions regarding the title to be delivered to the buyer. A title search revealed that the land was subject to an unsatisfied $50,000 mortgage and a right-of-way easement over a portion of the land. The buyer now claims that the title is unmarketable and has refused to close. Is the buyer correct? A) No, because nothing under these facts renders title unmarketable. B) No, because the buyer agreed to accept a quitclaim deed. C) Yes, because the right-of-way easement makes the title unmarketable. D) Yes, because the unsatisfied mortgage makes the title unmarketable.

C An easement that reduces the value of the property, such as an easement of way for the benefit of a neighbor, renders title unmarketable. (D) is incorrect because, even though a mortgage can render title unmarketable, the seller has the right to satisfy the mortgage at closing with the proceeds of the sale. Thus, if the price is sufficient, the closing will result in marketable title.

A landowner devised her home "to my daughter for life, then to the eldest survivor of her two children, my grandson and granddaughter, for life, remainder to the eldest surviving offspring of the two grandchildren who is alive at the death of the last life tenant." After the landowner's death, the daughter lived in the family home for 15 years. Upon the daughter's death, both of her children were alive, so the home passed to the grandson, the eldest. He lived in the house for three years, and then conveyed it to the city historical society, which converted it into its headquarters and museum. Eight years later, the grandson died. At the time of his death, he was survived by his widow, his two sons, the granddaughter, and the granddaughter's daughter, who was the eldest of the niece and nephews. Four years after the grandson's death, the granddaughter's daughter brought an action for ejectment and to quiet title against the city historical society. The jurisdiction has a statutory period of adverse possession of 10 years, or five years if entry was made by the adverse possessor under color of title. How should the court rule in the granddaughter's daughter's action? A) For the society, because it has occupied the home for the statutory period required for adverse possession. B) For the society, because it purchased the home in fee simple absolute from the grandson. C) For the granddaughter's daughter, because the society has not been in adverse possession for the requisite period. D) For the granddaughter's daughter, because a purchaser of property from a life tenant cannot acquire a fee simple absolute through adverse possession.

C) The statute of limitations that determines the time period for adverse possession does not run against the holder of a future interest (e.g., a remainder) until that interest becomes possessory, because the holder of the future interest has no right to possession (and thus no cause of action against a wrongful possessor) until the prior present estate terminates. Here, the society has possessed the home for eight years; however, as against the granddaughter's daughter, the holder of the remainder, the statute did not begin to run until the death of the grandson. Prior to the termination of the grandson's life estate, the granddaughter's daughter had no cause of action against the society because she had no right to possession. Upon the grandson's death, when the granddaughter's daughter's interest became possessory, the statute began to run against her. Thus, as against her, the society has not been in adverse possession for the requisite period.

Property Clause,

Congress the power to "make all needful rules and regulations respecting the territory or other property belonging to the United States." This power permits Congress to acquire and dispose of all kinds of property, and to protect its property with a law such as the one here.

A backgammon player was upset after losing a match against the club champion. Rushing out of the club, he inadvertently grabbed the champion's board, which looked very much like his own but which was much more expensive. The player left the backgammon board in the trunk of his car, as was his usual practice. During the night, the car was stolen and along with it, the champion's expensive backgammon board. In an action by the champion against the player to recover the value of the backgammon board, What is the champion likely to recover? (conversion or trespass to chattel)

Conversion consists of (i) an act by defendant interfering with plaintiff's right of possession in the chattel, (ii) intent to perform the act bringing about the interference with plaintiff's right of possession, (iii) causation, and (iv) damages—an interference that is serious enough in nature or consequence to warrant that the defendant pay the full value of the chattel. Intent to trespass is not required; intent to do the act of interference with the chattel is sufficient for liability. Therefore, the player was guilty of conversion when he intentionally (i.e., volitionally) took the champion's board, which resulted in its loss, even though the player did not intend to lose it or even realize that he had taken the property of another.

A purchaser filed a federal diversity action against a seller, alleging breach of contract. The seller answered the complaint and included as a separate defense an allegation that the purchaser had brought and lost a similar contract claim against a different seller three years earlier, and that this history represented a pattern of filing frivolous lawsuits. The purchaser believes that the earlier lawsuit was factually completely different from the current one and is therefore irrelevant. What is the purchaser's best response to the seller's answer? A) File a reply that includes a denial of the separate defense. B) Move for sanctions against the seller for asserting a frivolous defense. C) Move to amend the complaint to add allegations about the differences between the lawsuits. D) Move to strike the separate defense as irrelevant.

D (D) is correct. Before responding to a pleading or, if no responsive pleading is permitted, within 21 days after service of the pleading, a party may move to have stricken any insufficient defense, or any redundant, immaterial, impertinent, or scandalous matter. Since the prior lawsuit is irrelevant to the current lawsuit, the purchaser should move to strike the defense. (A) is incorrect. A denial of the defense would be treating the defense as valid. It would leave the defense pending for trial, and evidence about the prior lawsuit in theory could be introduced. It is far better to have the defense stricken prior to trial, as it saves time at trial and prevents the jury from potentially hearing any facts about the prior lawsuit. (B) is incorrect. There is no indication that the seller knew the defense to be a sham, so it is unlikely that the court would impose any other sanction outside of striking the pleading. Absent any evidence of a Rule 11 violation, the matter may be best handled in a more direct manner by moving to strike the pleading. (C) is incorrect. If the lawsuit is irrelevant to the current proceedings, the purchaser should not add allegations about the differences between the two suits. Again, this strategy would be treating the defense as valid, which should not be the purchaser's strategy.

An employee filed in state court a civil action alleging sexual harassment in the workplace. She asserted federal statutory employment discrimination claims against her employer, and she asserted a state law battery claim against the co-worker who allegedly engaged in the sexual harassment. The plaintiff employee, the defendant employer, and the defendant co-worker are all citizens of the state in which the action was filed. May the defendants properly remove the action to federal district court? A) No, because the federal district court does not have subject matter jurisdiction over the action. B) No, because the defendants are citizens of the state in whose court the action is pending. C) Yes, the action may be properly removed to federal district court, provided the amount in controversy exceeds $75,000. D) Yes, the action may be properly removed to federal district court, regardless of the amount in controversy.

D A defendant may remove an action that could have originally been brought in the federal courts. Federal question jurisdiction is available when the plaintiff, in her well-pleaded complaint, alleges a claim that arises under federal law. In the instant case, the plaintiff is bringing a federal statutory employment discrimination claim. This presents a federal question, and removal is authorized on this basis. Once one claim satisfies the requirements for original federal subject matter jurisdiction, the court has discretion to exercise supplemental jurisdiction over related claims that derive from the same common nucleus of fact and are such that a plaintiff would ordinarily be expected to try them in a single judicial proceeding. The battery claim arises out of the same transaction or occurrence as the employment discrimination claim

A first-time home buyer financed their purchase with a mortgage loan from a bank. A title insurance policy obtained by the buyer as part of the transaction showed no issue with the title, so the closing proceeded without incident. However, there was a pre-existing mechanic's lien encumbering the property that the title examiner had overlooked. Fifteen years later the buyer paid off the mortgage. The next year, the buyer sold the property to a developer. As the developer was preparing its building plans, the holder of the mechanic's lien took action to enforce it against the developer. After settling the mechanic's lien, may the developer file a claim under the title insurance policy obtained by the buyer and obtain reimbursement? A) Yes, regardless of whether the title insurance policy is a lender policy or owner policy. B) Yes, so long as the title insurance policy is an owner policy. C) Yes, so long as the title insurance policy is a lender policy. D) No, regardless of whether the title insurance policy is a lender policy or owner policy.

D A title insurance policy insures that a good record title of the property exists as of the policy's date and agrees to defend the record title if litigated. Title insurance can be taken out by either the owner of the property or the mortgage lender. An owner's policy protects only the person who owns the policy (usually either the property owner or the mortgage lender) and does not run with the land to subsequent purchasers. In contrast, a lender's policy follows any assignment of the mortgage loan. However, the policy ends when the mortgage loan is paid off. Here, if the insurance policy obtained by the buyer was an owner policy, it would not run with the land to the developer as a subsequent purchaser, and if it was a lender policy, there is no longer any mortgage for the policy to follow (the buyer has paid it off), so that policy would have ended. Thus, the developer cannot recover in an action against the buyer's title insurance policy

A wealthy sportsman purchased a large old wooden sailing ship for $200,000. Although the boat was a classic, the sportsman wanted it to be modernized and made more comfortable. To that end, the sportsman entered into a written contract with a marine architect-engineer to draw up and then execute the modernization plans, for $7,500. At the time the parties entered into the agreement, the sportsman told the architect-engineer that his modernization plan would be subject to the approval of the sportsman's sister, that they would, in fact, have no deal unless the plans meet with her approval. The architect-engineer agreed to this. He finished his drawings and submitted them to the sportsman, who was enthusiastic about the designs. The sportsman's sister, a famous yachtswoman, was engaged in a trans-Pacific yacht race at the time and was not expected home for a number of weeks. Cheered by the sportsman's enthusiasm, the architect-engineer went ahead and modernized the ship according to his designs. When he finished the work, he submitted a bill to the sportsman, who refused to pay, pointing out that his sister had never approved the designs. If the architect-engineer sues the sportsman, which of the following issues of contract law is most likely to be decisive in determining the outcome of the case? A) Statute of Frauds B) Parol evidence rule. C) Rules of construction. D) Conditions precedent.

D Approval of the modernization plans by the sportsman's sister is a condition precedent because without such approval the parties have no agreement. Where there is an oral condition precedent, evidence of the condition falls outside the parol evidence rule. The parol evidence rule provides that where the parties to a contract express their agreement in a writing with the intent that it embody the final expression of their bargain, any other expressions, written or oral, made prior to the writing, as well as any oral expressions contemporaneous with the writing, are inadmissible to vary the terms of the writing. Certain forms of extrinsic evidence are deemed to fall outside the scope of the parol evidence rule. For instance, a party to a written contract can attack the validity of the agreement. One way of doing so is by asserting that there was an oral agreement that the written contract would not become effective until the occurrence of a condition. Such a condition would be deemed a condition precedent to the effectiveness of the agreement, and evidence of the condition will be freely offered and received. Here, the sportsman and the architect-engineer have entered into a written agreement that apparently embodies the full and final expression of their bargain. However, the sportsman's statement at the time of entering into the agreement indicates quite clearly that the parties had no agreement absent the approval of his sister, and the architect-engineer agreed with this statement. Thus, there is an oral agreement that the written contract would not become effective until the occurrence of a condition precedent. As discussed above, evidence of this oral condition does not come within the purview of the parol evidence rule and is therefore admissible. Consequently, the sportsman can assert the nonoccurrence of a condition precedent as a way to avoid liability on the contract. (B) is incorrect because, as explained above, the nature of the oral agreement takes it outside the scope of the parol evidence rule. Therefore, the rule will not be decisive in determining the outcome of this case.

At a waterfront bar, a college student sought to provoke a fight with a merchant seaman by making insulting remarks. Eventually the seaman had had enough and threw a punch that connected to the student's jaw and sent him sprawling to the floor. The seaman then told the student that he wanted no further trouble. Getting up off the floor, the student pulled a knife out of his pocket and charged at the seaman. Three other students were standing between the seaman and the exit door. The seaman tried to dodge, but was cut on the forearm by the student's knife. The seaman immediately drew a gun and shot the student, killing him. The seaman was charged with murder. Which of the following points raised in the seaman's defense will not be helpful for his defense? A) The student had no reason to fear serious bodily injury when he drew the knife.The student had no reason to fear serious bodily injury when he drew the knife. B) The student's drawing of the knife constituted an escalation of the fight.The student's drawing of the knife constituted an escalation of the fight. C) Three college students were standing between the seaman and the door, so there was no clear route of retreat.Three college students were standing between the seaman and the door, so there was no clear route of retreat. D) The student's comments were motivated by a desire to provoke the seaman.

D Even though the student's words may have been intended to provoke the seaman, this fact alone would not justify the seaman's use of deadly force. A person may use deadly force in self-defense if he: (i) is without fault; (ii) is confronted with unlawful force; and (iii) reasonably believes that he is threatened with imminent death or great bodily harm. Generally, one who is at fault for starting a confrontation has no right to use force in his own defense during that confrontation. However, if the victim of the initial aggression suddenly escalates a relatively minor fight into one involving deadly force and does not give the aggressor a chance to withdraw or retreat, the aggressor may use deadly force in his own defense. Here, although the student instigated the hostile situation by repeatedly insulting the seaman, the seaman's throwing of a punch probably calls for his being characterized as the aggressor. The student, as the victim of the initial aggression, escalated matters by using a knife, especially because the seaman had said that he wanted no further trouble.

Two neighbors owned summer homes adjacent to each other on the lake. After a week-long stay by the son of one of the property owners, the neighbor called the owner and said that his boat dock had been badly damaged and was told by another resident that the owner's son and some friends had gotten drunk and accidentally crashed their boat into his dock. The owner was surprised at the accusation because he was sure that if his son had caused the damages, he would have told him. However, he did not want to get into a dispute with his neighbor, so he told his neighbor that he would have the dock repaired and pay for the repairs if the neighbor agreed not to bring a claim against his son for the damage to the dock. The neighbor agreed, and the owner hired a local carpenter to do the work. Later, however, the owner discovered that his son did not damage the dock because the damages occurred after his son had returned to college. Is the owner obligated to pay for the repairs? ANo, because the owner never really believed that his son caused the damage. B) No, because his son in fact did not cause the damage. C) No, because the neighbor was wrong when he accused his son of causing the damage and it would be unfair to enforce an agreement when there was a mutual mistake of fact. D) Yes.

D Modern courts would hold that a promise to forbear suit on a claim that the promisor honestly and reasonably believes to be valid is good consideration to support an agreement, even if the claim ultimately turns out not to be valid.

A three-car accident occurred in a city in the Northern District of State A. The cars were driven by a citizen of State B who resides in the Southern District of State B, a citizen of State A who resides in the Southern District of State A, and a citizen of State C who resides in the Northern District of State C. The State B citizen filed a negligence action against the other two drivers in the United States District Court for the Southern District of State A. Although the two defendants believed that venue was improper, neither filed a pre-answer motion objecting to venue. They instead proceeded to file their answers, responding to the merits of the claim. The State C defendant, however, included in her answer a motion to dismiss the action for improper venue. How should the court rule on the State C defendant's motion to dismiss for improper venue? A) Deny the motion, because the defendants waived the defense of venue by not asserting it in a separate, pre-answer motion prior to filing their answers addressing the merits of the action. B) Deny the motion, because all defendants must object to venue before the court may dismiss on venue grounds. C) Deny the motion, because venue is proper. D) Grant the motion, because the motion is timely and venue is improper.

D Objections to venue may be raised in a defendant's answer if the defendant did not assert a Rule 12(b) pre-answer motion, as is the case here. Venue here is improper because (i) none of the events giving rise to the claim occurred in the Southern District of State A, and (ii) all defendants do not reside in State A.

A hot-air balloon touring company operated near a golf course. The company's property was separated from the golf course by a fence on which the company had posted signs warning people not to enter the property because of the dangers of balloons landing. A golfer on the golf course hit an errant shot onto the company's property, ignored the warning signs, and jumped over the fence to retrieve her golf ball. At about the same time, one of the company's balloons experienced mechanical problems and had to make an emergency landing to avoid crashing. The balloon, which was out of control when it landed, struck the golfer and injured her. The jurisdiction has decided that hot-air ballooning is an abnormally dangerous activity. In an action by the golfer against the company, does the company have any affirmative defenses? A) No, because the balloon was out of control when it struck the golfer. B) No, because the company was engaged in an abnormally dangerous activity. C) Yes, because the balloon landed to avoid crashing. D) Yes, because the golfer assumed the risk by coming onto the company's property.

D Plaintiff that knowingly or voluntary assumes the risk of injury caused by defendant conduct may be subject to the affirmative defense of assumption of the risk Assumption of the risk may be used in strict liability under some jurisdictions

A man went to his local sporting goods store and told the salesperson that he wanted a tennis racket that was very high-end. The salesperson showed him a racket that he said was made of the finest titanium and would probably last for years. The man bought the racket and left the store. After playing with the racket for three days, it suddenly snapped in two when he hit a hard shot. He showed it to the tennis pro at his club, who informed him that the racket was painted plastic. If the man sues the sporting goods store, which of the following best describes the legal basis for his suit? A) The store breached the implied warranty of fitness for a particular purpose and an express warranty that the racket was made of titanium. B) The store breached the implied warranty of merchantability and an express warranty that the racket would last for years. C) The store breached both the implied warranty of fitness for a particular purpose and the implied warranty of merchantability. D) The store breached the implied warranty of merchantability and an express warranty that the racket was made of titanium.

D The best argument is that the store breached both the implied warranty of merchantability and an express warranty that the racket was made of titanium. An express warranty will arise from any statement of fact or promise. Here, the salesperson said that the racket was made of titanium. This is a statement of fact that will give rise to a warranty. An implied warranty of merchantability will arise in every sale by a merchant unless disclaimed. To be merchantable, goods must be fit for ordinary purposes, and arguably a racket that breaks right away because it is made of plastic is not fit for ordinary purposes.

A defendant was on trial for burglary, and he took the stand in his own defense. On direct examination, the defendant vigorously denied having committed the burglary. Also on direct examination, the defendant stated that his last regular employment was as a bookkeeper for a corporation. On cross-examination, the prosecutor asked the defendant if he had embezzled funds from the corporation. The defendant denied that he had embezzled from the corporation or from anyone else. The prosecutor then wanted to call a police officer to the stand to testify that when she arrested the defendant for embezzlement, the defendant admitted to the officer that he had embezzled money from the corporation. Assuming that the defendant has not yet been tried on the embezzlement charges, may the prosecutor call the officer to the stand? A) Yes, but only for purposes of impeachment. B) Yes, both for impeachment of the defendant and as substantive evidence. C) No, because the defendant has not yet been convicted of embezzlement. D) No, because the evidence would be extrinsic.

D The officer may not testify about the embezzlement because it constitutes impeachment by extrinsic evidence of a specific instance of misconduct. A specific act of misconduct offered to attack the witness's character for truthfulness can be elicited only on cross-examination. If the witness denies the act, the cross-examiner cannot refute the answer by calling other witnesses or producing other evidence. Because the alleged embezzlement is admissible, if at all, only as impeachment evidence, when the defendant denied it the prosecutor could not call the officer to testify. (A) is wrong because extrinsic evidence, such as the officer's testimony, of an instance of misconduct is not admissible.

A township located in a farming community was composed mostly of persons belonging to a specific religious sect. To help instill proper respect for authority in children, which was a central tenet of the sect, and to maintain order in the classroom, the local school board allowed teachers to inflict corporal punishment. Such punishment was inflicted on a fourth grader in a township school immediately after his teacher saw him pulling a girl's hair. Neither he nor his parents belonged to the religious sect. When the boy's parents learned of the incident, they hired an attorney. Rather than suing the teacher for battery as permitted under state law, the attorney brought an action against the teacher under a federal statute providing a cause of action for damages against any government employee who deprives a person of his constitutional rights. Should the court find the policy allowing corporal punishment to be constitutional? A) No, because the punishment policy violates the First Amendment Establishment Clause. B) No, because the boy was denied any kind of hearing, in violation of his right to procedural due process under the Fourteenth Amendment. C) Yes, because under the doctrine of parens patriae states may impose any punishment they see fit. D) Yes, because the punishment was not grossly disproportionate under the Eighth and Fourteenth Amendments.

D The punishment here is constitutional because it does not violate any constitutional provision. The best answer reflecting this reasoning is (D)-there was no Eighth Amendment violation here-because paddling students as a disciplinary measure has not been found to be cruel and unusual punishment. (B) is incorrect because there has been no deprivation of procedural due process. The Supreme Court has held that although corporal punishment may involve a liberty interest, no hearing is required prior to inflicting such punishment; the possibility of a common law action in tort is sufficient procedural protection. [

A defendant was tearing up a stretch of pavement with a jackhammer when a rock flew up and struck a plaintiff in the head, causing him to be hospitalized. Because the jackhammer manufacturer had been out of business for several years, the plaintiff filed a lawsuit for his medical costs, lost work time, and pain and suffering solely against the defendant. At trial, the plaintiff's attorney calls a witness who testifies that, at the time of the incident, the defendant stated, "It was my fault." The defense attorney objects, but the judge overrules the objection on the ground that this is a declaration against interest. Are the grounds for the judge's decision correct? A) Yes, because the statement subjected the defendant to tort liability. B) Yes, because the defendant is a party to the litigation. C) No, because the statement is not against an important interest. D) No, because the defendant is available to testify.

D The statement against interest exception to the hearsay rule requires that the declarant be unavailable as a witness. A declarant is unavailable if: (i) she is exempted from testifying because the court rules that a privilege applies, (ii) she refuses to testify concerning the statement despite a court order to do so, (iii) she testifies to not remembering the subject matter of the statement, (iv) she cannot testify because she has died or is ill, or (v) she is absent and the statement's proponent is unable to procure her attendance or testimony by process or other reasonable means. [Fed. R. Evid. 804(a)(1) - (5)] None of the bases for a finding of unavailability is present here. The defendant, the declarant whose statement is at issue, is available as a witness; thus, the judge was incorrect in basing his decision on this exception.

Congress enacted a statute requiring state-supported institutions of higher education that provide federal student loan funds to their students to fund women's sports according to a complex formula intended to fairly support women's athletics and remedy past funding discrimination. Under the formula, a particular state military school will be required to allocate 25% of its athletic budget to its female athletics programs even though only 10% of the school population is female. A male student whose athletic program will be discontinued because of the budget allocation filed suit in federal court challenging the federal statute on various constitutional grounds. Is the court likely to find that the statute is constitutional? A) No, because the government will be unable to prove that the discriminatory funding requirements of the statute are necessary to achieve a compelling government interest. B) No, because the federal government does not have the power to dictate the budget allocations of state-supported educational institutions. C) Yes, because remedying past discrimination is a legitimate government interest, and the student will be unable to prove that the statute's funding requirements are not rationally related to that interest. D) Yes, because the government will be able to prove that the statute's funding requirements are substantially related to an important government interest.

D The statute will likely be held constitutional because the attempt to compensate for past discrimination against women is substantially related to an important government objective. When examining federal government action involving classifications of persons, the Supreme Court, using the Due Process Clause of the Fifth Amendment, applies the same standards that it applies to state actions under the Fourteenth Amendment Equal Protection Clause. When analyzing government action based on gender classifications, the Court will apply an intermediate standard and strike the action unless the government proves, by an exceedingly persuasive justification, that the action is substantially related to an important government interest. Applying this standard, the Court has generally upheld classifications benefiting women that are designed to remedy past discrimination against women, because remedying past gender discrimination is an important government interest. Here, the federal statute establishes a formula designed not only to ensure current "gender equity" in funding of intercollegiate athletic programs but also to correct specific past inequities, and the school's required funding allocation in favor of women is designed to correct inequitable allocations by the school in prior years. Hence, even though the statute's allocation requirement may discriminate against the student and other males at the school, the government can satisfy its burden of showing a substantial relationship to an important government interest.

The driver of a tractor-trailer lost control of his vehicle after driving onto an icy bridge in excess of the speed limit and slid off the road. A state trooper responding to the accident got out of his squad car and walked toward the vehicle on the roadway. Before he could set up warning flares or safety cones, a delivery van approached the bridge in the oncoming lane. The van slid on the ice and spun out of control, striking and injuring the trooper. The trooper brought an action against the driver of the delivery van and the driver of the tractor-trailer. Will the trooper prevail as against the driver of the tractor-trailer? A) Yes, because the trooper's injuries were a foreseeable result under the circumstances. B) Yes, because the "firefighters' rule" does not apply in this case. C) No, because the delivery van striking the trooper was the actual cause of his injuries. D) No, because the trooper was injured during the course of his job.

D The trooper will not prevail. As a general rule, a defendant is liable if he negligently puts himself in peril and the plaintiff is injured in attempting a rescue. However, the "firefighters' rule" will bar firefighters and police officers, on public policy or assumption of risk grounds, from recovering for injuries caused by the risks of a rescue. Here, it was an inherent and foreseeable risk of the job that the trooper might be struck by another vehicle at an accident site. Hence, the trooper probably will not be able to recover,

Question A law student was sued by a student loan provider in federal court for failing to pay back her student loans. At the close of a federal civil trial, the court held in favor of the student loan provider for the full amount plus post-verdict interest at 8%. When the court clerk typed up the final order to send to all parties, she accidentally typed in 9% as the post-verdict interest rate. The law student did not notice the error until 14 months after the judgment was entered. She immediately filed a motion for relief from judgment based on a clerical mistake to correct the order to reflect the accurate interest rate. Will the court likely grant this motion? A) No, because motions for relief from judgment to correct clerical mistakes must be made within a reasonable time, not to exceed one year. B) No, because motions for relief from judgment to correct clerical mistakes must be made within 28 days of the order. C) Yes, because motions for relief from judgment to correct clerical mistakes must be made within a reasonable time. D) Yes, because motions for relief from judgment to correct clerical mistakes can be made at any time.

D There is no time limit to file a motion for relief from judgment to correct clerical mistakes, and the court order correcting the error dates back to the time judgment was entered. [Fed. R. Civ. P. 60(a)] A clerical error is one arising from oversight or omission and may occur in judgments, orders, or other parts of the record. Since there is no time limit, the only issue is whether the error constitutes a "clerical mistake."

A plaintiff filed an action in federal district court and served the defendant with the summons and complaint. The defendant moved to dismiss the complaint for failure to state a claim. Instead of opposing the motion to dismiss, the plaintiff voluntarily dismissed the action and filed a new action, alleging the same claims but also addressing the pleading defects outlined in the defendant's motion to dismiss. The defendant then moved to dismiss the second action, and the plaintiff again voluntarily dismissed the second action instead of filing opposition papers. The plaintiff then filed a third action, alleging the same claims but also including additional allegations that were responsive to the defendant's second motion. The defendant has moved to dismiss the third action; the plaintiff opposes the motion. Is the court likely to grant the defendant's motion? A) No, because the plaintiff has promptly and diligently attempted to address the pleading defects. B) No, because the plaintiff voluntarily dismissed each previous action before the defendant filed an answer or moved for summary judgment. C) Yes, because the plaintiff failed to seek a court order dismissing the second action. D) Yes, because the plaintiff's previously dismissed actions asserting the same claims operate as an adjudication on the merits.

D Two dismissal Rule: a dismissal by notice is without prejudice unless plaintiff previously dismissed action of same claim in which the case is dismissed with prejudice

The defendant was fired from his sales job while calling on customers in another city. He failed to return the company car that he was using for his sales visits; instead, he sold the car to a "chop shop" for cash. As to the car, what crime has the defendant committed? Press Enter or Space to submit the answer

Embezzlement Embezzlement is the fraudulent conversion of the property of another by a person in lawful possession of it. In the instant case, the car belonged to the company for which the defendant worked, but the company probably gave the defendant lawful possession of it so that he could make sales calls. When he sold the car rather than return it, he wrongfully converted the car. This is embezzlement. (

A plaintiff sued an industrial facility in her neighborhood for injuries to her health caused by air pollution. At trial, the plaintiff was asked questions on direct examination about the days on which she had observed large amounts of dust in the air and how long the condition had lasted. She testified that she could not remember the specific times, but that she maintained a diary in which she had accurately recorded this information on a daily basis. When her attorney sought to refresh her recollection with her diary, she still could not remember. The plaintiff's attorney seeks to have the information in the diary admitted at trial. Is the information admissible? A) No, because reviewing it did not refresh the plaintiff's recollection. B) No, unless it is offered by the defendant. C) Yes, and the plaintiff should be allowed the option of reading it into evidence or having the diary received as an exhibit. D) Yes, and the plaintiff should be allowed to read the diary into evidence.

D Where a witness states that she has insufficient recollection of an event to enable her to testify fully and accurately, even after she has consulted a memorandum or other record given to her on the stand, the record itself may be read into evidence if a proper foundation is laid for its admissibility. This is known as the recorded recollection exception to the hearsay rule. Here, all the foundational requirements are met-the plaintiff had personal knowledge of the facts when she made the diary; she made the diary herself and in a timely manner each day; she has vouched for its accuracy; and she currently has insufficient recollection to testify fully and accurately about the facts contained in the diary. Thus, the information in the diary may be read into evidence.

A state law provides that all persons who have been residents of the state for more than three years shall be entitled to free tuition at the state's main university. It further provides that persons who have resided in the state for three years or less shall pay the nonresident tuition rate, which is significantly higher. A student at the state's university who had been a state resident for less than three years filed a class action in federal court on behalf of himself and other similarly situated university students, seeking a declaration that the state statute is unconstitutional. When the case came to trial, the student had been a resident of the state for more than three years and was no longer required to pay tuition. By that time, a number of amicus curiae briefs had been filed in the case, some supporting and some opposing the student's position. Nevertheless, the state moved to dismiss the case as moot. Should the state's motion to dismiss be granted? A) Yes, because the student is now a three-year resident. B) Yes, because the student lacks standing. C) No, because amicus curiae briefs have been filed. D) No, because there is a live controversy.

D a class action is not moot, and the class representative may continue to pursue it-even if the representative's own controversy has become moot-because the claims of others in the class are still viable. Here, the student filed his suit as a class action for university students with less than three years' residency; undoubtedly some of those students will still have a real controversy at this time. Thus, the case is not moot. (A) is wrong although it states a true fact.

A husband and wife acquired land as common law joint tenants with right of survivorship. One year later, without his wife's knowledge, the husband executed a will devising the land to his best friend. The husband subsequently died. Is the wife now the sole owner of the land? A) No, because a joint tenant has the unilateral right to end a joint tenancy without the consent of the other joint tenant. B) No, because the wife's interest in the husband's undivided 50% ownership in the land adeemed. C) Yes, because of the doctrine of after-acquired title, or estoppel by deed. D) Yes, because the devise to the friend did not sever the joint tenancy.

D Although as a general rule a joint tenant's interest is freely alienable during his or her lifetime without the consent of the other joint tenant, that interest cannot be devised in a will. In this case, on the death of the husband, the wife's interest in the joint tenancy immediately expanded and she became the sole owner of the land as the surviving joint tenant.

A contractor agreed to remodel a homeowner's garage for $5,000. Just before the parties signed the one-page written contract, the homeowner called to the contractor's attention the fact that the contract did not specify a time of completion. The parties orally agreed but did not specify in the contract that the contractor would complete the work in 60 days, and then they both signed the contract. The contract did not contain a merger clause. The contractor failed to finish the work in 60 days. The homeowner has sued the contractor for breach of contract. Is the court likely to admit evidence concerning the parties' oral agreement that the work would be completed in 60 days? A) No, because the court must ascertain the meaning of the agreement from the terms of the written contract. B) No, because the oral agreement was merely part of the parties' negotiations. C) Yes, because the contract is ambiguous. D) Yes, because the time limit is an additional term that does not contradict the partially integrated written contract.

D) is correct. The time limit is an additional term that does not contradict the partially integrated written contract, and evidence concerning it will be admissible. When the parties to a contract express their agreement in a writing with the intent that it embody the full and final expression of their bargain, the writing is an "integration," and under the parol evidence rule, admissibility of evidence seeking to vary its terms is limited. If the writing is only a partial integration, and not a complete embodiment of the parties' intentions, under the parol evidence rule, it cannot be contradicted, but it may be supplemented by proving up consistent additional terms. Here, the writing likely was only a partial integration - it was only one page, it did not include a merger clause (inclusion is strong evidence the parties intended the writing to be a full and final integration), and it did not specify a time of completion. The parties did not intentionally leave out the time of completion, it just wasn't brought to their attention until just before signing. At that point, the parties orally agreed to that supplementary term. Therefore, parol evidence would be allowed to supplement the terms of the partial integration.

A state bans the use of disposable diapers to reduce the volume of nonbiodegradable material in its landfills. The ban was a boon for diaper services within the state, but many parents of young children were displeased with the use of conventional diapers. With support from retail establishments that lost business from the disposable diaper ban, a grass roots coalition formed to fight the ban funded a study showing that the trucks and cleaning supplies used by diaper services within the state harmed the environment more than disposable diapers. The coalition and retailers then filed suit seeking to have the ban on disposable diapers declared unconstitutional. If the court strikes down the statute, on which of the following constitutional provisions would its decision most likely be based?

Due Process Clause- Sub. DP tests the reasonableness of a statute: it prohibits arbitrary gov't action. Under substantive dp, gov't action limits a fundamental right the government must prove the action is necessary to promote a compelling interest. If not, the challenging party must prove the law is not related to any rationally related to any legitimate government interest

Contract Defense- Unilateral Mistake

General rule is that a contract will be not be avoided by unilateral mistake Exception- non-mistaken party either knew or should have known of the mistake. If the parties reform the contract due to the mistake but do not reduce it to writing the court will still enforce due to the mistake

The defendant is on trial for murder. During the trial, the prosecution offers into evidence a properly authenticated affidavit summarizing the results of the defendant's fingerprint test, as proof that the fingerprints on the murder weapon are those of the defendant. The affidavit was prepared pursuant to statute by the lab technician that conducted the test. The defendant objects to the evidence. How should the court rule?

Inadmissible because the defendant rights under the confrontation clause were violated A criminal defendant has the constitutional right, under the Confrontation Clause, to confront and cross-examine the witnesses against him. Affidavits that summarize the findings of forensic analysis and have the effect of accusing the defendant of criminal conduct are testimonial in nature and are not admissible into evidence against the defendant unless the preparer is unavailable and the defendant previously had an opportunity to cross-examine. Here, admission of the affidavit precludes the defendant from cross-examining the lab technician, and there is no evidence suggesting that the technician is unavailable. Thus, the affidavit is inadmissible-the lab technician must be brought in to testify as to the findings of the fingerprint test.

A beneficiary has filed a petition in the probate court to contest the validity of a testator's will. The beneficiary contends that when the testator executed the will eight years before, he had a severe mental illness and was incapable of forming a valid testamentary intent. In support of this contention, the beneficiary seeks to offer an affidavit prepared by the testator's former attorney, which states that she was asked to prepare a will for the testator just four months before this will was made. The attorney had refused to do so because it was her opinion that the testator seemed incoherent and paranoid. How should the judge rule on the admissibility of this affidavit?

Inadmissible, because it is hearsay not within any exceptions

A husband and a wife were arrested by federal agents and charged with distributing obscene materials through the United States mails. When called before a grand jury, the wife refused to say anything, invoking her Fifth Amendment right to be protected from compelled self-incrimination. The husband was terrified of the grand jury and readily admitted under questioning that he sent obscene matter through the mail. He also incriminated his wife in the illegal activity. The thought of a trial and a prison term drove the husband over the edge, and he committed suicide two days before his trial was to begin. A month later, the wife was put on trial in federal district court. The federal prosecutor seeks to introduce a transcript of the husband's grand jury testimony into evidence against the wife. The defense attorney objects. How should the court rule on the admissibility of the grand jury transcript?

Inadmissible, because the husband's testimony was not subject to cross-examination The husband's testimony was hearsay because it was an out-of-court statement offered to prove the truth of the matter asserted. [Fed. R. Evid. 801(c)] If a statement is hearsay, and no exception to the rule is applicable, the evidence is inadmissible. [Fed. R. Evid. 802] Under the former testimony exception to the hearsay rule, the testimony of a now unavailable witness given at another hearing is admissible in a subsequent trial as long as there is a sufficient similarity of parties and issues so that the opportunity to develop testimony or cross-examine at the prior hearing was meaningful. [Fed. R. Evid. 804(b)(1)] The party against whom the former testimony is offered must have had the opportunity to develop the testimony at the prior proceeding by direct, cross-, or redirect examination of the declarant. Thus, the grand jury testimony of an unavailable declarant is not admissible as former testimony against the accused at trial. This is because grand jury proceedings do not provide the opportunity for cross-examination.

A plaintiff's attorney is taking the deposition of a witness. Although within the scope of the discovery plan submitted by the parties' attorneys to the court, the witness's attorney objected to some of the questions on the grounds that they are unduly invasive and improper. The plaintiff's attorney insisted that the witness must answer the questions. What should the witness's attorney do next?

Instruct the client not to answer the questions and move for a protective order in the court in the district where the deposition will be taken If the motion for the protective order is granted, the court may require the party opposing the motion to pay the moving party's reasonable expenses, including attorney fees. Discovery plans do not preclude deponents from seeking protective order

A construction company that was putting in a swimming pool for a homeowner left a couple of large pieces of equipment in the backyard overnight. The equipment was not owned by the construction company but was leased from an equipment company, which was responsible for its repair and maintenance. After the workers had left, a seven-year-old boy came onto the homeowner's property to play. The homeowner was aware that the boy often came onto his property to play with his dog. The boy climbed up on one of the pieces of equipment and began pushing buttons and moving levers. The engine started and the equipment began to move because the equipment company had not replaced a defective safety locking device on the ignition. The boy became frightened and jumped off, falling into the hole that had been dug that day, and was injured. The boy's parents brought suit against the homeowner and the construction company. If the construction company is held liable for the boy's injuries, may it recover anything from other tortfeasors?

It may obtain contribution from the equipment company because the equipment was negligently maintained in an unsafe condition. because the equipment company, which was responsible for repair and maintenance of the equipment, negligently performed such maintenance, resulting in the absence of a working safety locking device, then the equipment company's negligence would have combined with that of the construction company to proximately cause the boy's injuries. This would render the companies jointly and severally liable to the boy for the entire damage incurred. Thus, if the construction company is held liable for the injuries, it has a claim against the equipment company, as a jointly liable party, for the amount it pays in excess of its share of damages. Note that, in the usual case, the equipment company would have been included in the lawsuit. However, the fact that it was not included does not preclude the construction company from recovering contribution in a separate action

In lieu of a snack vending machine, a small company put candy bars into a box in the office kitchen. Using the honor system, employees were asked to put a dollar into an envelope in the box if they took out a candy bar. The office manager would periodically take the money collected in the envelope and use it to purchase new candy. One night while cleaning the kitchen, the company's night janitor impulsively took all the money from the envelope. His actions were recorded on the company's newly installed security camera. The janitor later admitted that he took the money to fund illegal dogfights. Which of the following crimes did the janitor commit?

Larceny, because he did not intend to purchase replacement candy which consists of the taking and carrying away of the tangible personal property of another by trespass, with intent to permanently deprive the person of his interest in the property

The owner of an apartment building contracted with a painter to paint the porches of the apartments for $5,000. The contract was specifically made subject to the owner's good faith approval of the work. The painter finished painting the porches. The owner inspected the porches and believed in good faith that the painter had done a bad job. The painter demanded payment, but the owner told him that the paint job was poor and refused to pay. The painter pleaded that he was desperately in need of money. The owner told the painter that she would pay him $4,500, provided he repainted the porches. The painter reluctantly agreed, and the owner gave the painter a check in the amount of $4,500. The painter went to his bank, indorsed the check "under protest" and signed his name, then deposited the check in his account. He never returned to repaint the porches. The painter sues the owner for $500, which he believes is still owed to him on his contract to paint the porches. Will he prevail if he repaints the porch?

NO! he did not satisfy the condition precedent to payment under the contract. A party does not have a duty to perform if a condition precedent to that performance has not been met. Here, the parties made the owner's satisfaction with the painter's paint job a condition precedent to the owner's duty to pay the $5,000. Because the owner was not satisfied with the paint job, her duty to pay the painter never arose. The fact that the owner offered to give the painter $4,500 if he repainted the porches has no effect on this analysis, because the offer constituted a new contract, the owner having been excused from the old one.

When a co-defendant is killed during the commission of the crime, can the defendant be charged with 1st degree murder of the co-defendant?

NO! the death would not be murder but would be a justifiable homicide

A driver was driving north on a local road when his car went out of control, crossed the center line, and struck the vehicle of another driver who was driving south on the same road. Immediately after the accident, an off-duty officer came by and photographed the accident scene for the police report. In a suit between the drivers, the plaintiff seeks to introduce the photograph taken by the officer. The officer is present in court but has not been called as a witness. Is the photograph of the scene of the accident admissible?

No because a proper foundation has not been laid To be admissible, a photograph must be identified by a witness as a portrayal of certain facts relevant to the issue, and verified by the witness as a correct representation of those facts. It is sufficient if the identifying witness is familiar with the scene or object that is depicted. Here, the photograph taken by the officer must be verified by a witness who is familiar with the accident scene as an accurate representation of that scene. Absent such verification and identification (i.e., a proper foundation), the photograph is not admissible.

An antique lover spotted a beautiful Early American bedroom ensemble at her favorite antique store. The ensemble included a bed, a mirror, and two dressers. Over a period of several weeks, the shop owner and the antique lover negotiated over a price, but they were unable to come to an agreement. On April 3, the shop owner and the antique lover signed a statement whereby the shop owner offered to sell to the antique lover an Early American bedroom ensemble, recorded as items 20465, 20466, 20467, and 20468 in the shop's registry, if the parties agree upon a price on or before April 12. On April 6, the shop owner sent a letter to the antique lover, telling her that she could have the bedroom ensemble for $22,000. Also on April 6, the antique lover sent a letter to the shop owner telling him that she was willing to pay him $22,000 for the bedroom ensemble. Both parties received their letters on April 7. Without assuming any additional facts, was there a valid contract?

No because there was no mutual assent. Neither party accepted Identical deals crossing at the same time in the mail do not give rise to the contract despite the apparent meeting of the minds

How many days does a plaintiff get to amend a complaint without court intervention?

No later than 21 days after answer

A defendant is on trial for murder. The only evidence linking the defendant to the crime is some blood found at the scene. The lead detective testifies that an officer took a vial containing a blood sample that had been retrieved by a crime scene technician and drove off with it. The officer is now dead. Next, the prosecution presents as a witness a crime lab chemist. The chemist will testify that he took a vial of blood that contained a label identifying it as having been retrieved from the subject crime scene, and that he performed tests that established a match between that blood and a blood sample taken from the defendant. Is the testimony of the chemist admissible?

No, Insufficient Evidence for the Chain of Custody One of the general requirements for admissibility of real evidence is that it be authenticated; i.e., that it be identified as being what its proponent claims it is. If the evidence is of a type that is likely to be confused or can be easily tampered with, the proponent of the object must present evidence of chain of custody. The proponent must show that the object has been held in a substantially unbroken chain of possession. It is not necessary to negate all possibilities of substitution or tampering; rather, what is required is to show adherence to some system of identification and custody. Here, the proponent of the blood sample (the prosecution) has not shown what the officer did with it after leaving the crime scene. There is no showing that the vial was placed directly in a properly secured area so as to diminish the possibility of tampering. In short, it has not been demonstrated that there was adherence to some defined system of identification and custody.

A company operated a small amusement park on property it owned near a residential neighborhood. On a day when the park was closed, a 10-year-old girl snuck into the park with some friends by climbing over a chain link fence. While climbing on one of the carnival rides, the girl slipped and cut her leg on an exposed gear assembly, sustaining serious injuries. Through her guardian ad litem, the girl brought suit against the company to recover damages for her injuries. At trial, she presented evidence of the accident and her injuries. In defense, the company established that the girl read and understood the "No Trespassing" signs that were attached to the fence. The company also established that it had not had any previous reports of children sneaking into the park when it was closed. Before submission of the case to the jury, the company moved for summary judgment. Is the court likely to grant the company's motion?

No, because a jury could find that the company should have foreseen that children would sneak into the park The jury decides if attractive nuisance would apply

A homeowner returned home from work one day to find a robber in her living room. After a brief physical altercation, the homeowner ran to a bedroom, hid in a closet, and called 911 on her cell phone. Police officers arrived in less than two minutes and were able to apprehend the robber as he tried to run out the front door. Once they made sure he was locked in the police car, one of the officers went to speak with the homeowner about what had happened. She was still crying and shaking when the officer found her, and she said, "Thank you for catching him! He punched me in the head as I was running away!" The robber was charged with robbery and assault. Traumatized, the homeowner left the country and cannot be traced, despite the efforts of the prosecutor. The prosecutor intends to call the officer to testify as to the homeowner's statement. Should the court allow the officer's testimony?

No, because admitting the homeowner's statement would violate the defendant's constitutional rights Under the Confrontation Clause, an accused has the right to be confronted by the witnesses against him. A hearsay statement will not be admitted-even if it falls within a hearsay exception-when: (i) the statement is offered against the accused in a criminal case; (ii) the declarant is unavailable; (iii) the statement was testimonial in nature; and (iv) the accused had no opportunity to cross-examine the declarant's "testimonial" statement prior to trial. When the primary purpose of the interrogation is to establish or prove past events potentially relevant to a later criminal prosecution, statements are testimonial. Here, the homeowner's statement was hearsay because it was made out of court and is being offered for its truth-that the robber punched her in the head. It appears that the emergency had already resolved by the time the statement was made. Although only a few minutes had passed since the physical altercation and the homeowner was still upset, the robber no longer posed any danger because he was locked in the police car and the homeowner was aware of this ("Thank you for catching him!"). Therefore, the homeowner's statements to the officer were testimonial. Because the homeowner is unavailable to testify at trial and the robber has had no opportunity to cross-examine the statements, admitting them at trial through the testimony of the officer would violate the Confrontation Clause. (

A pedestrian was injured in an auto accident caused by a driver. The pedestrian's injuries included a broken nose and a broken toe. Not sure of the strength of her case, she sued the driver only for the injuries to her nose. She was awarded $15,000 in damages. Encouraged by this success, she now wishes to sue the driver for the injuries to her toe. May she sue the driver again?

No, because all related claims "merged" with the final decision in the 1st case

A witness is called in a contract action between a plaintiff and a defendant. The witness takes his oath and testifies. During cross-examination, the defendant's attorney asked the witness this question: "Isn't it true that even though you took an oath to tell the truth so help you God, you are an atheist and don't even believe in God?" Upon the proper objection, will the judge require that the witness answer this question?

No, because evidence of the beliefs or opinions of a witness on matter of religion is not admissible to impair credibility Lack of religious belief is no longer a basis for excluding a witness, and are irrelevant in determining the competency of the witness

Three drivers were in an automobile accident in a city in State A. The drivers were citizens of State A, State B, and State C. The State B driver filed a tort action against the other two in a State A state court, seeking $300,000 for her severe injuries. The State C driver wants to remove the action to a federal district court. Is the action removable?

No, because one defendant is a citizen of State A case may not be removed on the basis of diversity jurisdiction if a defendant is a citizen of the state in which the action was filed.

A man from a foreign country obtained a doctorate in political science from a state university and applied to teach there. The man was denied employment at the university under a state law requiring all teachers within the state to be United States citizens. Is the state's citizenship requirement constitutional as it applies to the man?

No, because the citizenship requirement is not necessary to achieve a compelling state interest. The Supreme Court has upheld state statutes prohibiting aliens from teaching primary or secondary school on the rationale that teachers at the elementary and high school level have a great deal of influence over the attitudes of young students toward government, the political process, and citizenship. It is doubtful that the Court would extend this rationale to university teachers

A foreign student who had entered the United States on a student visa four years ago was notified by federal immigration authorities that he was subject to being deported because his visa had expired. Federal law provided that an alien who is subject to being deported has the right to appear before an administrative officer appointed by the Attorney General's office for a hearing on whether he should be deported. This officer, appointed by the executive branch of the government, has the right under law to make a final order concerning whether the alien should be deported. After a hearing, the administrative officer entered an order allowing the student to remain in the United States as a permanent resident. However, a congressional rule permitted the House of Representatives, by resolution, to deport "undesirable aliens." After the administrative judge entered his order, the House passed a resolution that the student should be deported. The student petitioned the federal court to declare the legislative resolution invalid. Should the court find the resolution to be valid?

No, because the federal law removed congressional power with regard to aliens in this circumstance, and the resolution of the House violates the separation of powers doctrine. separation of powers doctrine forbids Congress from trying to control the exercise of the power delegated in various ways, such as by overturning an executive agency action without bicameralism (i.e., passage by both houses of Congress). By enacting the federal law allowing the administrative law judge to enter a final order with regard to aliens, Congress has given up any control it may have had previously in these situations. The resolution by the House here is an unconstitutional legislative veto that violates the separation of powers doctrine

A motorist driving home one night on a desolate two-lane road stopped when he saw a person lying on the road next to a bicycle. The cyclist had slipped and fallen off his bicycle, and was knocked unconscious when he hit his head on the pavement. Not wishing to get involved and seeing that no one else was around, the motorist got back into his car and drove away without making any effort to help the cyclist, even though he had a cell phone with which he could have summoned aid. The cyclist remained lying in the same place and was later struck by another car. If the cyclist brings suit against the motorist for injuries suffered when he was struck by the other car, will the cyclist prevail?

No, because the motorist was not responsible for causing the cyclist to be lying by the side of the road There is no general affirmative duty to rescue!! If one decides to assist then he must act as a reasonable person in that situation however the motorist getting out of the car did nothing to worsen the situation such as one not stopping because they assumed the situation was handled

A defendant visited her doctor to seek treatment for a bullet wound. While he was treating the wound, the doctor asked the defendant how she was shot. The defendant replied that she was struck by a police officer's bullet while running away from a jewelry store she had robbed, but she implored the doctor not to tell this to anyone. The doctor promised that he would not. Although the defendant was never charged by the police, the owner of the jewelry store brought suit against her seeking the value of the stolen goods. The defendant denied robbing the store. At the trial, the owner calls the doctor to testify to the statement made to him by the defendant. The defense attorney objects on the ground that such testimony is barred by the jurisdiction's physician-patient privilege. Should the objection be sustained?

No, because the physician-patient privilege is inapplicable for the defendant's statement

On December 6, the owner of an electronics store sent a written request to a computer manufacturer asking for the price of a certain laptop computer. The manufacturer sent a written reply with a catalog listing the prices and descriptions of all of his available computers. The letter stated that the terms of sale were cash within 30 days of delivery. On December 14, by return letter, the store owner ordered the computer, enclosing a check for $4,000, the listed price. Immediately on receipt of the order and check, the manufacturer informed the store owner that there had been a pricing mistake in the catalog, which should have quoted the price as $4,300 for that computer. The store owner refused to pay the additional $300, arguing that his order of December 14 in which the $4,000 check was enclosed was a proper acceptance of the manufacturer's offer. In a suit for damages, will the manufacturer prevail?

No, because the store owner's 12/14 letter was a proper acceptance of the Manufactures offer Price quotations may be considered as offers if given in response to a specific inquiry. The courts will look to the surrounding circumstances, and here a court would probably determine that the catalog that the manufacturer sent was an offer because it was sent in response to the store owner's specific inquiries about prices on a specific computer and it included delivery terms and conditions of sale. although the letter called for payment in cash, tender by check is sufficient unless the seller demands legal tender and gives the buyer time to obtain cash. Moreover, because the contract called for payment within 30 days of delivery, even if the check was not sufficient, the store owner still had time under the contract to obtain cash.

An officer on routine patrol noticed a flashlight moving within a darkened house and stopped to investigate. The suspect, who had broken into the home to steal valuables, caught sight of the patrol car, dropped the bag of valuables as he was about to carry them out of the house, and tried to sneak out the back way. The officer saw him sneaking out and seized him. The suspect, who had a lock-picking device in his possession, pulled out two $100 bills from his wallet, stating that he did not take anything and would like to forget the whole thing. The officer took the money, stating that she would give him a break this time around, and let the suspect go. May the officer be charged with accessory after the fact for the lacerny and the burglary?

No, the officer is only charged with being an accessory after the fact for the burglary. the facts do not indicate that the officer knew that the defendant had committed larceny when she let him go. She had stopped him outside of the house, and because he apparently had none of the home's valuables in his possession, she had no reason to doubt his claim that he had not gotten anything from the house. Although she probably surmised that he had broken in with the intent to commit larceny, she had no way of knowing that he had completed the crime of larceny by carrying the bag of valuables almost out of the house. An accessory after the fact is one who receives, relieves, comforts or assists another knowing that he has committed a felony, in order to help the felon escape arrest, trial or conviction. The crime committed by the principal must have been completed at the time aid is rendered.

In comparative negligence, does the defendant's willful and wanton conduct eliminate the plaintiff's comparative negligence?

No, the plaintiff's negligence will still be considered

The defendant discovered that his friend had hit and killed a pedestrian while driving that afternoon, and that he had fled from the scene of the crime before the police arrived. To keep his friend out of trouble, the defendant fixed all the dents in the car caused by the collision and had the vehicle painted a different color. The friend, distraught about hitting and killing someone, eventually turned himself in and told the police what he had done and what the defendant had done for him. The defendant was charged as an accomplice to vehicular manslaughter in a state that follows the modern trend regarding accomplice liability. How should the defendant be found?

Not guilty, because he only helped his friend after the crime was already committed An accomplice is one who with the intent that the crime be committed, aids, counsels, or encourages the principal before or during the commission of the offense. An accessory after the fact is one who receives, relieves, comforts or assists, another knowing they committed a felony and has committed a separate crime from the principal

A softball coach was charged with sexual assault of one of the players on the team. Three days before the start of trial, the prosecutor receives evidence that the coach sexually assaulted other players in the past. The coach was never criminally charged in connection with these incidents. The prosecutor immediately discloses the evidence to defense counsel. What may the prosecutor do with this evidence?

Offer it at trial as substantive evidence in the prosecution's case in chief Evidence of Defendant's prior acts of sexual assault are admissible in a criminal case in which the defendant is accused of sexual assault. Generally this evidence must be disclosed to Defendant within 15 days before trial, the court may waive that requirement on a showing of good cause, since the prosecutor just discovered these allegations the court may find good cause. And it would be up to the defense counsel to seek delay on the required notice.

Several members of a small terrorist group are on trial in federal court for conspiring to bomb a military installation. The prosecution would like to introduce the testimony of a military guard at one of the installation's gates. The guard had been present when a bomb that was being planted by a member of the group had exploded prematurely. The guard will testify that she ran over to administer first aid to the member, who in great pain told her that his group was in the process of planting three other bombs in other areas of the military installation and was going to detonate them all at the same time to get publicity for their cause. The guard will also testify that the member disclosed the locations of the other bombs and the names of two other members of the group. The authorities were able to prevent the other bombings and arrest the other members of the group. The member died from his injuries. What is the best basis for allowing the guard to testify as to the member's statements?

Statement against interest Under the Federal Rules, statements of a person, now unavailable as a witness, against that person's pecuniary, proprietary, or penal interest when made are admissible as an exception to the hearsay rule.

The defendant robbed a bank and fled in a getaway car driven by an accomplice, not realizing that one of the bundles of money he took had the serial numbers recorded and had a tiny tracking device attached to the wrapper. The bank's security consultant obtained portable tracking equipment and was able to trace the bundle of money to the defendant's house. The police were notified and they arrived at the defendant's house a few hours after the robbery. They knocked on the door, announced their presence, and saw someone matching the description of the robber in the hallway. They entered and arrested the defendant, and then conducted a protective sweep of the house for the accomplice, who they believed had a gun. They did not find him, but while checking a closet, they discovered several of the bundles of money from the bank and a gun the defendant had used in the robbery. The police also discovered two clear plastic bags of what appeared to be marijuana sitting on top of a dresser. They seized the money, the gun, and the two bags. Later testing confirmed that the substance in the bags was marijuana. The defendant was charged with the bank robbery and with possession of the marijuana. At a preliminary hearing, he moves to suppress introduction of the money, gun, and marijuana. How should the court rule?

Suppress all the evidence the police must have an arrest warrant to effect an arrest of an individual in his own home. There is no general "emergency" exception to the warrant requirement. While police officers in hot pursuit of a fleeing felon or trying to prevent the destruction of evidence may sometimes make a warrantless search and seizure, the burden is on the government to show that one of those exceptions applies. Here, the police did not arrive at the defendant's house in hot pursuit of the defendant, and there was no indication that the defendant might be destroying the money or other evidence; i.e., there were no circumstances precluding them from keeping the house under surveillance while they obtained a warrant. Hence, the arrest was unconstitutional. Because an arrest constitutes a seizure under the Fourth Amendment, the exclusionary rule applies, and evidence that is the fruit of the unconstitutional arrest may not be used against the defendant at trial. Here, all of the evidence was seized without a warrant, and none of the other exceptions to the warrant requirement are applicable.

On January 1, a car salesman offered to sell an antique car to a collector for $35,000 cash on delivery. The collector paid the car salesman $100 to hold the offer open for a period of 25 days. On January 4, the collector called the car salesman and left a message on his answering machine, asking him whether he would consider lowering the price to $30,000. The car salesman played back the message the same day but did not reply. On January 9, the collector wrote the car salesman a letter, telling him that he could not pay more than $30,000 for the antique car, and that if the car salesman would not accept that amount, he would not go through with the deal. The car salesman received this letter on January 10 and again did not reply. The car salesman never heard from the collector again. When did the offer that the car salesman made to the collector on January 1 terminate?

Terminate on 1/25 when the 25 day option expired An option is a distinct contract in which the offeree gives consideration for a promise by the offeror not to revoke an outstanding offer. The collector paid the car salesman $100 to hold the offer open for a period of 25 days, and the offer could not be terminated before that time, not even by the offeree (here the collector). Nor did the offer survive the option period because the option specifically identified how long the offer would be open. unequivocal words of rejection by the offeree will not extinguish an option, absent detrimental reliance on the part of the offeror, which was not the case here.

An owner obtained a loan of $60,000 from a bank in exchange for a promissory note secured by a mortgage on his land, which the bank promptly and properly recorded. A few months later, the owner obtained another loan of $60,000 from a lender, in exchange for a promissory note secured by a mortgage on the land, which the lender promptly and properly recorded. Subsequently, the owner sold the land to a buyer for $150,000 and conveyed a warranty deed. The buyer expressly agreed with the owner to assume both mortgages, with the consent of the bank and the lender. A few years later, the bank loaned the buyer an additional $50,000 in exchange for an increase in the interest rate and principal amount of its mortgage on the land. At that time, the balance on the original loan from the bank was $50,000. Shortly thereafter, the buyer stopped making payments on both mortgages and disappeared. After proper notice to all appropriate parties, the bank instituted a foreclosure action on its mortgage, and purchased the property at the foreclosure sale. At that time the principal balance on the lender's mortgage loan was $50,000. After fees and expenses, the proceeds from the foreclosure sale totaled $80,000. Assuming that the jurisdiction permits deficiency judgments, which of the following statements is most accurate?

The bank keeps $50,000, the lender is entitled to $30,000, and only the lender can proceed personally against the owner for its deficiency. The bank's original mortgage has priority in the proceeds, followed by the lender's mortgage, and only the lender can proceed against the owner because the bank modified its mortgage after the owner had transferred to the buyer. Generally, the priority of a mortgage is determined by the time it was placed on the property, and the proceeds of a foreclosure sale will be used to pay off the mortgages in the order of their priority. However, if the landowner enters into a modification agreement with the senior mortgagee, raising its interest rate or otherwise making the agreement more burdensome, the junior mortgage will be given priority over the modification. Thus, if the first mortgage debt is larger because of the modification, the second mortgage gains priority over the increase in the debt. Here, the bank and the buyer modified the original mortgage by increasing the principal amount and the interest rate. This modification is not given priority over the lender's mortgage, and foreclosure proceeds will not be applied against it because the senior lender's mortgage was not fully satisfied from the proceeds. With regard to the deficiency, the owner is liable to the lender because when a grantee signs an assumption agreement, becoming primarily liable to the lender, the original mortgagor remains secondarily liable on the promissory note as a surety. Here, the buyer assumed the lender's mortgage and became primarily liable; however, the owner remained secondarily liable as surety and can be required to pay off the rest of the lender's mortgage loan. On the other hand, the owner will not be liable to pay off the balance of the bank's loan, because when a mortgagee and an assuming grantee subsequently modify the original obligation, the original mortgagor is completely discharged of liability. The owner had nothing to do with the modification agreed to by the bank and the buyer that increased the amount of the mortgage debt, and will not be even secondarily liable for that amount.

A landlord leased office space to a business owner for five years, ending on November 1, reserving a yearly rent of $24,000, payable monthly. On October 1 of the fifth year, the business owner notified the landlord that he was preparing to move, but would greatly appreciate if the landlord could extend the lease for a month or two. On October 10, the landlord wrote to the business owner that she thought they could reach a satisfactory arrangement, but did not hear back from the business owner. The business owner did not vacate the office until November 20. On November 30, the landlord received a check from the business owner in the amount of $1,333 for "November's rent" and a note that he had vacated the premises. If the landlord brings an action against the business owner for additional rent, how will the court rule?

The business owner is bound to a year-to-year tenancy, because he did not vacate the premises until November 20. When a tenant fails to vacate the premises after the termination of his right to possession, the landlord may: (i) treat the hold-over tenant as a trespasser and evict him; or (ii) bind the tenant to a new periodic tenancy. The terms and conditions of the expired tenancy apply to the new tenancy. At least in commercial leases, the new tenancy will be year-to-year if the original lease term was for one year or more. Here, the businessman was a tenant for years because his lease was for a five-year fixed period of time. A tenancy for years ends automatically on its termination date. Therefore, as of November 1, the business owner became a hold-over tenant and the landlord had a right to bind him to a new periodic tenancy. Because the original lease was for more than one year, the business owner may be held to a year-to-year tenancy, at the stipulated rent of $24,000 per year.

A businesswoman entered into a written contract with a general contractor to build a studio and broadcast transmitter for $3 million by July 1. Among his tasks, the contractor was to install underground cables and fiberoptic lines necessary to broadcast. When digging the deep trench necessary to lay the conduit containing the fiberoptic lines, the contractor encountered a stretch of extremely soggy soil. This was an indication that an offshoot of the nearby city's aquifer underlay the property. This was not indicated on any of the geological survey maps available in the office of the county recorder of deeds. The contractor told the businesswoman that it would cost an additional $50,000 to lay the conduit through that stretch of soil. The businesswoman had already launched an advertising campaign indicating that the station would begin broadcasting on July 4, which was rapidly approaching. Therefore, when the contractor threatened to quit the job without the additional $50,000, the businesswoman reluctantly agreed orally to the contractor's demand as long as he promised that all of the work would be completed by the middle of June. The contractor agreed, proceeded to lay the conduit, and completed building the studio and transmitter by June 15. The businesswoman paid the contractor $3 million, but when the contractor demanded $50,000 more, she refused to pay it. The contractor sues the businesswoman for the $50,000. Who will prevail?

The contractor because the modification was supported by consideration Generally, a modifying agreement must be mutually assented to and supported by consideration. In most cases, consideration is found to be present in that each party has limited her right to enforce the original contract as is. Generally, if a modification will benefit only one of the parties, it may be unenforceable without some consideration being given to the other party. If, however, a promisee has given something in addition to what he already owes in return for the promise he now seeks to enforce, or has in some way agreed to vary his preexisting duty, there is consideration. Here, the businesswoman agreed to pay the contractor an additional $50,000 and the contractor agreed to complete the work early—by mid-June instead of July 1. Thus, there was sufficient consideration to support the modification.

On February 1, a national department store chain entered into a written agreement with a canoe manufacturer providing that the manufacturer would sell the department store any quantity of 16-foot aluminum canoes that the department store desired at a price of $250 per canoe, deliveries to be made 30 days after any order. The agreement was signed by authorized agents of both parties. On March 1, the department store sent the manufacturer an order for 500 canoes to be delivered in 30 days. The manufacturer immediately e-mailed the department store a confirmation of the order. Ten days later, the department store sent the manufacturer an order for an additional 500 canoes, to be delivered in 30 days. Five days after receiving the department store's second order, the manufacturer e-mailed the department store and explained that a large sporting goods chain was willing to purchase all of the manufacturer's output of 16-foot canoes at $275 per canoe and that the manufacturer would be unable to fill any of the department store's orders. The department store found another canoe manufacturer willing to provide it with 16-foot aluminum canoes for $280 per canoe and on April 15 filed an action against the manufacturer seeking damages for the manufacturer's failure to deliver the 1,000 canoes ordered. How should the court rule?

The department store is entitled to cover damages of $30 per canoe only for 500 canoes but is not entitled to any damages for breach of the duty of good faith. The original "agreement" between the parties was nothing more than an invitation seeking offers. It did not create a contract between the department store and the manufacturer because it was illusory—an agreement to buy only what is desired is not consideration. The "agreement" probably does not even qualify as an offer. An offer must express a commitment to conclude a bargain on the offered terms. Absent some quantity limitation, a court would probably find the "agreement" here too vague to constitute an offer; otherwise, the manufacturer could be committing itself to sell more canoes than it can supply. Thus, the department store's first order will be construed as an offer, and the manufacturer's confirmation will be construed as an acceptance of the offer, thus creating a contract.

A department store buyer and a manufacturer of food processors entered into a written contract whereby the manufacturer would sell to the buyer 50 of its top-of-the-line models for $100 each. When the delivery arrived on May 15, several days early, the buyer noticed that the food processors were a different model that did not have all of the features as the top-of-the-line model that was ordered. The buyer contacted the manufacturer and told him that he was rejecting the food processors that were delivered to him and expected the manufacturer to send 50 top-of-the-line models immediately. The manufacturer replied that because of a backlog of orders that had not yet been filled, the top-of-the-line models could not be delivered until August 15. Because the department store had contracted with a restaurant to deliver three top-of-the-line models by May 31, the buyer delivered three of the nonconforming food processors along with a promise to replace them with three top-of-the-line models in mid-August. The buyer returned the remaining food processors to the manufacturer. How much could the department store recover from the manufacturer for the three food processors that it delivered to the restaurant?

The difference between the market price of the top of the line models and the existing food processors actual value The acceptance did not waive its right to collect for damages The damages are measured based on market value rather than cost. The agreement was to accommodate the department store only and was not the foreseeable by the manufacture

A proud grandfather who planned to take pictures of his grandson's graduation purchased a camera from a camera store. He used the camera on several occasions over the next few weeks without incident, but when he used it on the day before his grandson's graduation, it caught fire and exploded, burning him and destroying an expensive coat he was wearing. Although the grandfather was in a great deal of pain because of his injuries, he insisted on attending his grandson's graduation. However, because he no longer had a workable camera, the grandfather hired a professional photographer to take pictures of the special day. In a breach of warranty action, which of the following represents the most that the grandfather may recover?

The difference between the value of the camera accepted and the value if it had been as warranted, medical costs for treating the burns, and the cost to replace the coat. When the buyer accepts goods that turn out to be defective he may recover as damages any loss resulting in the normal course of events from breach which includes the difference between the value of the goods accepted and the value they would have had it they had been as warranted, plus incidental and consequential damages. The cost of hiring the photographer is not foreseeable, the seller was not told of the particular requirements and needs of the camera

A driver and his passenger were involved in an automobile accident when the driver ran a red light and crashed into another car. Due to a manufacturing defect in the automobile's airbag system, the passenger side airbag did not deploy. The passenger was killed on impact. The passenger's estate brought suit against the driver and the airbag's manufacturer. At trial it is established that the driver was negligent in running the red light. What effect would such proof have on the claim of the passenger's estate against the airbag manufacturer? A) It would reduce recovery by the estate if the action against the manufacturer is based on negligence.It would reduce recovery by the estate if the action against the manufacturer is based on negligence. B) It would bar recovery by the estate if the trier of fact finds that the driver was the sole legal cause of the passenger's death.It would bar recovery by the estate if the trier of fact finds that the driver was the sole legal cause of the passenger's death. C) It would bar recovery by the estate if it is shown that the driver is the sole legal heir of the passenger's estate.It would bar recovery by the estate if it is shown that the driver is the sole legal heir of the passenger's estate. D) It would have no effect on recovery by the estate as long as the action against the manufacturer is based on strict liability.

The driver's negligence would bar recovery if it was the sole legal cause of the passenger's death. Regardless of the theory that the plaintiff is using in a products liability action, actual and proximate cause must be established. If the driver's negligence is the sole legal or proximate cause of the passenger's death, it would preclude the estate's suit against the airbag manufacturer because the defect was not a legal cause of the passenger's death.

Novation

occurs where a new contract substitutes a new party to receive benefits and assume duties that had originally belonged to one of the original parties under the terms of the old contract. A novation discharges the old contract. A novation will be found when there is (i) a previous valid contract; (ii) an agreement among the parties, including the new party to the new contract; (iii) the immediate extinguishment of contractual duties as between the original contracting parties; and (iv) a valid and enforceable new contract. Here, the facts do not state or otherwise indicate that a new contract was created and the duties under the old contract were discharged.

How long does a party have for the court to relieve a final judgment that was based on fraud or misrepresentation?

one year after trial

Statutory redemption is the right of a mortgagor to recover the land after the foreclosure sale has occurred, usually by paying

The foreclosure sale price AFTER the foreclosure sale

A landlord in a small city leased three adjacent single-family homes on a year-to-year basis. When one of them became vacant, the landlord placed an ad in the local paper stating: "Two-bedroom home available for rent. No children, please." The landlord included the request for no children because the landlord's other two homes on either side of the vacant one were leased by persons who had complained about the noise from the previous tenant's children. A parent with two children who sought to lease the unit was refused by the landlord despite meeting all of the other leasing requirements. The landlord owned only those three homes and so could not offer the parent any other accommodations. Which of the following statements is correct under the federal Fair Housing Act?

The landlord's expressed preference for lessees without children violated the act The Fair Housing Act prohibits advertisements that indicate any preference, limitation, or discrimination based on race, color, religion, sex, disability, familial status, or national origin. This restriction applies even when the dwelling itself is otherwise exempt from the other provisions of the Act. Here, advertising for no children constitutes discrimination based on familial status, which is prohibited by the Act.

A seller entered into a written contract to sell a tract of land to a buyer. The buyer was to pay $1,500 per month for five years, at which time the seller would deliver a warranty deed. The contract was silent as to the quality of title to be conveyed. After making 12 payments, the buyer discovered that a neighbor had an easement of way over the land, which was not discussed at the time the seller and buyer entered into the contract. The neighbor had not used the easement over the previous year because she had been out of the country. On the basis of the easement, the buyer wishes to cancel the contract. Which party is more likely to prevail?

The seller because the buyer has no basis to rescind the contract Where an installment land contract is used, the seller's obligation is to furnish marketable title when delivery is to occur, e.g., when the buyer has made his final payment. Thus, a buyer cannot withhold payments or seek other remedies on grounds that the seller's title is unmarketable prior to the date of promised delivery. Here, there is a valid easement on the property (see below), but the seller has four years in which to cure this defect. Thus, the buyer cannot yet rescind on grounds that title is unmarketable. The seller has until time of defective to cure the deed.

A fee simple owner of a restaurant provided in his will that the property should go on his death "in fee simple to my friend, but if during my friend's lifetime my son has children and those children are alive when my friend dies, then to said living children." When the owner died, the friend took over the restaurant. If the son has children and one or more of them are alive when the friend dies, who will take title to the restaurant at that time?

The son's Children because their interest will vest within a life being plus 21 years. The friend has a fee simple subject to an executory limitation

A seller conveyed her residential city property to a buyer by a general warranty deed. On taking possession of the property, the buyer discovered that the garage of his neighbor encroached six inches onto his property. If the buyer wishes to compel the seller to assist him in a suit against the neighbor, which of the following covenants may he rely on to do so?

Warranty and further assurances Under the covenant of warranty, the grantor agrees to defend, on behalf of the grantee, any lawful or reasonable claims of title by a third party, and to compensate the grantee for any loss sustained by the claim of superior title. The covenant for further assurances is a covenant to perform whatever acts are reasonably necessary to perfect the title conveyed if it turns out to be imperfect. These covenants are "continuous" (run with the land) and require the grantor to assist the grantee in establishing title.

A landlord brought suit against a tenant in federal court for overdue rent payments on a commercial lease. The landlord sought to recover on the six rent installments that were past due and unpaid at the time of the suit. The landlord won the case, and judgment was entered in her favor. The lease has an acceleration clause that states that all future rent payments become due if the tenant falls behind three months or more. The landlord now files suit against the tenant for the remaining rent payments. The tenant moves to dismiss, asserting that the landlord's claim is barred by claim preclusion (res judicata) principles. Should the tenant's motion to dismiss be granted?

Yes because the suits arise out of the same transaction or occurrence The plaintiff's suit is unlikely to be successful because the two suits arise out of the same transaction or occurrence. Claim preclusion (res judicata) requires that (i) a valid, final judgment on the merits was entered in the first case; (ii) the cases were brought by the same claimant against the same defendant; and (iii) the same cause of action is involved in the later lawsuit. Generally, a claimant is required to assert all causes of action arising out of the same transaction or occurrence that is the subject matter of the claim. In the situation of installment payments, the claimant is required to sue on all installments due at the time of the suit. If there is an acceleration clause, the claimant must sue for all installments. Here, the plaintiff should have sued for all installments in the first lawsuit. Because she did not, her second suit is barred by claim preclusion.

A federal statute designed to stop organized crime enumerated certain activities as crimes and provided that, in addition to charging these activities as the crimes they constitute, the activities would also constitute the criminal act of intentional furtherance of the goals of organized crime. Among the enumerated activities was the interstate distribution of cocaine. The statute's constitutionality has been upheld by the Supreme Court. The defendant was arrested by federal agents after having driven a truck containing cocaine from Florida to Illinois, where he delivered his illicit cargo as directed. At trial, the defendant is convicted of interstate distribution of cocaine in violation of federal law, and convicted of a violation of the federal statute above. How may the defendant be sentenced under both statutes?

Yes, Double jeopardy does not prohibit the imposition of cumulative sentences for two or more statutorily defined offenses specifically intended by the legislature to carry separate punishments, even though constituting the "same" crime under the Blockburger test (i.e., each offense does not require proof of some additional fact that the other does not) when the punishments are imposed at a single trial. Absent a clear intention, it is presumed that multiple punishments are not intended for offenses constituting the same crime under Blockburger. Here, it is clear that Congress, in enacting the statute, intended that certain offenses, such as interstate distribution of cocaine, be subject to separate punishments

An environmental group, wishing to stop the issuance by a federal agency of a mining permit to a coal company, commences an action in federal court against the federal agency, seeking, among other things, a permanent injunction barring the issuance of the permit to the coal company. If the coal company seeks to join the litigation as a matter of right, must the federal court grant the motion?

Yes, UNLESS the court concludes that the coal company's interest in getting the permit is adequately protected by the federal agency

The owner of a factory that uses widgets in making its product and a widget maker entered into negotiations over the telephone and, after a time, reached a general understanding that the factory owner would buy widgets from the widget maker. Following their conversation, the widget maker sent the factory owner a contract, which he had already signed, agreeing to sell 1,000 widgets to the factory owner for a total contract price of $10,000. Upon receipt of the contract in the mail, the factory owner signed the contract and deposited an envelope containing the contract in the mailbox located in front of his workplace. Before the widget maker received the contract, the factory owner had a change of heart. He telephoned the widget maker and told him that he could not afford to buy the widgets he had ordered, and he was "not interested in that contract we talked about." The widget maker replied, "That's all right, I understand. Maybe we can do business some other time." The next day, the signed contract was delivered to the widget maker's office. The widget maker, also having had a change of mind, decided that he wanted to enforce the contract. Is the contract enforceable against the factory owner?

Yes, acceptance occurred prior to revocation mailbox rule applies here. Acceptance by mail creates a contract at the moment of posting, properly addressed and stamped, unless the offer stipulates that acceptance is not effective until received, or an option contract is involved. If the offeree sends an acceptance and then rejects the offer, the mailbox rule applies; i.e., a contract is created upon dispatch of the acceptance. Because no option contract is involved here, and the widget maker's offer did not state that the factory owner's acceptance would be effective only when received, his acceptance was effective the moment he placed the envelope containing the contract in the mailbox. His attempt to reject occurred after acceptance took place. Thus, a valid contract was formed and the widget maker may enforce it.

Based on recommendations of a state commission studying the effect of pornographic films on violent criminal activity, a state adopted legislation banning films intended for commercial distribution that appealed as a whole to the prurient interest in sex of the average person in the community, portrayed sex in a patently offensive way to citizens of the state, and which a reasonable person in the United States would find had no serious literary, artistic, political, or scientific value. In ruling on a constitutional challenge to the legislation from a film distributor in the state who was convicted of distributing films in violation of the legislation, will the federal court likely find the legislation to be constitutional?

Yes, because it uses a national person standard for determining the social value of the work legislation here is constitutional because it uses a reasonable person standard, rather than a community standard, for determining the value of the work Obscenity, which is not protected speech under the First Amendment, is defined by the Supreme Court as a description or depiction of sexual conduct that, taken as a whole, by the average person, applying contemporary community standards, appeals to the prurient interest in sex, portrays sex in a patently offensive way, and—using a national reasonable person standard—does not have serious literary, artistic, political, or scientific value.

A landlord entered into a written lease of a bakery for a term of 25 years with a baker. The parties agreed to a right of first refusal if the bakery was offered for sale during the term of the lease. The lease also permitted assignments and subleases on notice to the landlord. Three years later, the baker retired and, after notifying the landlord, transferred the lease to a chocolatier. Twenty-one years later, the landlord entered into a contract with a buyer for the sale of the bakery for $100,000. The landlord had informed the buyer of the lease but had forgotten about the right of first refusal. When the chocolatier learned of the sale to the buyer, she informed both the landlord and the buyer that she wanted to exercise her option and was prepared to purchase the bakery for the contract price. The jurisdiction's Rule Against Perpetuities is unmodified by statute. Can the chocolatier enforce the option?

Yes, because the option touches and concerns the leasehold estate The chocolatier can enforce the option to purchase because it is a covenant that runs with the land. When a tenant makes a complete transfer of the entire remaining term of his leasehold interest, it constitutes an assignment. The assignee and the landlord are then in privity of estate, and each is liable to the other on all covenants in the lease that run with the land. The covenant runs with the land if the original parties so intend and the covenant "touches and concerns" the leased land, i.e., burdens the landlord and benefits the tenant with respect to their interests in the property. Here, the transfer of the lease to the chocolatier was an assignment, making all covenants in the lease that run with the land enforceable by the assignee. The right of first refusal burdens the landlord's power of alienation over the bakery, and there is nothing to indicate that the parties intended the option to be personal to the baker. Hence, the chocolatier can enforce the option and purchase the property.

A local news station broadcast a live interview with a bystander about his views concerning the state of local education. The bystander responded by saying that the principal of his daughter's high school had been embezzling school funds for years. The principal saw the telecast and also recorded it. He sued the owner of the station for defamation. At trial, the principal sought to testify to the defamatory statement made in the interview. Will the principal's testimony likely be held to be admissible?

Yes, because the principal personally saw the interview on TV Admissible because he saw the news unless there is a specific rule excluding the evidence. This is not hearsay because it is a defamation case and the evidence is offered to show that the actionable statement was made Best evidence does NOT apply because the recording is not an essential repository of the facts recorded.

During a presidential campaign, a candidate's campaign manager secretly engaged in activities that may have violated both state and federal laws. After the candidate was elected President, the FBI investigated the manager's activities as well as whether the President was involved. After the campaign manager was indicted in federal court, but before trial, the President granted a blanket pardon to the campaign manager for "all federal crimes that may have been committed in the past 20 years." Is the pardon valid?

Yes, the pardon power is an unqualified power (except for impeachment) The pardon power is superior to laws found in statutes

To reduce deer overpopulation in state forests, state Blue adopted a statute allowing anyone with a valid deer hunting license from any state to hunt deer within state Blue. The act also imposed a $0.25 per pound tax on each deer killed within the state. Funds from the tax were earmarked to support state forest land. State Red is adjacent to state Blue and also has an overabundance of deer. To encourage hunting, state Red does not impose a tax on deer taken from its forests. A hunter who is a resident of state Red and who is licensed to hunt there earns his living by supplying wild game to several high-end restaurants in state Red. While legally hunting deer within state Red, the hunter inadvertently crossed the state line and killed a deer in state Blue. Upon hearing the hunter's shot, a state Blue game warden arrived at the scene, approximated the weight of the kill, and handed the hunter a tax bill based on the approximation. The bill provided a method for challenging the approximated weight of the deer, but the hunter refused to pay any tax on his kill. He instead filed suit in federal court to enjoin collection of the state Blue tax on constitutional grounds. Is the tax constitutional?

Yes, the tax is valid under the commerce clause Valid under the commerce clause if: (i) the tax does not discriminate against interstate commerce (ii) there is a substantial nexus b/w the activity taxed and the taxing state (iii) the tax is fairly apportioned (iv) the tax fairly relates to services or benefits provided by the states Here the tax applies equally to in and out of state residents,

Rules Enabling Act

a Federal Rule is valid if it deals with "practice or procedure" and does not "abridge, enlarge, or modify" a substantive right. ex: federal procedural rule that is on point [Fed. R. Civ. P. 48], which requires jury verdicts to be unanimous, unless the parties agree otherwise.

A plaintiff sued a defendant for damages suffered when a load of bricks fell off the defendant's truck directly in front of the plaintiff while she was driving on a highway. The plaintiff charged that the defendant was negligent in supplying his truck with a defective load chain clamp, which helped tie the load to the bed of the truck, and in failing to secure the load properly on the truck. The plaintiff calls a witness who testifies that he was formerly employed as a truck driver and is an acquaintance of the defendant. The witness further testifies that immediately prior to the accident he had coffee with the defendant at a cafe, and mentioned to the defendant that the tie chains holding the load of bricks looked kind of loose. Assuming proper objection by the defendant's attorney, how should the court rule on the admissibility of such testimony?

admissible nonhearsay The statement by the witness is not being offered to prove the truth of the matter asserted therein and thus is not hearsay. in a negligence case, where knowledge of a danger is at issue, a person's warning statement is admissible for the limited purpose of showing knowledge or notice on the part of a listener. Here, one of the theories of recovery underlying the plaintiff's lawsuit is that the defendant negligently failed to secure the load. Therefore, the plaintiff must show that the defendant either knew or should have known that the load was not properly secured. Consequently, the witness's statement that the chains looked loose is admissible to show that the defendant had notice of the possible danger. If this same out-of-court statement were offered to show that its contents were true (i.e., that the chains were in fact loose), then it would constitute hearsay, but because the statement is offered to show notice to the defendant of a possible danger, it is nonhearsay

The Interlocutory Appeals Act

permit a review of an interlocutory order, but it is discretionary, and may be available only when (i) the trial judge certifies that the interlocutory order involves a controlling question of law, as to which there is substantial ground for difference of opinion, and immediate appeal from the order may materially advance the ultimate termination of the litigation; and (ii) the court of appeals then agrees to allow the appeal. A party obtaining such a certificate from the trial judge must, within 10 days, apply to the court of appeals, where two out of three judges must agree to hear the appeal

Interlocutory orders and exceptions to allow immediately appeal

rulings that trial judges make during the course of pretrial proceedings and trials that do not completely resolve the case. Therefore, they are not final. As a result, interlocutory orders are typically not immediately reviewable on appeal until a final order is made, unless they meet one of the exceptions permitting an appeal as of right (i.e., orders granting injunctions; orders appointing a receiver; orders in admiralty cases finding liability but leaving damages to be assessed later; patent infringement orders where only an accounting is ordered; and orders affecting or changing possession of property).

A statute in the jurisdiction, which was enacted with the express purpose of preventing public employees from taking advantage of the status of undocumented immigrants, made it a felony to accept money or other benefits in exchange for issuing a state identification card. During an undercover investigation, an undocumented immigrant was recorded offering $500 to a clerk in exchange for issuance of a card. The clerk agreed to the deal and later that day exchanged the card for the money, after which both parties were arrested. Can the clerk and the immigrant be charged with violating the statute?

the clerk can be charged with violating the statute however the immigrant cannot, the statute was designed to protect the immigrant. Although the immigrant would normally be an accomplice because of the legislative attempt to protect him.

To finance the purchase of a vineyard, a vintner borrowed $500,000 from a bank, secured by a mortgage on the vineyard. Due to a clerical error, the bank's mortgage was not immediately recorded. Six months later, the vintner borrowed $10,000 from a creditor, also secured by a mortgage on the vineyard. The creditor immediately recorded its mortgage. The following week, the bank discovered its error and recorded its mortgage. Subsequently, the vintner defaulted on her payments to the bank. The bank instituted foreclosure proceedings but did not join the creditor in the action. A buyer purchased the property at the foreclosure sale. A statute of the jurisdiction provides, "No conveyance or mortgage of real property shall be good against subsequent purchasers for value without notice unless the conveyance is recorded." If a court finds that the buyer took title subject to the creditor's mortgage, what is the most likely reason? Press Enter or Space to submit the answer

the creditors mortgage was senior to the bank's PMM If a court finds that the buyer took title subject to the creditor's mortgage, it will be because the creditor's mortgage was senior to the bank's purchase money mortgage ("PMM"). A PMM is a mortgage typically given to a third-party lender, who is lending the funds to allow the buyer to purchase the property. A PMM, whether recorded or not, has priority over mortgages, liens, and other claims against the mortgagor that arise prior to the mortgagor's acquisition of title. However, PMM priority is subject to being defeated by subsequent mortgages or liens by operation of the recording acts. Here, the creditor's mortgage has priority over the bank's PMM under the jurisdiction's notice statute because the creditor had no notice of the bank's interest at the time of the loan. Because foreclosure does not affect any interest senior to the mortgage being foreclosed, the buyer takes title subject to the creditor's mortgage.

A vintner divided his vineyard into two parcels, drawing the boundaries so that the single well that had irrigated the entire vineyard fell on the border of the two properties. The vintner then conveyed the eastern parcel to his friend by a deed that contained the following covenant: "If the well located on the boundary of the eastern and western parcels continues to be used for irrigation purposes and becomes in need of repair or replacement, the grantee, his heirs, and assigns and the grantor, his heirs, and assigns each promise to pay one-half of the cost of such repair or replacement. This covenant shall run with the land." The deed from the vintner to the friend was not recorded, and the vintner did not record a copy of the deed with the records for the western parcel. The friend later sold the eastern parcel to a farmer. The farmer's deed did not contain the covenant about the well. After 15 years of use by the owners of both the eastern and western parcels, the well began to fail. The farmer took it upon himself to have the well repaired at a cost of $30,000. About two weeks later, the farmer discovered the deed from the vintner to the friend in some old files. By this time, the western parcel had passed to the vintner's son by inheritance and again to the son's daughter by inheritance from the now-deceased son. The daughter knew nothing of the covenant concerning the well. The farmer presented the daughter with the bill for the well repair with a copy of the vintner/friend deed and a note that said he expected to be reimbursed for $15,000. The daughter refuses to pay, and the farmer sues. The jurisdiction has a 10-year statute of limitations for acquiring property by adverse possession, and the following recording statute: "Any conveyance of an interest in land shall not be valid against any subsequent purchaser for value, without notice thereof, unless the conveyance is recorded." For whom is the court most likely to rule?

the farmer because the covenant runs with the land. To be bound the parties must agree 1. the covenant run with the land 2. OG parties have horizontal privity -The original parties were in horizontal privity because at the time the vintner entered into the covenant, he and the friend shared an interest in the land independent of the covenant—as grantor and grantee. 3. the succeeding party have vertical privity -daughter is in vertical privity with the vintner because she holds the entire interest in the western parcel held by the vintner. 4. the covenant must touch and concern the land -The covenant touches and concerns the land because promises to pay money to be used in a way connected with the land are held to touch and concern the property. 5. the burden party must have actual or constructive notice of the covenant -While it is generally true that the owner of the burdened land must have notice, it should be remembered that the requirement is a function of the recording statute. (At common law, the covenant was enforceable in an action for damages regardless of notice; this was changed by the recording statutes.) However, because the daughter is a donee (an heir) and not a bona fide purchaser, she is not protected by the recording statute and thus is subject to the covenant even without notice.

A landowner validly conveyed a small office building to the Green Party "as long as they use it for operating quarters until the next presidential election." After the next presidential election, which was in three years, the building would go to a private organization that monitors and prepares comprehensive listings of gas prices throughout the country. A year after the conveyance, the landowner died, validly devising all of her property to her son. Although this jurisdiction is a common law jurisdiction with respect to all real property considerations, the state's probate laws provide that future interests or estates in real property may be passed by will or descent in the same manner as present or possessory interests. Last week, the Green Party and the gas monitoring organization joined together to sell the office building in fee simple absolute to a developer. The son filed suit to prevent the sale of the property to the developer. In this action, who should prevail?

the son, because he did not sign the contract of sale The Green Party's interest in the office building is a fee simple determinable because it lasts as long as the Party is using the building for operating quarters. However, the grant does not provide for the contingency of the Green Party ceasing to use the building as operating quarters before the next presidential election. This gap would be filled by a possibility of reverter retained by the landowner. Because the landowner passed that interest to her son in her will, there can be no contract to sell the property without his signature.


Kaugnay na mga set ng pag-aaral

Chapter 11: Childhood & Neurodevelopmental Disorders

View Set

2.6 social, political, economic effects of the industrial revolution

View Set

DD5 Medium Duty Engine Fuel System - Introduction - OEP61E

View Set

Infant & Child final quiz questions

View Set